X



トップページ数学
1002コメント357KB
大学学部レベル質問スレ 14単位目
レス数が1000を超えています。これ以上書き込みはできません。
0001132人目の素数さん
垢版 |
2020/07/15(水) 05:27:54.74ID:xmF7sJYz
大学で習う数学に関する質問を扱うスレ

・質問する前に教科書や参考書を読むなりググるなりして
・ただの計算は
http://wolframalpha.com
・数式の表記法は
http://mathmathmath.dote ra.net
・質問のマルチポストは非推奨
・煽り、荒らしはスルー

関連スレ
分からない問題はここに書いてね478
http://rio2016.5ch.net/test/read.cgi/math/1511604229/

※前スレ
大学学部レベル質問スレ 13単位目
https://rio2016.5ch.net/test/read.cgi/math/1577771353/
0002132人目の素数さん
垢版 |
2020/07/15(水) 11:24:39.89ID:JG4qV0Js
前スレの質問
999 132人目の素数さん[] 2020/07/15(水) 10:59:39.04 ID:GC4Nr3Jf
離散位相、密着位相という名前はなぜそうつけられたんですか?直感的な説明をお願いします。



例えば位相空間Xから実数Rへの連続写像を考えてみる

Xが離散位相ならば、この写像は完全に自由で良い
つまりXをバラバラに引きちぎってRに送れる

Xが密着位相ならば、この写像は1点に潰すしかない
つまりXの各点は全く離れたくないかのように振舞う
0003132人目の素数さん
垢版 |
2020/07/15(水) 14:19:56.96ID:Nn5BJat7
閉包=くっついてる でもいいな
密着位相で閉包は全部くっついてしまう
離散位相の閉包は全然くっつかない
0005132人目の素数さん
垢版 |
2020/07/15(水) 18:26:40.40ID:piuFlzQv
自分は数学科ではないのですが、「悪魔の証明」って、対偶を利用すれば可能ではないのですか?
論理記号で表現できないのですが。素朴な疑問。
0007132人目の素数さん
垢版 |
2020/07/15(水) 19:42:42.55ID:9v8EKDM9
類体論周りの代数的整数論って、
ガロア理論とか関数論とかと比べてあまり標準的ではない割に退屈な理論を勉強しないといけないですよね
例えば、デデキント環の理論とか付値の理論とか位相群上の積分とか、…
皆さんはこういう理論を勉強するときにどうやってモチベーションを維持していますか?
0008132人目の素数さん
垢版 |
2020/07/15(水) 19:57:02.09ID:jVLKyjwF
一つの方法としては群のコホモロジーからアプローチする
デメリットは和書にそういう本が殆どない
0009132人目の素数さん
垢版 |
2020/07/15(水) 20:17:01.82ID:JG4qV0Js
勉強できる人はみんな強いモチベーションや先人への盲信を持ってるように見える
それか賢い人は次々に登場する概念の必要性を瞬時に理解できるからかも知れないけど
0010132人目の素数さん
垢版 |
2020/07/15(水) 20:29:06.41ID:9v8EKDM9
「数論やるなら類体論くらい知っておかないと」みたいな感じで始めても全然モチベーションが維持できないんですよね
なんというか、類体論そのものに面白さを見出せないというか
向いていないのかな
0011132人目の素数さん
垢版 |
2020/07/15(水) 20:56:18.55ID:QT16sYkb
>>9
盲信も何も、数学者は普通に俺らより優秀じゃん
でも高校では数学ナンバーワンで自信満々で数学科に入ったのに心折られちゃった人は、その適応機制として数学者より自分のほうが正しいんだという思想に染まりやすいんだろうね
そんな根拠のない自信は持たないことが大事
0012132人目の素数さん
垢版 |
2020/07/15(水) 21:24:17.57ID:JG4qV0Js
歴史的な流れもあるかと思う
だから数学史を見ることで当時の問題意識などがわかって概念の理解しやすくなることもあるし
逆に現代的な統一的視点で見た方が分かりやすいこともある
当時の動機とは全く違う重要性が後に見つかって、そちらが現在では重視されていることもある
歴史の中の数学者は全員天才だけども概念や理論自体のあり方は絶対的なものではないはずで
もしかしたら全く別の体系・整備があったかもしれない
まあ、かといってそれを己で一から開拓するなんてのは難しいだろうから素直に歴史に従うべきだというのはあるが
0013132人目の素数さん
垢版 |
2020/07/15(水) 21:33:03.12ID:/Q3fMOpu
>>10
意味分からん
理解できずに先行けるわけない
微積の小手先線形の小手先を面白いと思っているだけでは何にもならないのと同じなのだがそれが分からないというのが分からない
0014132人目の素数さん
垢版 |
2020/07/15(水) 21:40:16.49ID:JG4qV0Js
まず数論に対してどんなモチベがあるかだね
類体論やらなくても解析的数論や組み合わせ的数論で出せる結果もあるだろうから
0015132人目の素数さん
垢版 |
2020/07/15(水) 21:48:01.82ID:9v8EKDM9
>>13
個人的には、「役に立つけど面白くない理論」と「役に立たないけど面白い理論」があると思っていて、
類体論は前者に属すると思っています
私は前者を学ぶモチベーションが維持できないんです
0016132人目の素数さん
垢版 |
2020/07/15(水) 22:26:09.94ID:kmlc9c1w
(0,1) 1/√(x(1-x^2))
この広義積分ってどんな関数で上から抑え込めばいけますかね?
それとも発散しますか?
0017132人目の素数さん
垢版 |
2020/07/15(水) 22:57:51.51ID:JG4qV0Js
x→√yと変数変換したら
ベータ関数B(x,y)使って
B(1/4,1/2)/2
になるから
=Γ(1/4)Γ(1/2)/(2Γ(3/4))=√πΓ(1/4)/(2Γ(3/4))
0018132人目の素数さん
垢版 |
2020/07/15(水) 23:39:35.15ID:JG4qV0Js
ガンマ関数の乗法公式使うと
=Γ(1/4)^2/√(8π)=2.62....

評価についていうなら
積分範囲をA=[0,1/2]とB=[1/2,1]で分けて考えれば
3/4≦(1+x)(1-x) (x∈A)
3/4≦x(1+x) (x∈B)
だから
∫1/√(x(1+x)(1-x))≦2/√3(∫_A 1/√x + ∫_B 1/√(1-x))
=2/√3(√2+√2)=4√2/√3=3.26....
で抑えられる
0019132人目の素数さん
垢版 |
2020/07/15(水) 23:49:17.59ID:JG4qV0Js
>>2
Ω={0,1} O(Ω)={φ,{1},Ω}という位相空間を用意すると

Xが離散位相 ⇔ XからΩへの連続写像は任意(バラバラ可能)
Xが密着位相 ⇔ XからΩへの連続写像は定値のみ(1点に潰すしかない)

と書けるか
0020132人目の素数さん
垢版 |
2020/07/16(木) 00:30:43.48ID:pfmavIEH
>>15
数学の本道に進まず
何にも成し遂げられないってことだよ?
0021132人目の素数さん
垢版 |
2020/07/16(木) 00:34:19.60ID:pfmavIEH
>>5
具体的に書いて
0023132人目の素数さん
垢版 |
2020/07/16(木) 00:58:07.78ID:UfJojVu6
>>21
質問は十分具体的だと思うけど
まさか可能かどうかを尋ねる質問に対して答えを具体的に書けと言ってる?
0024132人目の素数さん
垢版 |
2020/07/16(木) 01:02:51.05ID:pfmavIEH
>>23
悪魔の証明の具体例を書いてって言ってるだけ
0025132人目の素数さん
垢版 |
2020/07/16(木) 01:03:43.77ID:pfmavIEH
>>22
そうかね
0026132人目の素数さん
垢版 |
2020/07/16(木) 01:24:36.29ID:KbCXPG4/
数学上の話ならそもそも「悪魔の証明」なるものは存在しない
証明もしくは反証できるものそもそも悪魔の証明にならないし、証明も反証もできないものは単にその公理系から独立してるだけというスタンス
0027132人目の素数さん
垢版 |
2020/07/16(木) 01:54:06.05ID:i1fI3qOK
画像の定理を証明してください
多変量正規分布の最尤推定に使う定理らしいのですが、全く分かりません...
参考になるサイトでも構いませんので、よろしくお願いします


https://i.imgur.com/gptqgco.jpg
0028132人目の素数さん
垢版 |
2020/07/16(木) 03:11:37.52ID:zpHFCDP2
うーん、少なくともΣ^(-1)Aが対角化出来るものとして考えれば、その固有値たちを使って
(Π(λ_i)^n(-exp(λ_i)))^(1/2)|A|^(-n/2)
と書けて、これを最大にするには各iについて
(λ_i)^n(-exp(λ_i))
が最大になれば良くて、それはλ_i=nのときだから
Σ ^(-1)Aが単位行列のn倍のとき
つまりΣ=A/nのときと分かるけどね
0029132人目の素数さん
垢版 |
2020/07/16(木) 03:39:54.35ID:zpHFCDP2
あ、微分していけるわ
Σ^(-1)A=BとしておいてBの各成分b_ijの多変数関数と思って最大を求める
元の式は
(|B|^n(exp(-trB)))^(1/2)|A|^(-n/2)
と書けて、これは
|B|^n(exp(-trB))
を最大にすればよい
∂/∂b_ij |B|=(B^(-1))_ji|B|
∂/∂b_ij trB=δ_ij
などに注意して微分すれば
B^(-1)_ji=1/nδ_ij
という条件を得る
つまり
B^(-1)=E/n (Eは単位行列) よって Σ=A/n
0030132人目の素数さん
垢版 |
2020/07/16(木) 03:40:19.15ID:i1fI3qOK
ありがとうございます!
質問なのですが、|Σ|^(-1/2)の部分が|A|^(-1/2)になっているのは、どのような計算をしたのですか?
初歩的・筋違いな質問だったら本当にごめんなさい
0031132人目の素数さん
垢版 |
2020/07/16(木) 03:51:24.18ID:zpHFCDP2
>>30
まず>>28の式のexpのとこ誤植
(Π(λ_i)^n(-exp(λ_i)))^(1/2)|A|^(-n/2) 誤
(Π(λ_i)^n(exp(-λ_i)))^(1/2)|A|^(-n/2) 正

タイミング的にまだ>>29の方は見てないですかね
Σ^(-1)A=Bと置いたからΣ=AB^(-1)
|Σ|^(-n/2)=|B|^(n/2) × |A|^(-n/2)
となってAの部分が出てくる
上のλ_iバージョンの式も同じ理由で|A|が出てます
0032132人目の素数さん
垢版 |
2020/07/16(木) 04:00:52.68ID:i1fI3qOK
>>31
なるほど!!!!! それで|B|の方は左の括弧に組み込まれてるんですね!
完全に理解できました! 本当にありがとうございましたm(_ _)m
0033132人目の素数さん
垢版 |
2020/07/16(木) 05:03:10.20ID:zpHFCDP2
補足
上の証明でλ_i>0の範囲で最大を言ってるけど
正定値対称行列の積は対角化可能で固有値も全て正だから良さそう

F,G:正定値対称とするとそれらの積は
FG=√F(√G√F)^t(√G√F)√F^(-1)
とかけて
正定値対称な(√G√F)^t(√G√F)に相似だから
FGも対角化可能で固有値は全て正
0035132人目の素数さん
垢版 |
2020/07/16(木) 11:19:57.37ID:pfmavIEH
>>34
モーチベーションは面白いから
分からなければそこでお仕舞い
0036132人目の素数さん
垢版 |
2020/07/16(木) 11:22:35.32ID:q2BlT9bf
>>35
難しいだけで面白くない理論の場合はどうしてる?
>>7にあるような理論とか、測度論とか
0037132人目の素数さん
垢版 |
2020/07/16(木) 11:42:14.52ID:pfmavIEH
>>36
面白いが?
0038132人目の素数さん
垢版 |
2020/07/16(木) 11:46:50.46ID:q2BlT9bf
>>37
そっか…
面白くない理論に出会ったことがないか、
あるいはどんな理論でも面白いと思えるタイプの天才か
どっちにしても羨ましい
0040132人目の素数さん
垢版 |
2020/07/16(木) 17:15:03.10ID:4xC7cAJD
>>18
thank you so much!!
0041132人目の素数さん
垢版 |
2020/07/16(木) 18:04:11.43ID:UXbgSefO
フルヴィッツ・クーラントの楕円関数論の翻訳書ってどれくらいの予備知識があれば読めますか?
0043132人目の素数さん
垢版 |
2020/07/18(土) 23:34:51.48ID:GjWRz82I
R^nの開球U = { x | |x| < 1}からR^nの開集合への連続な全単射fがあったとき、
逆写像f^-1: f(U)→Uは連続ですか?
0047132人目の素数さん
垢版 |
2020/07/27(月) 01:01:55.17ID:7pRPm+5q
無限基数って全部アレフ_αっていう形で一意的に漏れなく表せれるのに
何で一々「κは無限基数である」みたいな表現するんかな
前々からずーーーーーっと思ってるんだが?
0048132人目の素数さん
垢版 |
2020/07/27(月) 01:34:20.93ID:H/LImTqu
整数は全部10進法で表せるのに
何でいちいち「nは整数である」みたいに書くのかね〜?
0049132人目の素数さん
垢版 |
2020/07/27(月) 01:55:01.51ID:68G4t9zX
>>48
すみません、どうやるのか分からないのですが
整数全部10進法で表してみてもらえますか?
0050132人目の素数さん
垢版 |
2020/07/27(月) 01:59:08.78ID:H/LImTqu
>>49
じゃあその前に無限基数全部アレフ_αで表して
0051132人目の素数さん
垢版 |
2020/07/27(月) 02:21:20.37ID:68G4t9zX
>>50
僕は>>47では無いのでそれもどうやるか分からないです
整数全部10進数で表せる方なら無限基数全部アレフ_αで表すこともできるので、それについても教えて頂けると助かります
0052132人目の素数さん
垢版 |
2020/07/27(月) 02:25:14.15ID:H/LImTqu
>>51
じゃあ助けないよ
0054132人目の素数さん
垢版 |
2020/07/27(月) 02:35:23.79ID:G3DlQJCx
どの整数も十進法で表すことができるが
すべての整数を十進法で表す為には
この掲示板には余白が足りない
0055132人目の素数さん
垢版 |
2020/07/27(月) 03:52:33.45ID:7pRPm+5q
>>48
ワロタww
一応お前の中では皮肉を頑張って言ってみたんだろうけど、
整数が10進法で表せる事といちいち「nは整数である」と断る事の関係性言ってみw
0057132人目の素数さん
垢版 |
2020/07/27(月) 04:20:19.15ID:7pRPm+5q
>>48
追加で課題
大体同じ事だけど、その皮肉を言う事によって>>47の何に問題があると言いたかったのかも言ってみ
0059132人目の素数さん
垢版 |
2020/07/27(月) 23:05:09.03ID:ofcl67c4
悔しくて死にそう
> ID:7pRPm+5q
0060132人目の素数さん
垢版 |
2020/07/27(月) 23:29:28.51ID:WzxAniYp
アホに因縁つけられたのに言い返してるだけで別に悔しい要素ないやろ
偉そうに言った割りに突っ込まれたら何も言い返せないID:H/LImTquが悔しくて死にそうというなら分かるけど
0063132人目の素数さん
垢版 |
2020/07/28(火) 00:30:09.89ID:JY7OMgas
よく分かってないけど、可算無限個のものをすべて書き下せないということと、ある基数の大きさについて記述できないこと(ZFCから独立)って少し種類が違うように思うんだが
それとも論理体系をゲーデル数化(?)してみると根本では同じ問題とか?
0064132人目の素数さん
垢版 |
2020/07/28(火) 08:22:40.64ID:2jWQxh3X
世の真理

アホは論破できない
アホは自分がアホだという事が自覚出来ない
0066132人目の素数さん
垢版 |
2020/07/28(火) 18:02:23.68ID:dOm/4wzo
すいません。2番と3番を教えてください。
ちなみに2番の問題文のs^2の部分はΣのつけ忘れです。
https://imgur.com/a/DIfWsBb
0068132人目の素数さん
垢版 |
2020/07/28(火) 21:40:44.01ID:qT5SSfn7
課題の中で「直積集合S=Z×(Z/{0})の元(m,n)」ってあったんですけど、ということはn=0ってことですか?バカ丸出しだったらすいません。
0069132人目の素数さん
垢版 |
2020/07/28(火) 22:12:53.11ID:6ZkvpI0i
Z/{0}=Z
0070132人目の素数さん
垢版 |
2020/07/28(火) 23:36:55.33ID:qT5SSfn7
つまりただ単にS=Z×Zとして考えればいいってことですか。。。
0071132人目の素数さん
垢版 |
2020/07/28(火) 23:46:45.68ID:6tqpbtDy
Z/{0}≅ZではあるけどZ/{0}=Zではなくないか
ところで集合A,Bに対して{x|x∈Aかつx∉B}をA\Bと書く
0072132人目の素数さん
垢版 |
2020/07/29(水) 00:01:23.11ID:eZ0B4Nze
俺、「○○と書いても誤解の余地が無い時はそのようにする」って言うやり口大嫌いだわ
0073132人目の素数さん
垢版 |
2020/07/29(水) 00:46:18.70ID:vbta4alb
>>72
面倒くさい奴だなw
0074132人目の素数さん
垢版 |
2020/07/29(水) 01:07:18.14ID:Q8u5RfL7
位相空間や群環体や測度空間とかもいちいち台集合だけでなく組として明示するの?
(コ)ホモロジーの系列とか一つ書くだけですごく時間掛かりそうだね
0079132人目の素数さん
垢版 |
2020/07/29(水) 19:02:16.47ID:zxtk4W6O
偏微分の問題かなー?
y(t) = ∫ u^2 dx として y の微分方程式を出せばいいんじゃないの?
0080132人目の素数さん
垢版 |
2020/07/29(水) 19:40:29.17ID:E8sEy/O6
>>79
その方針で一度試したんですけど微分して条件式代入した時に出てくる
∫uu_{xx} dxの項が上手く評価出来ないんですよね
部分積分使ってもuu_{x}が出てきて邪魔になってしまって
0081132人目の素数さん
垢版 |
2020/07/29(水) 23:08:06.52ID:vbta4alb
>>75
>Z/1=Z
ん?
Z/1={0}やろ
0082132人目の素数さん
垢版 |
2020/07/30(木) 14:45:54.29ID:uE9U+Vhb
>>80
微分方程式を見ると成り立つと思えるのに不思議だね
この直感を数式にできれば解けるんだろうが…
0083ちびでぶハゲニート
垢版 |
2020/07/31(金) 01:20:53.66ID:4maQZwq5
VをdimV=nの線形空間とし,
Vからn個の一次独立なベクトルを取ることができたとする。
それらをv_1‥v_nとするとき
span(v_1…v_n)=V

はいえますかね(´・ω・`)
0084132人目の素数さん
垢版 |
2020/07/31(金) 01:32:28.70ID:bFWbbatH
span(v_1…v_n)=Vでないならv_1,…,v_nの一次結合で表されないVの元v_{n+1}が存在する
同様に元をつけ足していけばn+1個以上の元からなるVの基底が作れるので次元の一意性より矛盾
0086132人目の素数さん
垢版 |
2020/07/31(金) 09:28:20.37ID:bFWbbatH
今見たらそのやり方だとVの基底が作れることが言えねえわ
n個の元からなる基底から一次結合で表せないものとってこないと
0087132人目の素数さん
垢版 |
2020/07/31(金) 09:41:34.66ID:n8pY37yk
そもそも基底の存在定理の証明見てわかるように基底=極大一次独立系ですし次元は基底を構成する元の個数ですし
当然n個の元からなる一次独立系が基底にならなければ極大性に反する
0088132人目の素数さん
垢版 |
2020/07/31(金) 10:01:01.27ID:BFBkj1xN
>>83
当たり前だ
0092132人目の素数さん
垢版 |
2020/07/31(金) 19:30:44.76ID:OrMOzz9y
>>80
∫ uu_{xx} dx = [uu_{x}] - ∫u_{x}u_{x} dx
だけど、問題文の真ん中の等式からuとu_{x}はx→±∞で0になるので、
∫ uu_{xx} dx = - ∫(u_{x})^2 dx

なので、
(∂/∂t) ∫ u^2 dx = ∫-(u_{x})^2 - x^2 u^2 - u^4 dx < 0
で、∫ u^2 dxが単調減少までは言える気がする
t→∞で∫ u^2 dxが0になるかどうかは分からなかった
0093132人目の素数さん
垢版 |
2020/08/01(土) 01:43:20.77ID:BiMGM2/l
位相空間論の問題です
閉集合の部分被覆は、任意の開集合の部分集合になっている? みたいな問題です
どうかお助けください...
https://i.imgur.com/QDTW4QX.jpg
0094132人目の素数さん
垢版 |
2020/08/01(土) 02:33:12.81ID:L6kv8H1m
議論を開被覆に帰着させたらいいだけじゃね?
見た感じ定義から直接的な議論で証明出来そうな気がするが
0096132人目の素数さん
垢版 |
2020/08/01(土) 04:17:23.39ID:BiMGM2/l
https://i.imgur.com/QDTW4QX.jpg
ヒントをくれた方、ありがとうございます!
とりあえず回答を書いてみたのですが、合っているでしょうか?
O_ ⊂ K_ より、K_はO_の開被覆で〜って所から結構曖昧です
0097132人目の素数さん
垢版 |
2020/08/01(土) 08:24:16.82ID:hjm1Quqa
曖昧だというのはOの補集合がコンパクトかどうかかな?
コンパクト空間の閉集合はコンパクトだからokなんだけど、もしこれを知らないのであれば、K_λの補集合たちにOを追加すればXの開被覆が作れるからコンパクト性を使えばいい
0100132人目の素数さん
垢版 |
2020/08/01(土) 11:39:27.70ID:BiMGM2/l
画像貼り間違えてました... ごめんなさい

>>97
まさしく疑問に思っていたのはそこです!
あとはO_とOは共通部分がないから、∪Kから勝手に取っても大丈夫で...って感じですかね
ご回答ありがとうございましたm(_ _)m
0101132人目の素数さん
垢版 |
2020/08/01(土) 12:29:12.19ID:XkWSy5jj
「コンパクト空間の任意の閉部分集合はコンパクト」って基本的な事実だよね
もし知らないならこの機会に勉強したほうがいいと思う
0105132人目の素数さん
垢版 |
2020/08/01(土) 22:05:49.13ID:XFQX9+V0
 特殊解の間違いをご指摘ください

  y'''(x) + 6y''(x) + 12y'(x) + 8y(x) = 5x^2e^(-2x)

  D^3 + 6D^2 + 12D + 8 = (D+2)^3 = 0
  D = -2(3重解)
 よって余関数 Y は
  Y = (C3x^2+C2x+C1)e^(-2x)

  ((D+2)^3)y = 5x^2e^(-2x)

  e^(αx)/(D-α)^n = (x^n/n!)e^(αx)
を使うと特殊解は
  y0 = 5x^2・e^(-2x)/(D+2)^3
    = (5x^2)(x^3/3!)e^(-2x)
    = (5x^3/6)e^(-2x)

 ところが wolframa で計算すると
  y0 = (5x^3/12)e^(-2x)
になります。
0107132人目の素数さん
垢版 |
2020/08/02(日) 10:24:07.49ID:AFwVvtYt
ソートの話

何か巷ではクイックソートが最強みたいな感じでソートの話題になるといつもクイックソート(ドヤッ)ってなるんだが、
度数ソートの方が早いんだろ?
で、データのならび具合によってはクイックソートでもO(n^2)掛かる事あるんだろ?
だったら、マージソートの方が早いんじゃね?って感じるんだが、なぜマージソートや度数ソートが陰に隠れて雑魚扱いされるのか押してくれ
0108132人目の素数さん
垢版 |
2020/08/02(日) 10:29:55.43ID:o3fCNqCA
クイックソートはシンプルなのに早い
それにわざわざ遅くなるようなデータ順になってないことが多い
0109132人目の素数さん
垢版 |
2020/08/02(日) 10:32:08.06ID:o3fCNqCA
Javascriptでクイックソート
Q = A => A.length<=1? A :[
...Q(A.filter((x,i)=> i>0 && x<A[0]))
,A[0],
...Q(A.filter((x,i)=> i>0 && x>=A[0]))];
https://ideone.com/yXp8GZ


Pythonでクイックソート
def Q(A):
if len(A)<2: return A
p = A[0]
return Q(filter(lambda x: x<p, A)) + \
filter(lambda x: x==p, A) + \
Q(filter(lambda x: x>p, A))
https://ideone.com/TSgpzy
0110132人目の素数さん
垢版 |
2020/08/02(日) 13:41:05.77ID:v6UKSQkO
>>107
自分で考えた「区間2分ソート(仮称)」(最悪 n log n)
がググっても見つからないのがオドロキ
0111132人目の素数さん
垢版 |
2020/08/02(日) 15:21:48.63ID:QlRWt596
単位球 B:={x∈Rn:|x|<1} の境界 ∂B={x∈Rn:|x|=1} 上の外向き単位法線ベクトルを ν とする.
また,熱方程式の基本解 E(x,t) に対して w(x,t):=logE(x,t) とする.このとき,任意の x∈∂Ω に対して ∂w(x,t) / ∂ν= ?1/2 となるような t の値を答えよ。(半角数字)

ちょっとこの問題が意味分からんです
ベクトルで微分してるのにベクトルじゃなくて実数値?? ってなって困惑しております

アホな質問だったらごめんなさい
0114132人目の素数さん
垢版 |
2020/08/02(日) 23:46:46.54ID:il+Dk9El
>>111
> ベクトルで微分してるのにベクトルじゃなくて実数値?? ってなって困惑しております
方向微分の定義を確認して下さい。
0115132人目の素数さん
垢版 |
2020/08/03(月) 00:55:05.15ID:XGv27MXU
Hをヒルベルト空間、LをHの閉凸部分集合とする
いまx∈Hに対してd(x,L)=|| x-a ||となるa∈Lが一意に存在している
このとき任意のy∈Lに対して (x-a,a-y)≧0 を示せ

射影定理の話だと思うのですがなかなか示せず困っています
わかる方いたらお願いします
0116132人目の素数さん
垢版 |
2020/08/03(月) 13:16:44.48ID:KxCx7L/K
(x-a, a-y) < 0 となる y があったら b = a + ε(y-a) として || x-b || を展開してみるといい
0117132人目の素数さん
垢版 |
2020/08/03(月) 16:07:38.72ID:xqqHn6XK
すみません、この問題の1_(A_n)ってのは一体どう意味なんでしょうか?
誤植でもないっぽいんですが
https://i.imgur.com/yvcS46T.jpg
0119132人目の素数さん
垢版 |
2020/08/03(月) 16:20:42.33ID:pGHv6+Gw
台関数とは言わないか
指示関数、特性関数、定義関数などの名前があるらしい
0120132人目の素数さん
垢版 |
2020/08/03(月) 17:09:11.88ID:ixa7Rh6l
>>118,119
ありがとうございます!
A_nが座標だと誤解しててずっと迷宮入りしてましたw
0121132人目の素数さん
垢版 |
2020/08/03(月) 20:00:46.08ID:XGv27MXU
>>116
試したことありますが展開しても分からなくないですか?
内積以外の項を上手いこと評価出来ればいいけど私のやり方が間違ってるのか
0123132人目の素数さん
垢版 |
2020/08/03(月) 21:03:09.35ID:pGHv6+Gw
あ、もしかしてRe(x-a,a-y)≧0てことか

そうだとすると
c=(1-t)a+ty∈L(ただし0≦t≦1)として
||x-c||^2=||a-y||^2t^2+2Re(x-a,a-y)t+||x-a||^2
となるけど
これの(0≦t≦1)における最小値は問題の条件により||x-a||^2(これはt=0切片)でなければならない
よって、このtに関する2次関数の中心軸(t=-Re(x-a,a-y)/||a-y||^2)はゼロ以下の位置になければならない
0124132人目の素数さん
垢版 |
2020/08/04(火) 01:38:26.98ID:tJmvcEik
>>121
|| x-b || = || x-a+ε(a-y) || = (|| x-a ||^2 + 2ε(x-a, a-y) + ε^2 || a-y ||^2)^(1/2)
= || x-a || + ε(x-a, a-y)/|| x-a || + O(ε^2)
(x-a, a-y) ≧ 0 でないと || x-a || が最小でなくなる
0125132人目の素数さん
垢版 |
2020/08/04(火) 09:37:32.87ID:snzAYXki
無限の値を取る確率変数の平均が存在しない場合があるけれど
現実にはあり得ない状況のように思える
確率概念はそういう状況を自然に排除できるようになっているべきではなかろうか
0127132人目の素数さん
垢版 |
2020/08/04(火) 17:26:18.32ID:CN6AwSy3
特性類の本を読んでいるのですが次の証明がなぜ言えているのかわかりません

ファイバーの次元が奇数ならオイラー類の位数は2という命題の証明で
奇数次元のベクトル束は向きを逆にする同型(b,v)→(b,-v)をもち
ベクトル束の向きを逆にするとオイラー類は符号が変わるのでe(ξ)=-e(ξ)とあります
この最後の等号はなぜ言えるのでしょうか
0129132人目の素数さん
垢版 |
2020/08/06(木) 10:09:44.15ID:/SelLL0m
代数幾何で見かける、引き戻しf^*、f_*の定義が分かりません
引き戻し(ファイバー積)の定義はwikipediaやレンスターのBasic Category Theoryを見て勉強していますが、*についての記載がありません

例えばスキームの圏だと、Spec(Z)は終対象なので、対象X,Yに対して引き戻し(ファイバー積)X ×_{Spec(Z)} Yが存在しますが、
この状況において射fやf^*、f_*は何を指すことになるんでしょうか?
0130132人目の素数さん
垢版 |
2020/08/06(木) 15:27:31.25ID:nQj2SPhA
>>129
インデューストマップでしょう
考えてるカテゴリーで定義違いましょう
0131132人目の素数さん
垢版 |
2020/08/06(木) 15:36:28.91ID:re8pB6GB
>>130
確かに、引き戻しの文脈で誘導された射を考えることがあるような記憶はあります

ところで調べていて思ったのですが、引き戻し(pullback)の対義語としてpushforwardがあるようですが、引き戻し(ファイバー積)の対義語、というより双対はpushoutですよね

このあたりについて調べてみます
0132132人目の素数さん
垢版 |
2020/08/06(木) 15:39:19.24ID:nQj2SPhA
普通プッシュアウトと言います
0133132人目の素数さん
垢版 |
2020/08/06(木) 15:50:55.40ID:re8pB6GB
>>132
引き戻し(ファイバー積)のwikipediaにはpushoutしか記載されていませんので、そう思っていましたが、pushforwardと共に検索してもそれなりに記事がありました

調べる中で、
https://en.wikipedia.org/wiki/Pullback
に答えと思われることがありました
pullbackはprecompositionとfiber-productという2つの異なる、しかし関連するプロセスのことだそうです
pushforwardもこちらに記載がありました
引き戻し(pullback)には異なる定義があるようですが、
日本語のファイバー積のwikipediaだけを見るとそれが分からないので、中々ややこしいですね

ご回答ありがとうございました、ヒントが得られました
0134132人目の素数さん
垢版 |
2020/08/06(木) 15:55:50.29ID:h+AXPwF3
ワキペディアだよりでなく本をみろ
どこかには買いてあるだろ
教科書ならば
0136132人目の素数さん
垢版 |
2020/08/10(月) 10:26:19.93ID:RKSK+UXb
[前スレ.398] に補足

t/T≠整数 のとき
{t/T} = t/T - [t/T]
 = t/T - floor(t/T)
 = t/T + 1 - ceiling(t/T)
 〜 1/2 + arctan(tan(π(t/T-1/2)))/π,
0138132人目の素数さん
垢版 |
2020/08/10(月) 21:50:03.85ID:XPfHIHPL
集合A, B, C, Dについて、包含関係
(A∪B)△(C∪D) ⊂ (A∪C)△(B∪D)
を示したいのですが、どうすればよいでしょうか。△は対称差です。
x ∈ (左辺)としてx ∈ (右辺)を出そうとしているんですが、なんかゴチャゴチャになってしまって。
0139132人目の素数さん
垢版 |
2020/08/10(月) 22:16:22.65ID:OMgDriQH
>>138
対称差の定義って
A△B := (A-B)∪(B−A)
だっけ
それなら一般には成り立たなくね
例えば A := {1, 2}, B := {2, 3}, C := {4}, D := {5} とすれば、
A∪B = {1, 2, 3}
C∪D = {4, 5}
より (A∪B)△(C∪D) = {1, 2, 3, 4, 5} だが、
A∪C = {1, 2, 4}
B∪D = {2, 3, 5}
より (A∪C)△(B∪D) = {1, 3, 4, 5}
0142138
垢版 |
2020/08/10(月) 23:03:34.91ID:XPfHIHPL
誠に申し訳ありません。とんでもない入力ミスです。正しくは
(A∪B)△(C∪D) ⊂ (A△C)∪(B△D)
でした。
0144132人目の素数さん
垢版 |
2020/08/10(月) 23:38:25.48ID:OMgDriQH
>>142
それならほとんど明らかじゃね
x ∊ (A∪B)-(C∪D) ⇒ x ∊ A-C または x ∊ B-D
x ∊ (C∪D)-(A∪B) ⇒ x ∊ C-A または x ∊ D-B
の成立を確認するだけでしょ
0145132人目の素数さん
垢版 |
2020/08/11(火) 06:41:43.34ID:fIDp8D6U
>>142
ブール代数の演算でやったら簡単だぞ
集合A,B⊆Xに対し、A⊆BとはA∩X\B=φである
これに対応するブール代数の演算はa・b~=0
0146132人目の素数さん
垢版 |
2020/08/11(火) 07:01:49.57ID:fIDp8D6U
>>142
さっき検算したが、成立が確認出来た。
単項式4つ×単項式4つの計算で結構しんどいが、力技でいける

AΔBはab~+a~bという式に対応する、。ってので展開してやるだけ
0147138, 142
垢版 |
2020/08/11(火) 19:37:46.77ID:nxzZDsbw
>>144
ありがとうございます。その方針で解決出来ました。
x∈(左辺)として、それをゴチャゴチャいじっていたら、泥沼にハマってしまっていましたが、別に難しく考えることないですね。

>>145
ブール代数ですか。
ブール代数について、ボクは特に何も知らないので…。
0148132人目の素数さん
垢版 |
2020/08/12(水) 01:30:21.80ID:UgQsH+Ky
今野一宏『リーマン面と代数曲線』に出てくる解析的形成体の説明が分からないのですが教えてもらえますでしょうか
解析的形成体でググってもあまり参考になりそうなものが出てこなくて、行き詰っています。
本に書いてある説明は大まかに以下の通りです。

(a)2つのローラン級数P(t),Q(t)は高々有限個の項からなる主要部を持ち、あるρ>0に対して0 < |t| < ρで収束
(b)任意の異なるt1,t2∈{|t| < ρ}に対して (P(t1),Q(t1))≠(P(t2),Q(t2))のとき、組(P(t),Q(t))を関数要素と呼ぶ
(c)関数要素の同値関係として、(P1(t),Q1(t))〜(P2(t),Q2(t))であることを、
 「t=0の近傍で収束するべき級数τ(t)=c1t+c2t^2+・・・(但しc1≠0)が存在して、P2(τ(t))=P1(t)、Q2(τ(t))=Q1(t)」となることと定め、同値類を[P(t),Q(t)]と書く
(d)関数要素(P(t),Q(t))(|t|<ρ)に対し、|t0|<ρなるt0をとり、t=t0のまわりで展開し、τ=t-t0とおくことで新しい関数要素(P_t0(τ),Q_t0(τ))が得られる。
 これを(P(t),Q(t))の直接接続と呼ぶ。
(e)関数要素全体を、(c)の同値関係で類別して得られる同地類全体の集合をSとする
(f)(P(t),Q(t))の直接接続の同値類がなす集合をp=[P(t),Q(t)]∈Sのρ近傍と定める。これによりSは位相空間となる。
(g)Sの連結成分を解析的形成体と呼ぶ

補題:P(t)が定数でない関数要素(P(t),Q(t))は、以下のいずれかの形の関数要素(z(τ),w(τ))と同値である
(1)正整数mと複素数cがあって、z=c+τ^m、w=Q1(τ)
(2)正整数mがあって、z=τ^{-m}、w=Q1(τ)

解析的形成体は可能な限り解析接続継続していきリーマン面を作るという思想でリーマン面を表現したもの、とのことですが
具体的にどう対応しているのかがわかりません。
補題を見ると、P(t)の方は座標に対応していて、Q(t)の方は解析接続を行う関数に対応しているのかなという気もするのですが
(a)-(g)の解析的形成体の定義ではP(t)とQ(t)が対称に見えるので、PとQのどちらかが座標でどちらかが解析接続を行う関数という訳でもないのかなという気もします。

大まかな方向性でもいいのでアドバイスいただければと思います。
0150132人目の素数さん
垢版 |
2020/08/12(水) 02:30:11.80ID:IKWN0ZI7
解析接続はおおまかにわかってるつもりだけどこんなのは初めてみたな
どれであってもべき級数τが出てきてたのか? じぶんが忘れただけか
0151132人目の素数さん
垢版 |
2020/08/12(水) 13:00:37.31ID:zDKmpSRX
スキームの剰余体について教えてください

スキームの剰余体はスキームの点xに対して構造層の茎O_{X,x}の剰余体として定義されると思いますが、
スキームの点とは、底空間である位相空間の元とZ値点のどちらになりますか?
ザリスキ位相との対応を考えると前者のように見えますが、幾何的点における剰余体を考えると幾何的点は後者なので後者のように見えます
0152132人目の素数さん
垢版 |
2020/08/12(水) 14:38:53.25ID:2FGhPbsd
>>148
リーマン面はどちらが座標とか関数とかはない
分岐がある関数では逆関数でリーマン面を描く必要がある
そういうわけで両方向から見てリーマン面を描く
0153132人目の素数さん
垢版 |
2020/08/13(木) 02:12:42.15ID:/2QvAtuX
>>152
そうなんですね
複素平面を繋げたものがリーマン面というよりは、関数で表される曲面自体がリーマン面というイメージなんでしょうか
0157132人目の素数さん
垢版 |
2020/08/15(土) 20:14:54.26ID:ow828sBE
n∈N、f:Rn→Rが連続で、A={x∈Rn |f(x)≠0}で定義されるRnの部分集合AはRnの開集合であることを示せ。

また、B、CをRnの閉集合とする時B∩CはRnの閉集合であることを示せ。



お願いします。
0158132人目の素数さん
垢版 |
2020/08/15(土) 21:19:36.28ID:3iIf4ygs
お願いしますってなんだよ
(1) 連続写像の定義を思い出す
(2) 閉集合の性質を思い出す
これだけの話だと思うが
0161132人目の素数さん
垢版 |
2020/08/16(日) 01:42:53.10ID:BYdJWaFf
RとR^2の濃度が等しいことって選択公理必要?
RからR^2への全射は作れるんだが
0162132人目の素数さん
垢版 |
2020/08/16(日) 02:03:57.99ID:f0CPlVcj
全単射が作れるじゃん
有理点以外は小数1つ置きで全単射だし
有理点は可算で全単射
0163132人目の素数さん
垢版 |
2020/08/16(日) 02:04:22.29ID:gz5hhscQ
R〜(0,1) 開区間
(0,1)の2つの元を小数展開して交互に組み替えてやれば(0,1)^2から(0,1)への単射が作れる
よってR^2からRへの単射が作れる
RからR^2への単射はふつーに作れる
ベルンシュタインの定理に選択公理は必要ない、したがってR^2とRが対等であることは選択公理なしに証明可能
0165132人目の素数さん
垢版 |
2020/08/16(日) 02:12:46.57ID:ocOa8mpd
>>161
(0,∞)\Q=Sとおいて
S×S≡SはS×Sの成分のそれぞれの十進数表示を互いに組んで一つにすれば良い
S≡RはQ'=Q+√2、T=S\Q'として
S=T+Q'、(0,∞)=T+Q+Q'
を考えれば良い
0166132人目の素数さん
垢版 |
2020/08/16(日) 02:28:10.33ID:BYdJWaFf
理解した
ありがとう
0168132人目の素数さん
垢版 |
2020/08/16(日) 03:11:28.66ID:q2TJL0k3
「単調収束定理」の主張において,正値であっても単調でない確率変数列では結論が成り立たないことを,例を挙げて示せ.

なお、単調収束定理は

確率変数 X_n (n=1,2,...) に対し、
X_n が非負値で、かつ X_n が単調増加ならば
lim_{n→∞} E[ X_n ] = E[ lim_{n→∞} X_n ]

全く思いつきません...どなたかお願いします
0171132人目の素数さん
垢版 |
2020/08/16(日) 15:15:20.21ID:90aPshrv
https://i.imgur.com/xDZqMVY.jpg
初歩的な質問だったらごめんなさい
この堵(z)F_n(dz)というのはどういう計算なのでしょうか?
ルベーグ積分というやつですか?
0173132人目の素数さん
垢版 |
2020/08/16(日) 15:47:24.45ID:DSzhmZTG
>>171
そうだねルベーグ(スティルチェス)だね
でも多分前の方に定義は書かれてるんじゃないの?離散型と連続型を区別せず統一して書いてるだけでg(x)F_n(x)の積分(和)でしょ
0178132人目の素数さん
垢版 |
2020/08/21(金) 23:52:11.01ID:5qiPpY9M
イデアルの指数関数は考えられますか
0180132人目の素数さん
垢版 |
2020/08/22(土) 01:45:19.88ID:q1c4vPQp
n≧2、Aをn次複素正方行列とする
A^(n-1)が対角化不可能、A^nが対角化可能であればA^n=0となることを示して下さい
0181132人目の素数さん
垢版 |
2020/08/22(土) 19:38:34.25ID:zDSoUqSs
位相空間論の同相によって"遺伝"する性質としない性質の違いについての研究って何かありますか?
0182132人目の素数さん
垢版 |
2020/08/22(土) 20:44:28.12ID:RBw7QHAo
>>180
Aによる作用でn次元ベクトル空間をR=k[x]加群とみなしたものをMとおく
S=k[x^(n-1)]、T=k[x^n]とおく
Mの直既約成分にR/(x-a)^i (i≧2, a≠0)が有ればMがT加群として完全可約である事に反するのでそのような直既約成分はない。
もし一つでもR/(x-a) (a≠0)となる成分をもてば全ての成分はR/(x)^i (i≦n-1)の形かR/(x-b) (b≠0)の形となりMはS加群として完全可約となるのり仮定に反する
以上により全ての因子はR/x^iの形であるがi=nであるものがなければやはりMはS加群として完全可約となり仮定に反する
0183132人目の素数さん
垢版 |
2020/08/22(土) 21:39:51.91ID:x5Hhsclx
そんなに難しく考えなくても、
ジョルダン標準形のべき乗と(加法的)ジョルダン分解の一意性を使えばいいんじゃないの?
0184132人目の素数さん
垢版 |
2020/08/22(土) 22:24:58.61ID:+Utih2B4
>>183
まぁそうだけどこのジャンルやってる人間にとっては正方行列の作用でk[x]加群作るのは定石
ちなみにjordan理論も既約k[x]加群が代数的閉体上の既約加群がk(x)/(x-a)^iの形をする事に対応してる
なので上の証明はまさにjordanの理論を加群の理論で書いたに過ぎない
この読みかえは定石中の定石だから、それをわざわざもう一回jordan理論に戻す意味もない
もちろん一回生レベルの話に落とし込めるのはメリットかもしれないけど、まぁ意味ないね
0185132人目の素数さん
垢版 |
2020/08/22(土) 22:37:10.62ID:DVxKX/UW
>ちなみにjordan理論も既約k[x]加群が代数的閉体上の既約加群がk(x)/(x-a)^iの形をする事に対応してる
と書くあたり、jordan理論に戻す意味もない訳ではないことを認識してると思うけどな
0186132人目の素数さん
垢版 |
2020/08/22(土) 22:42:48.05ID:MNXWYBPS
同じことだけど加群の言葉は少し難しいからな
天与の標準形で行列計算した方がわかりやすい学部生も多そう
0187132人目の素数さん
垢版 |
2020/08/22(土) 22:46:16.19ID:MNXWYBPS
>>181
位相空間論は同相で変わらない性質のみを研究する
同相で変わってしまう性質があれば、それはもはや位相空間論ではない
普通、遺伝的といった場合、部分空間に対して同じ性質が成り立つことを言うはず
0189132人目の素数さん
垢版 |
2020/08/22(土) 23:30:05.07ID:MNXWYBPS
>>188
まず性質と構造の区別をしておく必要がある
距離構造や一様構造は位相空間上の構造であって性質ではない
つまりその話は同相で距離が保たれないものがあるという話と本質的に同じ
0194132人目の素数さん
垢版 |
2020/08/23(日) 14:08:53.51ID:/XWty28K
>>190
何だ。こいつ消えたの?
こんな奴の下らん言葉遊びに付き合わせられたってワケか…
0195132人目の素数さん
垢版 |
2020/08/23(日) 14:15:44.74ID:/dDPgAS0
ある数学的概念に対してその定義に使った構造のみで書ける条件はその概念の性質で、他の構造を追加して書いた条件は(付加)構造もしくは新たな概念に対する性質と見なされる
(数学的概念は構造の追加によって下部から上部へ階層化されていることが多く、どの階層から見るかでこの区別は変わる)

線形空間論において内積やノルムは構造、次元は性質
環論において付値は構造、局所性やネーター性は性質
位相空間論において距離は構造、距離化可能性は性質

最後の例のように「ある構造が入るかどうか」自体は性質になることがあり混同されることもある
0196132人目の素数さん
垢版 |
2020/08/23(日) 14:58:54.69ID:/XWty28K
>>195
あ〜、完備性って言ってる時点で、単なる位相空間だけじゃなしに、距離関数の存在までをも含めて言及してるから、それを位相空間ってレベルで捉えるなっていいたいのか
でも結局はそれは完備性って言葉に噛みついただけか。

でもそれだと、俺の最初の質問の答えになってないし、「同相で変わってしまう性質は位相空間論の対象では無い」は答えになってないというか
どこまで言及すれば同相でも変わってしまうのか、どの程度まで言及を抑えておけば同相で維持出来るのかの境目辺りを知りたいって疑問には未だ答えになってない
というかこれは数学基礎論の話になってしまうのか?
0197132人目の素数さん
垢版 |
2020/08/23(日) 15:06:14.38ID:/XWty28K
つまり、>>195の理解に立てば、言及が構造にまで及べば同相で遺伝させれない例(完備性)が出てくるって言いたいわけだ
0199132人目の素数さん
垢版 |
2020/08/23(日) 15:59:22.81ID:QksPy6GM
同相写像は開集合を開集合に、閉集合を閉集合に写し、位相的構造を保つ。つまり、位相空間としての性質(コンパクト性、連結性など)を一切変えない。

https://ja.m.wikipedia.org/wiki/位相同型


これが全てでしょ
完備性が位相空間としての性質として扱われてない理由は、自分で書いてるから分かるだろう
0200132人目の素数さん
垢版 |
2020/08/23(日) 16:10:33.72ID:/dDPgAS0
>>196
位相(あるいはそれより下部の)構造の言葉のみで書かれた条件が同相写像で保存されるのは当たり前で、それは条件の言明を見ればただちにわかる
一方で、付加構造の言葉で書かれた条件が任意の同相写像で保存される(位相不変である)かは自明ではない
しかし、そういう条件があったとしても結局それは位相(あるいはそれより下部の)構造で記述出来てしまうというのが歴史の示唆するところ
というか、位相不変であることを示すとなるとそれが位相の言葉で書けることを示すパターン以外になさそう…
これは常にそうなりそうな感じはするけど、確かに基礎論的に定式化する必要があるかもね
0201132人目の素数さん
垢版 |
2020/08/23(日) 16:26:46.54ID:LqU9/x0L
>>200
位相不変な性質である時点で「付加構造の取り方によらないwell-definedな性質」として位相の言葉で語れてないか
もちろん自明な付加構造を取れる必要はあるけど
0202132人目の素数さん
垢版 |
2020/08/23(日) 17:34:12.93ID:io3wPy1C
距離から導かれる位相が同相なら完備性も同じって証明できなかった?
0204132人目の素数さん
垢版 |
2020/08/23(日) 19:42:27.85ID:/dDPgAS0
>>201
超越的にはそうだけどその条件を位相の言葉で記述しきること無しに位相不変性が示せるんだろうか
仮に超越的に示せたとしてそれが位相の言葉でどうやっても記述出来ないことは可能性としてはある(ないと思いたいが)
それと自明な付加構造が入らない場合も多いから、その場合も考えたい
まあ、この場合は病的な付加構造を与えて位相空間のクラスを厳しく制限することで位相的に記述できないが位相不変になるような条件が作れるかも

>>202
そんなことはない
0205132人目の素数さん
垢版 |
2020/08/23(日) 20:30:00.66ID:LqU9/x0L
>>204
無知で申し訳ないんだけど、「超越的」っていう用語があったりするの?
それと>>201のどこで位相の言葉以外を用いているか教えて
0206132人目の素数さん
垢版 |
2020/08/23(日) 22:09:52.25ID:/dDPgAS0
>>205
超越的は「〜を満たすもの全てを集めてその共通部分をとる」みたいな構成とか存在定理のようなものを経由して示されてることのイメージで使った(用語ではないけどたまに使われてると思う)

なるほど、言いたいことがわかった
「任意に1つ(その付加構造)を入れて、そこで(その条件)を満たす」てのを条件と考えれば付加構造自体は量化子で束縛されてるから文として位相的記述になってると見なすべき、ということか
0207132人目の素数さん
垢版 |
2020/08/23(日) 22:29:05.52ID:/dDPgAS0
量化子で束縛は謎すぎ撤回、自明な付加構造を選べばいいか
その上で記述すれば自明な付加構造は位相的に記述されてるはずだから良いのか

自明な付加構造を持ってない状況だと位相不変な条件が位相的に記述できるかは明らかじゃないよね?
0208132人目の素数さん
垢版 |
2020/08/23(日) 22:38:45.66ID:RARNs6n/
高級な言葉で煙に巻こうと必死になってて、自分でも何言ってるかよくわからなくなってるんじゃないですか?
0209132人目の素数さん
垢版 |
2020/08/23(日) 23:26:35.61ID:/dDPgAS0
まあ数学的な議論というよりはお話だからそう言われても仕方ないが、煙に巻こうなんて1ミリも思ってないよ
0210132人目の素数さん
垢版 |
2020/08/23(日) 23:36:46.87ID:xP2EGYNO
>>206
超越的は初めて知ったわ
さんくす

大方言いたかったことが伝わったみたいでよかった
簡単のために構造と付加構造として集合と位相空間を考えると、位相空間に定義される性質(命題)Pが集合に対して不変っていうのは
∀A∈Set,∀O,O'∈{Aの開集合系},P(A,O)⇔P(A,O')
ってことだと思う
で、このときAは離散位相をもつから
∀A∈Set,∀O∈{Aの開集合系},P(A,O)⇔¬∀O∈{Aの開集合系},¬P(A,O) (※)
ゆえにAが性質Pを持つことは
∀O∈{Aの開集合系},P(A,O)
とか
∃O∈{Aの開集合系},P(A,O)
とか表せると思う
ただし位相と距離だと{Aの距離関数の全体}が空の場合があって(※)の辺りからが上手くいかない
みたいなことを言いたかった
0212132人目の素数さん
垢版 |
2020/08/24(月) 09:37:13.31ID:ulqLR7TI
哲学論争みたいで萎える
0215132人目の素数さん
垢版 |
2020/08/24(月) 11:00:09.95ID:ulqLR7TI
>>213
理論そのものの話じゃ無いからさ
0219132人目の素数さん
垢版 |
2020/08/25(火) 18:46:41.06ID:LqiSh/C2
RとRのZ上のテンソル積は、Rベクトル空間として有限次元ですか?
0221132人目の素数さん
垢版 |
2020/08/26(水) 00:45:02.95ID:fEj1xUS4
>>219
は?dim_Q Rは何だ?
0225132人目の素数さん
垢版 |
2020/08/26(水) 06:02:03.84ID:fEj1xUS4
>>223
R otimes R over Z = R otimes R over Q = R otimes Q^dim_Q R =R^dim_Q R
0226132人目の素数さん
垢版 |
2020/08/26(水) 06:06:06.97ID:fEj1xUS4
>>224
√2 otimes 1 ≠ 1 otimes √2
0228132人目の素数さん
垢版 |
2020/08/26(水) 09:47:49.28ID:8ae+cQFx
>>219
無限次元だろう
0229132人目の素数さん
垢版 |
2020/08/26(水) 10:08:08.06ID:fEj1xUS4
ベクトル空間構造左右で違うから注意な
0230132人目の素数さん
垢版 |
2020/08/26(水) 12:47:47.68ID:GH+n/Xbi
>>227
自分でも気がついて己の馬鹿さに絶望した!
大体、Z上のR⊗RのままじゃRベクトル空間でねーじゃん
自然な定義も一意じゃねーし(>>229)
0231132人目の素数さん
垢版 |
2020/08/26(水) 14:07:12.28ID:fEj1xUS4
>>230
Rベクトル空間だがよ
0233132人目の素数さん
垢版 |
2020/08/26(水) 23:14:12.79ID:qyH3W66h
テンソル積は有限次元だろう。
違うかい?
0235132人目の素数さん
垢版 |
2020/08/27(木) 11:37:23.45ID:3z1md/pl
>>232
右も左もRベクトル空間の旦那様
0237132人目の素数さん
垢版 |
2020/08/28(金) 00:13:16.42ID:y9rDl245
>>236
それ俺が書いたんだがよ
0240132人目の素数さん
垢版 |
2020/08/28(金) 14:40:38.09ID:y9rDl245
>>239
は?>>226も俺だが?
0242132人目の素数さん
垢版 |
2020/08/28(金) 15:41:34.23ID:y9rDl245
>>241
右と左でベクトル空間の構造が異なるから注意してなってことだけど
君こそ何書いているのか理解してないんじゃ無いの?
0244132人目の素数さん
垢版 |
2020/08/29(土) 17:19:58.87ID:wbbY+2Cx
>>243
しょーも無いな
0248132人目の素数さん
垢版 |
2020/08/29(土) 22:23:20.71ID:flLZuKTp
空間の向きじゃなくて作用の話だろ……上から目線で的外れな指摘って恥ずかしいやっちゃな>>246
*をテンソル積としてr(a*b)=(ra)*bと(a*b)r=a*(br)は異なる(整数倍では一致)ってだけでしょ
0249132人目の素数さん
垢版 |
2020/08/29(土) 23:50:47.80ID:wbbY+2Cx
>>245
右に有るRと左に有るRてだけ
別に右左で無く1番2番でもイイが?
0250132人目の素数さん
垢版 |
2020/08/30(日) 00:02:17.38ID:yvpJtH83
>>246
>いや右手系と左手系だって言われたら2ch〜5chで尋ねる前に検索しなさいよ
R otimes R otimes R
ならどう言うつもり?
0251132人目の素数さん
垢版 |
2020/08/30(日) 00:22:32.69ID:yvpJtH83
大体1次元方向に伸びるだけでなくても良いわけで
Rの積が可換だからグラフのノードにR置いて
辺で(Z上の)テンソル積とかでもイイ
0254132人目の素数さん
垢版 |
2020/08/31(月) 06:14:09.44ID:AUCOwLgd
Z/4Z×Z/6Z×Z/9Zの指数3の部分群の個数の求め方を教えてください。
あと、群論を独学しているのですが、演習不足で中々定着しません。おすすめの演習書あれば教えてください。
0255132人目の素数さん
垢版 |
2020/08/31(月) 11:13:22.70ID:yedj+QqV
>>254
G=Z/4Z×Z/6Z×Z/9Zの指数3の部分群はHom(G,Z/3Z)の0でない元と一対一に対応するから、その数は#Hom(G,Z/3Z)-1
ここで
Hom(G,Z/3Z)
= Hom(Z/4Z×Z/6Z×Z/9Z,Z/3Z)
= Hom(Z/4Z,Z/3Z)×Hom(Z/6Z,Z/3Z)×Hom(Z/9Z,Z/3Z)
= {0}×Hom(Z/3Z,Z/3Z)×Hom(Z/3Z,Z/3Z)
となる
一般にnが3の倍数でなければHom(Z/nZ,Z/3Z)={0}
n=3mの時には短完全列
0→Z/mZ→Z/nZ→Z/3Z→0
にHom(〜,Z/3Z)をヒットして得られる左完全列
Hom(Z/3Z,Z/3Z)→Hom(Z/nZ,Z/3Z)→Hom(Z/mZ,Z/3Z)
の最後の→は0写像になる(∵Z/mZの生成元は3+mZ)
故にnが3の倍数のときはHom(Z/3Z,Z/3Z)とHom(Z/nZ,Z/3Z)は同型になる
0256132人目の素数さん
垢版 |
2020/08/31(月) 12:38:54.85ID:OtyXTGIn
再帰的定義って集合論的にどうやってるの?
例えば加法N×N→Nを定義する時にm+nを
m+0=m
m+s(x)=s(m+x)
とやるけど、これで何で({m}×N)×Nの部分集合が定義できるのか分からない
0257132人目の素数さん
垢版 |
2020/08/31(月) 13:01:39.94ID:8lYFQyKD
>>256
公理的集合論の外のと中のがあるけど
中のは置換公理か何か使うんでしょ
0259132人目の素数さん
垢版 |
2020/08/31(月) 14:03:02.54ID:OtyXTGIn
>>257
そんな感じのアレか

>>258
m+の定義に+が使われてるけど俺が分からないのは写像+:N×N→Nの存在をどうやって証明するかというところ
0260132人目の素数さん
垢版 |
2020/08/31(月) 15:20:50.32ID:ZdMd6J8f
増大部分関数列{(m,0,m)}⊂{(m,0,m),(m,1,m+1)}⊂{(m,0,m),(m,1,m+1),(m,2,m+2)}⊂... の和集合とか?
プログラム意味論での再帰関数の定義は、そんな感じだった
0261132人目の素数さん
垢版 |
2020/08/31(月) 15:51:18.99ID:u7vtBaU2
>>259
こう書けばいいか?

mを任意の自然数とする。
数学的帰納法の定理より、plus_m:N→Nが一意に存在して、
plus_m(0)=m
plus_m(S(n))=S(plus_m(n))
が成り立つ。
(mに応じたplus_mの存在が単なる存在ではなく、一意に存在するから、選択公理を使わずして)列(plus_m)_{m∈N}が取れる。

+:N×N→Nを(m,n)→plus_m(n)
で定義すれば良い。
0262132人目の素数さん
垢版 |
2020/08/31(月) 15:57:57.68ID:u7vtBaU2
同一のことだが、こうとでも書けるかな

上述の一意存在により、
PLUS:N→(写像:N→N) を m→plus_m で定義出来る
+:N×N→N を (m,n)→PLUS(m)(n) (=plus_m(n)) で定義すれば良い
0263132人目の素数さん
垢版 |
2020/08/31(月) 17:22:07.36ID:u7vtBaU2
全く同一のことだが、
+ := { ((m,n),k)∈(N×N)×N | plus_m(n)=k }
と定義すれば良い
0264132人目の素数さん
垢版 |
2020/08/31(月) 17:58:30.36ID:PZ/FFL1O
合ってるか分からないけど

a_0=a,a_{n+1}=f(a_n)となる列(a_n)を定義する
これには任意のn∈Nに対して(x^n_m)が一意的に存在して(((x^n_m)が長さn+1の有限列)∧x_0=a∧(m<n→x^n_{m+1}=f(x^n_m)))が成り立てばa_n=x^n_nとして定義できる
以下上記の条件を∀n∈N,P(n)と書く
(x^0_m)はx^n_0=aとしてP(0)
P(n)ならば,m<=nに対してはx^{n+1}_m=x^n_m,m=n+1に対してはx^{n+1}_m=x^n_nとしてP(n+1)
よって数学的帰納法により∀n∈N,P(n)が示せた
0266132人目の素数さん
垢版 |
2020/08/31(月) 18:03:19.50ID:u7vtBaU2
数式のマナーとして、添字が先、冪乗が後
もうこの時点でイラついたわ
0269132人目の素数さん
垢版 |
2020/08/31(月) 18:13:42.85ID:u7vtBaU2
>>267
表記上の話じゃ
で、断りもない冪乗の添字使ってる時点でお前は死ね
お前みたいな相手側の理解に頼った表記する奴は死ね
0270132人目の素数さん
垢版 |
2020/08/31(月) 18:31:52.80ID:PZ/FFL1O
>>260
その証明だと{(m,0,m)},{(m,0,m),(m,1,m+1)}...という列の存在を仮定していて、その列は再帰的に定義されているから循環論法に陥る
>>261
数学的帰納法は自然数に関する1変数述語Pに対してP(0)かつ∀n∈N,P(n)→P(n+1)ならば∀n∈N,P(n)というものであって、最初の質問は帰納的定義をこの数学的帰納法のフォーマットで記述する方法を教えてっていうものだと思ったんだけど
>>268
確かに
0271132人目の素数さん
垢版 |
2020/08/31(月) 18:37:19.40ID:8lYFQyKD
>>267
全然問題ないよ
0272132人目の素数さん
垢版 |
2020/08/31(月) 18:38:54.41ID:8lYFQyKD
>>270
絶対何らかの形で置換公理使うと思うな
0273132人目の素数さん
垢版 |
2020/08/31(月) 18:45:12.12ID:PZ/FFL1O
>>272
ああなるほど
「列」の定義に写像であることが含まれているから関数クラスを定義するだけではだめで、写像として定義するためには確かに置換公理が必要っぽいね
0275132人目の素数さん
垢版 |
2020/08/31(月) 19:07:39.56ID:u7vtBaU2
自分より頭の悪い奴が相手は理解してないんじゃねって思い込んで分かりきったことを一々説明してきた時って、心から死ねって思うな
0277132人目の素数さん
垢版 |
2020/08/31(月) 20:16:03.36ID:AUCOwLgd
>>255
助かりました、ありがとうございます。
0279132人目の素数さん
垢版 |
2020/09/01(火) 01:54:24.32ID:2Y/yWQLJ
>>276
>>261の「数学的帰納法の定理より、plus_m:N→Nが一意に存在して、」が分からない
集合{0, ..., n}か不等号のどちらかを使うと思うんじゃないかと思うけど、そいつらはどうやって定義する?
自然数がs(a)=a∪{a}みたいに具体的に定義されていれば{0, ..., n}はs(n)のことだからいいけど
ペアノの公理から存在はいえるんだろうか
0280132人目の素数さん
垢版 |
2020/09/01(火) 01:55:20.13ID:2Y/yWQLJ
×使うと思うんじゃないかと思う
○使うんじゃないかと思う
0281132人目の素数さん
垢版 |
2020/09/01(火) 05:38:07.08ID:y7oAShns
>>268
よく考えたら(x^n_m)を二重列として扱うのはダメだから二重列の表記は不適切だ
二重列として扱うためには二重列を帰納的に定義する必要が出てきて循環論法になってしまう
飽くまでも「各nに対して有限列が一意的に存在する」という形で述べる必要がある
0282132人目の素数さん
垢版 |
2020/09/01(火) 09:37:58.36ID:avnwFqkU
>>279
>自然数がs(a)=a∪{a}みたいに具体的に定義されていれば{0, ..., n}はs(n)のことだからいいけど
そもそもNを定義するための公理的集合論の論理式はあるのかな?
もしかしたら無いんじゃない?
本質的に公理的集合論の外側で話を進めるべきということでは?
0283132人目の素数さん
垢版 |
2020/09/01(火) 09:43:35.08ID:avnwFqkU
そもそも論理式は可算個しかないので
定義できるのも可算個だけ
個々の自然数は全部定義できるけど
その全部
という言い方ができるかどうかがそもそもの疑問でしょ?
なら自然数全体だって定義できないのかもよ
0286132人目の素数さん
垢版 |
2020/09/01(火) 11:24:46.15ID:y7oAShns
>>285
「各nに対して有限列が一意的に存在する」という形で述べてるから間違っていないはず
0287132人目の素数さん
垢版 |
2020/09/01(火) 11:27:05.18ID:Ax57znWV
>>286
なら「各 n に対して有限列 x[n, m] が一意的に存在する」でええやん
添え字を分けるのと第一成分を固定するのと何が違うの?
0288132人目の素数さん
垢版 |
2020/09/01(火) 12:07:27.13ID:y7oAShns
>>287
ニュアンスの問題といえばそうだけど、x[n, m]はN×N→Vの元のxという対象の(n, m)成分を表す記号であって、N→N→Vのようなものの元を表す記号としてはx^n_mの方が適切だと思う
確認だけど中身は読んだ?
0289132人目の素数さん
垢版 |
2020/09/01(火) 12:15:19.26ID:Ax57znWV
>>288
中身は読んでない
二重数列に違和感があるのなら、 x[n][m] とすればいい
添え字を分ける表記の正当性は示されていない
x[n, m] がそのように定義されるというのなら、 x^n_m はどのように定義され、
それらはどのように違うのか述べよ
0291132人目の素数さん
垢版 |
2020/09/01(火) 12:57:11.98ID:avnwFqkU
>>284
ふーん
勉強できるような資料がネットに転がってませんかね?
Nが作れるんなら何とかなりそうな
0293132人目の素数さん
垢版 |
2020/09/01(火) 13:12:35.65ID:dbB8TRbl
>>279
>「数学的帰納法の定理より、plus_m:N→Nが一意に存在して、」が分からない
松坂和夫「代数系入門」327ページで、X=N、x_0=mとしてそのまんまの証明が載ってる

他にも解答してる人居るけど、俺の解答が一番的確だから。
0294132人目の素数さん
垢版 |
2020/09/01(火) 13:14:36.63ID:y7oAShns
>>289
ここでの(x^n_m)という書き方自体が略記なのでここにおける意味ではなく本来の記号の意味を説明すると、x^n_mはN→N→Vの元のnにおける値のmにおける値を表す記号
N×N→VとN→N→Vは集合として異なるのでその元は一般に一致しない
そもそもZFCで関数クラスを構成するのは好ましくなく関数記号も扱いが面倒なので、本来は略記を用いずより厳密に「任意のn∈Nに対して(x^n_m)が一意的に存在して」の部分を「任意のn∈Nに対してxが一意的に存在して」の形で書いて、「x^{n+1}_m」などを「n+1に対して一意的に存在するxの第m成分」などと書くべきで、記号を改めるなら冗長だけどこっちの方がまだいい
慎重な議論のためにある程度慎重に記号の使い方を選んでいるので批判するなら中身を読んだ上で批判をしてほしい
0295132人目の素数さん
垢版 |
2020/09/01(火) 13:20:42.89ID:Ax57znWV
>>294
>ここでの(x^n_m)という書き方自体が略記なのでここにおける意味ではなく本来の記号の意味を説明すると、x^n_mはN→N→Vの元のnにおける値のmにおける値を表す記号

それをちゃんと書かなきゃダメじゃん
結局みにくいことに変わりはないんだから、「その意味で x[n][m] と書く」と一言書けばいいのでは?
人に伝えたいなら読みやすく
0297132人目の素数さん
垢版 |
2020/09/01(火) 13:25:17.69ID:dbB8TRbl
っていうか、松坂の代数系入門の326ページからはペアノの公理をスタート地点としての厳密な定義(自然数の体系の一意性、和積の定義、整数の定義)がされてる
質問者は恐らくこの辺りやってないだけ。2週間もあればきちんと理解出来るぞ
0298132人目の素数さん
垢版 |
2020/09/01(火) 13:28:26.12ID:dbB8TRbl
厳密な、自然数→整数→有理数→実数→複素数 の構築は素朴集合論でいける
ZFで、集合論の公理→自然数 の構築がされる
0300132人目の素数さん
垢版 |
2020/09/01(火) 16:18:51.87ID:avnwFqkU
>>292
>ウィキペディア見れば充分
どこ?
0301132人目の素数さん
垢版 |
2020/09/01(火) 16:21:28.98ID:avnwFqkU
>>297
>松坂の代数系入門の326ページからはペアノの公理をスタート地点としての厳密な定義(自然数の体系の一意性、和積の定義、整数の定義)がされてる
探してみよっと
0302132人目の素数さん
垢版 |
2020/09/01(火) 16:27:14.38ID:avnwFqkU
>>297
>松坂の代数系入門の326ページからはペアノの公理をスタート地点としての厳密な定義興味有るのはZF内での自然数全体Nの定義
ここに書かれてる厳密性っていうのは
ZF内で自然数を厳密に定義したのを「全部集めて」っていうのではないんですよね?
0303132人目の素数さん
垢版 |
2020/09/01(火) 16:35:55.27ID:dbB8TRbl
>>302
まぁ、人様に質問させて頂くんなら、それなりの言葉遣いあるからな
お前みたいな奴に答える気は起きんわ
んじゃ
0304132人目の素数さん
垢版 |
2020/09/01(火) 17:55:02.93ID:fpY624Bn
f(x + y) = f(x) * f(y)
f(1) = 10
を満たす関数はg(x) = 10^x以外に、無数に存在することを示せ。
0305132人目の素数さん
垢版 |
2020/09/01(火) 17:59:49.07ID:fpY624Bn
>>256
の質問って下の質問と関係ある?


ある本に、関数f : N -> Nを
f(0) = 1
f(1) = 1
f(n) = f(n-1) + f(n-2) (n ≧ 2)
と定義するというのはおかしいと書いてあります。その理由は、これから定義しようという関数f : N -> Nを使ってfの値を定義しようとしているというものです。
そのかわり、関数f : N -> Nで
f(0) = 1
f(1) = 1
f(n) = f(n-1) + f(n-2) (n ≧ 2)
を満たすものが存在することを証明しています。

なんだか分かったような分からないような気がするのですが、上に書いたことは正しいのでしょうか?
0306132人目の素数さん
垢版 |
2020/09/01(火) 19:11:13.24ID:2qjbTlF5
1115
学コン・宿題ボイコット実行委員会@gakkon_boycott 9月1日
#拡散希望
#みんなで学コン・宿題をボイコットしよう
雑誌「大学への数学」の誌上で毎月開催されている学力コンテスト(学コン)と宿題は、添削が雑で採点ミスが多く、訂正をお願いしても応じてもらえない悪質なコンテストです。(私も7月号の宿題でその被害に遭いました。)このようなコンテストに参加するのは時間と努力の無駄であり、参加する価値はありません。そこで私は、これ以上の被害者を出さないようにするため、また、出版社に反省と改善を促すために、学コン・宿題のボイコットを呼び掛けることにしました。少しでも多くの方がこの活動にご賛同頂き、このツイートを拡散して頂ければ幸いです。
https://twitter.com/gakkon_boycott/status/1300459618326388737
https://twitter.com/5chan_nel (5ch newer account)
0307132人目の素数さん
垢版 |
2020/09/01(火) 19:53:14.54ID:dbB8TRbl
>>305
体系外で把握出来ることを体系内に式として表して把握したい、っていう要望か
こういうのは厄介な式変形だったり集合操作で把握するのにちょっと時間掛かるが出来るな
0308132人目の素数さん
垢版 |
2020/09/01(火) 19:55:07.38ID:dbB8TRbl
フィボナッチ数列では前の2項を使って新たに値を定義するけど、
ZFC集合論だと、もっと一般に、それまでの値全部を使って現在の新たな値を定義出来るっていう定理もある
0310132人目の素数さん
垢版 |
2020/09/01(火) 20:25:36.39ID:avnwFqkU
>>303
あーらら
まあ読めば分かるから探してみよっと
>>309
サンクス
0311132人目の素数さん
垢版 |
2020/09/01(火) 21:00:11.44ID:avnwFqkU
>>309
ありがとう
これでNが定義できてますね
たぶん>>297で紹介してくれている
松坂の代数系入門のp326〜もここに書かれているようなものなのだろうなあ
そちらにもありがとう

大体を言うと
後継者a+=a∪{a}
空集合φを含み含んでいる元の後継者を含む無限集合が存在するので(無限公理)
その中で一番小さいものを決めるために分出公理(置換公理の単純版)を使う
そうして得たものが1つしか無いことを証明する
ということなのね
0312132人目の素数さん
垢版 |
2020/09/02(水) 11:50:56.23ID:8HGOiuSr
>>305
おかしいというものではないよ
ただ
公理的に考えなくてはならないという前提での質問なら
後者の方がスッキリする
前者も特に問題はないはずだが
公理的に示すのは面倒そう
0313132人目の素数さん
垢版 |
2020/09/02(水) 12:21:53.18ID:HMn9l33T
x1,,,xmをパラメーターとして固定し、x~と略記する。
g1,...,gm,hを関数とする。

   f(1,x~)=g1(x~)
   ,...,
   f(m,x~)=gm(x~)
   と定義し、nに対し
   f(n+m+1,x~)=h(f(n+1,x~),...,f(n+m,x~),x~)
   と定義する

↑こういう帰納的定義って、直観的には定義出来てることは分かるけど、
公理的にこういう関数が存在することを証明するとなると絶対しんどいよな
0316132人目の素数さん
垢版 |
2020/09/02(水) 17:15:48.57ID:HMn9l33T
>>314
あれ?パターンだから>>313も同じで良いんじゃねぇの?
お前自分で言っておきながら答えれへんのか?
頭の状態は大丈夫か?
0317132人目の素数さん
垢版 |
2020/09/02(水) 19:31:49.46ID:i2k4QLcN
>>297
p.327の帰納的定義の原理(定理1)って難しいですね。

p.328のB'' := B - {(σ(k), z)}が条件(a), (b)を満たすことの証明は以下であっていますか?

(a) (0, x_0) ∈ Bである。また、p.326の公理N2により、0≠σ(k)であるから、(0, x_0) ≠ (σ(k), z)。∴(0, x_0) ∈ B''である。

(b) (n, x) ∈ B''であるにもかかわらず、(σ(n), φ(x)) ∈ B''でないと仮定する。(n, x) ∈ Bだから(σ(n), φ(x)) ∈ Bである。
∴(σ(n), φ(x)) = (σ(k), z)でなければならない。p.326の公理N1により、σは単射であるから、n = kでなければならない。
∴(n, x) = (k, u)でなければならない。すると、(σ(n), φ(x)) = (σ(k), φ(u)) = (σ(k), z)となってしまうが、これはφ(u) ≠ zである
という仮定に反する。
0321132人目の素数さん
垢版 |
2020/09/03(木) 00:39:30.35ID:lgxc8+Pl
>>317
その本の証明をちょっと確認したけど、Aに"余分な列"が入っていたら取り除いたとしても依然(x0,φ)集合のままであるが、
Bの最小性を考えると、"余分な列"は存在し得ない っていうロジックだな
0322132人目の素数さん
垢版 |
2020/09/03(木) 02:43:12.98ID:QHxsSkOO
>>317
はあっていますか?
0323132人目の素数さん
垢版 |
2020/09/03(木) 14:14:02.88ID:QHxsSkOO
松坂和夫著『代数系入門』の付録を読み終わりました。謎が解明されたという感じは全くせず、ただ責任を公理に添加しただけという感じが拭えません。
ただ、帰納的定義の原理は、これがあるとスッキリしますね。
0326132人目の素数さん
垢版 |
2020/09/03(木) 14:41:45.54ID:7p7EW6Y9
>>323
てか確かペアノの公理だけでは加法の構成はできなかったんじゃなかったけかな
で、加法の存在だけ仮定したプレスバーガー算術とか加法と乗法の両方の存在仮定したロビンソン算術とかあったと思う
その辺の細かい話は基礎論専攻するつもりでない限りはこだわりすぎない方がよさげ
0327132人目の素数さん
垢版 |
2020/09/03(木) 15:09:54.48ID:lgxc8+Pl
なるほどなるほど

そのモヤモヤした感情の原因は、松坂の代数系入門の付録に載ってる公理は、素朴集合論を前提としたペアノの公理だから、かもな
ZFCの公理を見せず、集合操作を直観的に行ってる当たりに、公理的議論をしてる感じが伝わってこないってことかもしれないんじゃね?
0328132人目の素数さん
垢版 |
2020/09/03(木) 15:46:14.46ID:ah8uPmWj
>>327
松坂のそれ見たいなあ
どこかに落ちていないかしら
0330132人目の素数さん
垢版 |
2020/09/03(木) 17:40:18.96ID:QHxsSkOO
島内剛一の数学の基礎はおすすめでしょうか?
0331132人目の素数さん
垢版 |
2020/09/03(木) 17:44:37.93ID:ah8uPmWj
>>329
知らない
0333132人目の素数さん
垢版 |
2020/09/03(木) 17:52:52.29ID:cFWcg2IL
>>330
>島内剛一 数学の基礎
おすすめ
心して完読せよ
0335132人目の素数さん
垢版 |
2020/09/03(木) 19:14:39.99ID:ah8uPmWj
>>332
知ってる?って質問に知らないと答えてこれはなぁ
0336132人目の素数さん
垢版 |
2020/09/03(木) 20:16:03.46ID:mi8pLAv4
>>332
盗人猛々しい
0338132人目の素数さん
垢版 |
2020/09/04(金) 05:04:35.50ID:iut7BU7o
(Z/8Z×Z/8Z×Z/3Z)/(Z/4Z×Z/4Z×Z/3Z)はZ/2Z×Z/2Z×{1}すなわち(Z/2Z)^2に同型である、という議論は可能ですか?
具体的には次の2つが可能であるかどうか知りたいです。
・(Z/6Z)/(Z/2Z)がZ/3Zに同型である、といったような通常の分数の割り算を思わせる計算。
・直積を分けて考えること。上の例では(Z/8Z)/(Z/4Z)がZ/2Zと同型、(Z/8Z)/(Z/4Z)がZ/2Zと同型、(Z/3Z)/(Z/3Z)が{1}と同型、よってこれらの直積を取って(Z/2Z)^2に同型であるという議論をした。
出来る場合と出来ない場合がある時はどのような時に出来てどのような時に出来ないのか教えてください。よろしくお願いします。
0339132人目の素数さん
垢版 |
2020/09/04(金) 10:20:16.51ID:CIoeKwyr
>>304
RをQベクトル空間と見て1の生成する所以外は適当なべき乗でに
0342132人目の素数さん
垢版 |
2020/09/05(土) 20:05:30.04ID:lbP0o9nI
>>340
極小な生成系というのがよくわからないけど
例えばQ上の代数Aを非負の実数でパラメトライズされた基底Xaてはられるベクトル空間にに積をXaXb=Xa (a≦b)で定めた代数としてMをその極大イデアルとする
この時集合SがMを生成する
⇔∀a>0∃b>a Xb∈S
だから“極小な生成系?”はない
0343132人目の素数さん
垢版 |
2020/09/05(土) 20:08:12.92ID:BtVdvkls
定理4.4.2の証明中の「つまり、どの辺も1度だけ使われる。」の言っていることが分かりません。
解説をお願いします。

定理4.4.1
木のどの2点もちょうど1本の道で連結している。

定理4.4.2
どんな位数nの木もn-1本の辺をもつ。
証明:
電話のネットワークを例にとって証明しよう。ある町で事件が起こり、他の町にメッセージを電話で送ろうとしたとする。
まず、その町の人は直接回線がつながっている町で電話する。電話を受けた町は直接つながっている町へ電話する。
電話を受けた町は、直接つながっている町でまだ電話を受けていない町へ電話する。…、グラフは連結なのでメッセージは
どの町へも伝わる。定理4.4.1より、どの町も事件のあった町とは1通りの道で結ばれている。つまり、どの辺も1度だけ
使われる。したがって、電話の回数は辺の本数と1対1に対応する。電話をかけないときに事件を知っている町はその町1つ
だけで、1回電話するたびに事件を知る町が1つずつ増える。したがって、点(町)の個数は辺(交信)の本数よりもちょう
ど1つ多い。
0344132人目の素数さん
垢版 |
2020/09/06(日) 00:03:15.19ID:+4r5cIxH
>>342
SがMの極小な生成系である⇔SがMを生成し、任意のSの真部分集合はMを生成しない
のつもりです
Aは単位的でない環のようなので極大イデアルをもたないと思うのですが
0345132人目の素数さん
垢版 |
2020/09/06(日) 06:51:20.10ID:rSTl7z29
>>344
不等号逆だ
XaXb = Xb (a<b)
X0が単位元
m=<xa | a>0>
が唯一の極大イデアル
SがMのgenerator ⇔ inf{ a | Xa∈S } = 0
0348132人目の素数さん
垢版 |
2020/09/07(月) 13:28:54.38ID:bE/6WhUJ
君ら、逆行列が存在する条件ってちゃんといえる?
0349132人目の素数さん
垢版 |
2020/09/07(月) 15:00:07.23ID:acw7LRjx
>>348
行列式で?階数で?
0350132人目の素数さん
垢版 |
2020/09/07(月) 16:05:50.81ID:hZJBZOMm
諸々の条件とかをエスパーする問題じゃないの?
0355132人目の素数さん
垢版 |
2020/09/09(水) 12:04:57.95ID:1XMffyUZ
uを全順序集合とする。v⊆uとする。
vはuの共終部分集合の定義は↓のどっちですか?
・∀x∈u∃y∈v ( x≦y )
・∀x∈u∃y∈v ( x<y )
0358132人目の素数さん
垢版 |
2020/09/09(水) 13:21:45.96ID:1XMffyUZ
>>357
そもそも共終数ってのは実際上は極限順序数しか考えないもの。
でも定義上は一般の順序数で考えてる
0361132人目の素数さん
垢版 |
2020/09/09(水) 14:51:21.80ID:YDbS9Hgz
>>355
どっちでんよか
0362132人目の素数さん
垢版 |
2020/09/10(木) 02:24:51.54ID:RhM7arcq
共終の概念は ≦ の方だな
証明は極限順序数だけで成り立つと明記すればいいだけ
0365132人目の素数さん
垢版 |
2020/09/10(木) 15:49:19.19ID:vX+5z49C
ユークリッド幾何学の1階の理論の決定可能性の証明を読みたいのですが、書籍や文献を教えて頂きたいです。出来れば日本語でお願いします。
0367132人目の素数さん
垢版 |
2020/09/15(火) 13:45:14.40ID:77sRtwsv
>>366
順序数αが極限数なら確かに分かるけど、後続順序数S(γ)の時は(1)⇒(2)は言えない気がするんだが
(S(γ)⊆∪Rng(f)が言えない…¬γ∈Rng(f)な気がする)
0368132人目の素数さん
垢版 |
2020/09/15(火) 20:53:54.51ID:6NaVD6qo
だから極限数しか考えてないと推察できるのさ
自力で正しい結論を出せるんだから教科書が全部正しいなんて思うなよ
0369132人目の素数さん
垢版 |
2020/09/15(火) 21:30:38.19ID:77sRtwsv
定義から他の定理を援用せず直接帰結出来るような命題については大体は自分で行間埋めれるけど、
証明の行間を埋めてる時もそうだけど、「やっぱりテキストの言う通りに理解したらおかしい気がするんだよなぁ」とは思いつつ、
「やっぱり俺の力では気づけていない点がどこかにあるかも」という気も捨てきれないから、
自分の中で区切りを付けて先に進めない時が多々がある。
0370132人目の素数さん
垢版 |
2020/09/15(火) 23:19:14.78ID:77sRtwsv
○○は位相空間でもベクトル空間でも軍でも何でもいい
 ∀基数κ∃○○空間X |X|=κ
って一般に真?
0371132人目の素数さん
垢版 |
2020/09/15(火) 23:36:14.24ID:pyAP0ceG
群(G,・,e)が
∀a,ae=ea∧∀a,∃b,ab=ba=e∧∀a,b,c,(ab)c=a(bc)
で定義される代数系であるのと同様に
∀a,b,a=b
で定義される代数系も考えられる
これの台集合は一元集合に限られる
0372132人目の素数さん
垢版 |
2020/09/15(火) 23:48:32.42ID:fRXUQm28
>>370
少なくとも位相空間と群は真だな
ベクトル空間だとcardinal cに対して
∃V: vector sp. |V|=c iff c;infinite or 素数冪
0374132人目の素数さん
垢版 |
2020/09/16(水) 00:12:01.83ID:a2TvNY+b
偽であるような代数系としては体があるな
そもそも有限濃度の場合ですら素数冪じゃないとだめだし
0375132人目の素数さん
垢版 |
2020/09/17(木) 00:31:38.86ID:UDD0bsv7
f_1(a,0) := a
f_1(a,b+1) := f_1(a,b)+1,

f_2(a,0) := 0
f_2(a,b+1) := f_1(f_2(a,b),a),

f_3(a,0) := 1
f_3(a,b+1) := f_2(f_3(a,b),a)
によって、f_i(a,b) (i=1,2,3)を定義する。

すると、一般に
f_{n+1}(a,b+1) := f_n(f_{n+1}(a,b),a)
と定義したくなる。
f_{n+1}(a,0)は何と定義すべきか?
0383132人目の素数さん
垢版 |
2020/09/19(土) 04:37:07.41ID:OU7LbAAl
0倍は当然0にしかならんのだから、F_2係数のベクトル空間span(X)はXの(形式的)有限和全体になってその濃度はXの有限部分集合の全体の濃度に等しいのでは?
可算集合Xの有限部分集合全体の濃度は2^Xではなく可算濃度でしょ
0384132人目の素数さん
垢版 |
2020/09/19(土) 16:33:37.32ID:Ty0Q2bdh
位相空間における基底って何で濃度についての最小性が定義に含まれて無いんかな
0385132人目の素数さん
垢版 |
2020/09/19(土) 18:17:29.98ID:OU7LbAAl
フィルター、開集合族、イデアルetc.

単なる生成系の意味で基底と呼んでるものの方が多くね?何らかの最小性を課したい気持ちはわかるけど、なんかもう気にしたら負けかなって
0386132人目の素数さん
垢版 |
2020/09/21(月) 00:26:01.59ID:5NRJFJOk
田中尚夫「公理的集合論」の具体的な内容について聞きたいんだけど、読んでる人いないよな?
0387132人目の素数さん
垢版 |
2020/09/21(月) 15:20:45.57ID:WPkexpEh
>>386
積ん読した
0388132人目の素数さん
垢版 |
2020/09/21(月) 18:38:48.95ID:ot+0b+tr
そういえば、なんで二重極限を定義する前に当たり前のようにコーシー列を扱ってるの?
0390132人目の素数さん
垢版 |
2020/09/21(月) 20:32:49.86ID:Lnon6Ca0
メギョウ
0391132人目の素数さん
垢版 |
2020/09/23(水) 20:10:03.77ID:AbsaUdOP
よろしくお願いします。
http://or2.mobi/index.php?mode=image&;file=287992.jpg
上の式がキューネンの数学基礎論講義に出てくる置換公理なのですが、少し弱いと思いました。
下の論理式のように包含関係を弱めたものと同値と思って良いですか?
頑張ったんですが論理的同値が、示せなくて良くわからなくなってしまいました。
0392132人目の素数さん
垢版 |
2020/09/23(水) 21:53:44.89ID:nbEKtL7T
>>391
上の式合ってる?
BとしてVersus classとってくればいつでも成立してしまう気がする
0394132人目の素数さん
垢版 |
2020/09/23(水) 23:22:28.76ID:NrPGw2Fi
>>392
ZFCでは
>>393
分出公理(内包公理図式)を認めると
∃B'∀y(y∈B'⇔y∈B∧∃x∈A,φ(x, y))
で→が言える
たぶんだけどその本は置換公理→分出公理を言わない立場で分出公理は使ってもいいと思う
0395132人目の素数さん
垢版 |
2020/09/23(水) 23:56:44.83ID:94rHPxrN
その本の置換公理って俺が今まで見てきた置換公理
∀x,y,z(P(x,y)∧P(x,z)⇒y=z)⇒∀a∃z∀y(y∈z⇔∃x∈a P(x,y))
と違うよな
0397132人目の素数さん
垢版 |
2020/09/24(木) 00:02:21.17ID:Muy9+wk0
良く見たら、その本、巾集合公理でも逆向きの矢印が書かれてないな
じゃあ、逆向きは部分集合公理を使えば良いって言うことなんだろうな
0398132人目の素数さん
垢版 |
2020/09/24(木) 00:07:29.14ID:Muy9+wk0
基礎論でどっかのテキストに書いてた文言コピペ↓
G.Boolos, [The Logic of Provability] cambride university press (1993)
 第二不完全性定理の証明が詳細

J.Barwise, [Handbook of Mathematical Logic], North-Holland 1977
入門者から専門家まで必携の読める辞典

R.Shoenfield [Mathematical Logic] Addison-Wesley 1967
基礎論の専門化を目指す人が一度は手にする最高級の教科書
0401132人目の素数さん
垢版 |
2020/09/24(木) 00:41:01.99ID:tZusWsqn
ラッセルのパラドックスを回避する流儀の違い
BG流は任意の命題φに対して
{x | φ(x)}
の形の“クラス”の存在を許す代わりにこのクラスの満たす公理を
a ∈ {x | φ(x)} ⇔ φ(a) ∧ (∃b a∈b)
にする事で回避する
ZF流は上のような“クラス”の存在は認めず、命題φと集合Aに対して
{x∈A | φ(x)} = {x | x∈A ∧ φ(x)}
の形しか許さない事で回避する
0403132人目の素数さん
垢版 |
2020/09/24(木) 00:56:19.71ID:MSVVn7EX
>>401
いやNBGのことであろうことは想像ついてたけどそれをBG流って呼ぶ人間はほぼいない(もしかしたらネット上ではお前だけ?)ってこと
0409132人目の素数さん
垢版 |
2020/09/24(木) 08:22:08.38ID:xIMVNCxk
>>391
∃と∀を逆にしてるのなぜ?
0410132人目の素数さん
垢版 |
2020/09/24(木) 08:22:29.86ID:xIMVNCxk
>>400
BGで
0411132人目の素数さん
垢版 |
2020/09/24(木) 08:26:35.20ID:xIMVNCxk
>>392
Bにクラスはダメしょ
0412132人目の素数さん
垢版 |
2020/09/24(木) 11:45:30.82ID:juHen0h9
院を探しているけど、今一番論文を書きやすい分野ってどこ?
0415132人目の素数さん
垢版 |
2020/09/24(木) 15:15:35.36ID:WRaiy9OC
>>409
2行目の式の後件ですか?
2行目の式は一般的な置換公理から類推して勝手に自分でたてました。
これがキューネンの式と同値になるかなと思ったんですけど、示せなくて困りました
0416132人目の素数さん
垢版 |
2020/09/24(木) 15:17:37.87ID:8r+XUoYF
ノイマンが嫌いなんだろ
知らんけど
0417132人目の素数さん
垢版 |
2020/09/24(木) 20:04:38.32ID:Muy9+wk0
数学基礎論シリーズ 02巻 公理的集合論 倉田令二朗,篠田寿一
にBG集合論の取り扱いがある

集合論 独立性証明への案内 Kenneth Kunen,訳:藤田博司
はZFCで議論してる
0419132人目の素数さん
垢版 |
2020/09/25(金) 10:59:06.04ID:SkScNr+e
>>401
部分クラスの全体というモノは有り得ないということは
べき集合についてはa∈bであるaについてのみってこと?
0420132人目の素数さん
垢版 |
2020/09/25(金) 14:25:23.52ID:RhlYSyQ3
数学基礎論シリーズ 02巻 公理的集合論 倉田令二朗,篠田寿一 によると、
無定義述語は ∈、=、とM( )。
M(x)は”xは集合である”を意味する。
というのも、BG集合論では変数記号はクラス全体を走るから。
で、∀X,Y(X∈Y⇒M(X))っていう公理がある。
これによって、"クラスがクラスに属することは無い"ことになる。
0421132人目の素数さん
垢版 |
2020/09/25(金) 14:27:58.13ID:RhlYSyQ3
数学基礎論シリーズ 02巻 公理的集合論 倉田令二朗,篠田寿一 のP13〜
を読め
0428132人目の素数さん
垢版 |
2020/09/26(土) 07:52:57.96ID:py3TZ3ga
>>420
>∀X,Y(X∈Y⇒M(X))っていう公理
∀X(∃Y(X∈Y)⇔M(X))じゃないのか
M()なくして∃Y(X∈Y)を使えば良いんじゃないのか
0430132人目の素数さん
垢版 |
2020/09/26(土) 18:18:08.18ID:py3TZ3ga
そんなYは無いワイ
0431132人目の素数さん
垢版 |
2020/09/26(土) 18:20:59.87ID:py3TZ3ga
いやあるワイ・・・
0432132人目の素数さん
垢版 |
2020/09/26(土) 20:29:06.90ID:py3TZ3ga
>>429
公理として
∃U∀A∀x(x∈A⇒x∈U)
を入れたらどうかな
このUは1つしか無いから
M(X)≡X∈U
あるいは
U={x|M(x)}
ということになるけど無定義述語Mよりよいような気がする
0433132人目の素数さん
垢版 |
2020/09/26(土) 20:31:48.95ID:py3TZ3ga
Uを導入すると補集合(補クラス)のある集合論ってことになるかなあ
集合の補クラスはプロパークラスだからあんまり良くない?
0438132人目の素数さん
垢版 |
2020/09/28(月) 16:27:05.71ID:PcTcLaOM
>>437
なるほど
補クラスの存在を公理にしたらUはφの補クラスになるということですね
頭良いなあ
0439132人目の素数さん
垢版 |
2020/09/29(火) 00:34:39.80ID:qaL+uvMf
(Z/pZ)^×からZ/2Zへの準同型って自明なものと平方剰余だけなんでしょうか?
0440132人目の素数さん
垢版 |
2020/09/29(火) 00:44:47.66ID:Q1LeNDVA
>>439
pが素数の時G=Z/pX^×は位数p-1の巡回群
よってZ(の加法群)からGへの全射準同型π:X→Gが存在する
Hom(G,Z/2X)→Hom(Z/2X)をf→f・πで定められるものとするとコレは単射
一方HomZ,Z/2X)はZ/2Zに同型なのでその元数は2個
0441132人目の素数さん
垢版 |
2020/09/29(火) 00:57:10.12ID:qaL+uvMf
>>440
なるほど!乗法群が巡回群であることとZからの射を使うと自然に示せるわけですね
この証明から平方剰余の自然な役割もよく分かりますし
超速かつ素晴らしい回答感謝です!
0442132人目の素数さん
垢版 |
2020/09/29(火) 01:08:36.26ID:qaL+uvMf
巡回群からZ/2Zへの準同型はZ/2Zと同型ということですが
対称群からZ/2Zへの準同型もZ/2Zと同型なんでしょうか?
つまり自明なものと置換符号だけかどうか
0443132人目の素数さん
垢版 |
2020/09/29(火) 01:34:09.88ID:qaL+uvMf
自己解決(?)
全射の場合、核は位数半分の正規部分群にならないといけないから核は交代群で、これは置換符号になる
でしょうかね…
0444132人目の素数さん
垢版 |
2020/09/29(火) 14:27:56.38ID:9flVjTac
コーシー・リーマンの関係式を満たすu(x, y), v(x, y)がC^∞級の関数であることをコーシーの積分定理を使わないで証明できるのでしょうか?
0445132人目の素数さん
垢版 |
2020/10/01(木) 03:03:30.93ID:7qZlKSjO
テイラーの定理を既知としてC^r級関数fに対してC^{r-2}級関数gが存在して
f(x)=f(0)+f'(0)x+g(x)x^2
が成り立つことの証明を教えてください
0446132人目の素数さん
垢版 |
2020/10/02(金) 06:47:43.79ID:vpjcCNTW
>>445
この質問ですがf,gがC^∞級関数のときだけでも教えていただけるとありがたいです
0447132人目の素数さん
垢版 |
2020/10/02(金) 21:18:20.15ID:+L+ZMxY8
可逆であると知られている対関数j:N×N→Nは
j(a,b)=(a+b)(a+b)/2+aという形だけど、この逆関数を具体的に表すとどうなりますか?
0450132人目の素数さん
垢版 |
2020/10/02(金) 22:38:35.63ID:cGFN1mwi
Nが自然数の集合だとすると写像になってないように見えるんだが(a+b)(a+b+1)/2+aのミスか
0452132人目の素数さん
垢版 |
2020/10/04(日) 23:30:14.93ID:wATKi81a
実射影空間P^2がR^3に埋め込めないことの証明ってどうやるのでしょうか?
多様体の基礎にアバウトな証明は載っていて,厳密にはホモロジーを使うと書いてあるのですが
ホモロジーは少し勉強したもののどう使うのかわかりません
この本に載ってるとかこういう知識を勉強すると分かるとかでも教えていただけるとありがたいです
0453132人目の素数さん
垢版 |
2020/10/05(月) 00:10:16.18ID:QjgWwv4N
>>452
ホモロジー完全列の切除定理じゃなかったかな
0454132人目の素数さん
垢版 |
2020/10/05(月) 11:35:54.11ID:3ASPpNbU
>>453
なるほど〜
ディスクを切除するとメビウスの帯だから〜と計算して無事矛盾が言えました
ありがとうございます
0455132人目の素数さん
垢版 |
2020/10/05(月) 17:55:18.17ID:3ASPpNbU
>>454
行けたと思ったのですが,よく考えてみたら切除定理の仮定が言えていませんでした
(R^3,P^2)のホモロジー完全列で対のH_i(R^3,P^2)を切除定理から計算しようにも
P^2のR^3における内部は空なので切除定理の仮定が言えません
P^2の管状近傍を取って考えてもうまくいかずどうしたらいいものか
詳しい証明をご存知の方いたら教えてください
0457132人目の素数さん
垢版 |
2020/10/06(火) 20:59:15.84ID:PY0SWOPs
>>456
すみません説明が足りていなかったみたいで
可微分多様体(C^r級多様体で1≦r≦∞)で埋め込みもC^r級の埋め込みを考えています
0458132人目の素数さん
垢版 |
2020/10/06(火) 22:09:09.85ID:XT+pZq2Z
ごく自然に、
輩<0ψ(x1,?,xn,y)=0,
輩<z+1ψ(x1,?,xn,y)=ψ(x1,?,xn,z)+輩<zψ(x1,?,xn,y)

Πy<0ψ(x1,?,xn,y)=1,
Πy<z+1ψ(x1,?,xn,y)=ψ(x1,?,xn,z)×Πy<zψ(x1,?,xn,y)
と定義する。

Pを述語、χをその表現関数(特性関数)とし、y<z∧P(x1,?,xn,y) を満たすyが存在する時はそのようなyの最小値を、そのようなyが存在しない時はzを値にとる"関数は、
μy<zP(x1,?,xn,y) := 背<z(Πy<wχ(x1,?,xn,y))
とのことですが、もしP(x1,…,xn,0)が成り立つ時、1≦zとすれば、
μy<zP(x1,?,xn,y)=1にしかならないとしか思えないんだが?
正しい式にするにはどういう風に式を修正すれば良いですか?
(注意:表現関数は、Pが成り立つ時に0,成り立たない時に1をとる)
0462132人目の素数さん
垢版 |
2020/10/07(水) 00:06:46.27ID:gJYHWynk
>>457
それならMayer Vitorisでいける
P=P^1がS=S^3に埋め込まれてるとしてx∈Pと閉球の近傍B=B_e(x)を十分小さくとってB∩P=Dが2次元円盤、(S\intB)∩P=Mがメビウスバンド、M∩F=CはS^1になるようにとれる
この時U=S\DとV=S\MについてU∩V=S\P、U∪V=S\Cになるから引き起こされるhomologyのMayer Vietoris列
→ H1(S\D)+H1(S\M)→H1(S\C)→H0(S\P)→ H0(S\D)+H0(S\M)
をみるとH0(S\P)→ H0(S\D)+H0(S\M)は単射だから H1(S\D)+H1(S\M)→H1(S\C)は全射にならなければならない
しかしH1(S\D)は自明、H1(S\M)=xZ、H1(S\C)=yZは共にランク1の自由群だけどH1(S\M)→H1(S\C)はxを2yにマップするから全射ではない
この証明ならPLでも可微分でもいける
general topologyだと絶望的に難してオレには無理だ
多分肯定的に解けてると思うけど
0463132人目の素数さん
垢版 |
2020/10/07(水) 01:16:19.25ID:rbeqNixx
>>462
おお…今度こそ理解できました
R^4になら埋め込めるというのはこの証明ではSの次元を上げるとH1(S\M)やH1(S\C)が消えて
同じ議論ができなくなるということに対応するわけですね
勉強になりました,ありがとうございます
0466132人目の素数さん
垢版 |
2020/10/07(水) 16:57:43.42ID:CbnrMrq7
疑問がおかしいやん
随伴関手って関手F:C→Dと関手G:D→Cについて定義される概念で
C→C^opが関手としても“圏の圏”は関手ではない
0468132人目の素数さん
垢版 |
2020/10/07(水) 17:56:32.00ID:sGlhMxW0
ん?
CatからCatへの随伴関手になるんじゃないの
というか実際は同型関手だけど
0469132人目の素数さん
垢版 |
2020/10/07(水) 18:44:31.73ID:0gH9xfI3
そもそも“随伴関手”とは何かと何かがあって初めて“随伴してる”というもので一個の関手捕まえて“随伴”なんて言い方はせんやろ
0471132人目の素数さん
垢版 |
2020/10/07(水) 21:25:26.74ID:0gH9xfI3
まぁそれでも「opは左手随伴をもつ」といい方はしても「opは随伴関手である」なんて言い方はせんやろ
聞いたことない
0473132人目の素数さん
垢版 |
2020/10/09(金) 01:15:30.09ID:aubUZd8U
全然学部レベルの質問じゃないけど他に適当なスレがなさそうなので

相場やってる人とか計測データの処理なんかで使われるEMAの説明として
よく単純な移動平均より追従が早いと言われるけど、例えば
EMA(n) = EMA(n - 1) + (X(n) - EMA(n-1)) / 10

この式で単純に1ずつ増えるデータ
1, 2, 3, ...

を処理すると生データX(n)とEMA(n)乖離は10の単純移動平均との乖離より
大きくなるよね?

なのに一般に上のような説明が与えられるのはどういうケースが想定されてるんだろうか?
0474ID:1lEWVa2s
垢版 |
2020/10/09(金) 09:12:14.57ID:6kNfE/WV
生データ感覚🤔☺🙄☺なので。
0477132人目の素数さん
垢版 |
2020/10/12(月) 11:06:57.40ID:fXjZs723
論理式xの長さ(文字数)がlh(x)の時、xに至る論理式の形成系列の長さは < lh(x)^2 みたいなんだが、
なぜそうなるかの説明教えて
0479132人目の素数さん
垢版 |
2020/10/12(月) 11:40:23.51ID:4y/xDDAh
二項分布と幾何学分布の違いについて教えてください
0482132人目の素数さん
垢版 |
2020/10/16(金) 22:15:50.10ID:HCddLgoC
可算集合から不可算集合を作る操作って冪集合を作る以外にありますか?
0484132人目の素数さん
垢版 |
2020/10/16(金) 22:35:14.07ID:HCddLgoC
>>483
最初それとか、コーシー列全体とか考えたんですけど、結局冪集合作ってからその中で議論してるのかなと思ったんですが
0485132人目の素数さん
垢版 |
2020/10/16(金) 22:41:54.82ID:Qu2Mzn/3
「ZFCからべき集合公理を抜いたもので非可算集合の存在を証明できるか?」って疑問だと思うけど、公理見れば分かる通り無理
0487132人目の素数さん
垢版 |
2020/10/16(金) 23:10:32.19ID:HCddLgoC
>>485
ちなみになんですけど
「zfcからべき集合公理を抜いたもので不可算集合の存在を示すことはできない」をzfcからべき集合公理を抜いたもので証明することってできますか?
0488132人目の素数さん
垢版 |
2020/10/17(土) 00:00:13.47ID:4mclkkia
>>487
「zfcからべき集合公理を抜いたもので不可算集合の存在を示すことはできない」の証明にはなんでも使うんだよ
0489132人目の素数さん
垢版 |
2020/10/17(土) 00:06:46.86ID:orr9lpPC
>>488
では、別の聞き方なんですけど、
そのなんでもつかった証明の全体を考えたときに、そのうちのどの証明にもべき集合公理って必ず含まれるんでしょうか
0491132人目の素数さん
垢版 |
2020/10/17(土) 00:46:54.59ID:wcaMFYLh
証明の依存関係についての証明をするなら
公理化は避けられまいが
それが証明されたわけでもない
0492132人目の素数さん
垢版 |
2020/10/17(土) 00:51:53.50ID:5tDWKSIe
メタ論理ってどういう概念かちゃんと理解してないのよね
論理学の本読んでも「〇〇ではない」とは書かれていても「〇〇である」とは書かれてなかったり
その辺についての説明がある資料ない?
0493132人目の素数さん
垢版 |
2020/10/17(土) 07:25:34.55ID:firaX3BT
証明の算術化と算術的階層、zfcの部分と算術の等価性、強弱なんかが分かれば良のかなぁ
0495132人目の素数さん
垢版 |
2020/10/17(土) 19:57:54.17ID:5tDWKSIe
>>494
いや、メタ論理が何に使われるものなのかは分かるけど、例えばそれはZFとかで公理的に扱われうるものなのかそれともただ人間の直感のみに基づくものなのかとかが分からないのよ
0496132人目の素数さん
垢版 |
2020/10/17(土) 20:13:28.10ID:+mCYMT4v
学部一年なんですけど、定理の証明とかって全部理解の上で書けるようになるまでやるのが普通ですか?
微分積分も線形代数も集合も、理解が容易なものから困難なものまで色々あってだんだん大変で
それを覚悟して数学科に来たんだろうとか、こんなこと書いてる間に勉強しろというのはもっともなんですけど、皆さんはどうされてましたか?
0497132人目の素数さん
垢版 |
2020/10/17(土) 20:23:07.84ID:wZXvYYFx
>>496
証明とはなんであるかは小学生のうちに理解しているものだろうに
0498132人目の素数さん
垢版 |
2020/10/17(土) 20:27:49.86ID:oPZ1A1Dj
>>497
いや、中学生の単元だ
教育より先取り学習するのが当たり前と謂わんばかりの其の発言は驕りに拠る意見
0500132人目の素数さん
垢版 |
2020/10/17(土) 21:37:47.54ID:dUrzRMG6
>>495
論理なんだから直感のみであるわけがない
>>496
もちろん、全部理解の上で書けるようになるまでやれ
それが楽しみにならなきゃ上には行けん
0501132人目の素数さん
垢版 |
2020/10/17(土) 21:59:39.25ID:5tDWKSIe
>>500
直感のみでは「ない」というだけの説明ではなくてどういうもので「ある」かについての説明がある資料を教えてくれると嬉しい
0502132人目の素数さん
垢版 |
2020/10/17(土) 22:43:59.48ID:jX8N3CX8
>>473
>全然学部レベルの質問じゃないけど他に適当なスレがなさそうなので

というか(数学科で言うところの)数学の質問じゃない
0503132人目の素数さん
垢版 |
2020/10/17(土) 22:50:28.99ID:jX8N3CX8
>>496
>学部一年なんですけど、定理の証明とかって
>全部理解の上で書けるようになるまでやるのが普通ですか?
>微分積分も線形代数も集合も、理解が容易なものから
>困難なものまで色々あってだんだん大変で

どういう風に大変だと思ってるかがよく分からないけど
飛ばしたいと思ってるならどんどん飛ばしたらいい
そんなのあなたの自由
いずれ「やっぱりあれは大事だったのだ」と分かった時に
また戻ってくればいいだけ

あと微分積分なんか必死にやらなくていい
どうせルベーグ積分をあとから学べばもっと精密にスッキリ理解できるので。
線形代数も同様
学部生ならガロア理論とかルベーグ積分を目標に学んだらいいかな
0505132人目の素数さん
垢版 |
2020/10/18(日) 13:50:32.87ID:WTbeM+w4
>>501
普通に証明見て分からんの?
そんな資料ありうるんかなー
>>473
質問は構わんが説明不足で意味不明
単に君が間違ったとしか思われんよ
0511132人目の素数さん
垢版 |
2020/10/19(月) 08:20:54.26ID:G2HNtQeL
「確率論」という言葉は普通にあるけど、「統計"論"」という言葉は殆ど聞かないのは何で?
0512132人目の素数さん
垢版 |
2020/10/19(月) 15:20:49.09ID:ZFnOe3lI
統計学だからな
0514132人目の素数さん
垢版 |
2020/10/20(火) 01:14:50.81ID:ycAZBa3o
"論"という限りにおいては、○○論⊆数学 だけど、
統計学と言ったら、独立してるニュアンスが強いから、統計学⊆数学じゃなくなるよな
まぁ、公理的かどうかって言われたらそうじゃ無いからそれでいいんかな
0515132人目の素数さん
垢版 |
2020/10/20(火) 01:22:54.12ID:/B8ZIA55
2変数関数の微分積分で「領域」という言葉を定義しますが、そのときに連結であるという条件を入れるのはなぜなんでしょうか。
例えば2つのディスジョイントな開集合に分かれていたりすると、何か有名な定理が成り立たなくなったりするものなんですか?
0516132人目の素数さん
垢版 |
2020/10/20(火) 01:28:00.72ID:2uuxDCNP
連結成分ごとに考えるという意味では
0518132人目の素数さん
垢版 |
2020/10/20(火) 07:45:24.95ID:4ocklzvA
可算選択公理って集合族の要素も高々可算だとすれば、超限帰納法で証明できますか?
可算選択公理自体はZFから独立とは知ってるんですが
0519132人目の素数さん
垢版 |
2020/10/20(火) 10:37:04.57ID:/B8ZIA55
>>516
>>517

レスありがとうございます
要するに、何か不味いことが起こるというより、1つの連結成分だけとってきて議論した方が話が単純ということですかね
0522132人目の素数さん
垢版 |
2020/10/20(火) 17:38:27.22ID:4ocklzvA
>>521
可算集合の直積を帰納的に定義すれば、そのまま選択集合の集合になりませんか?
なんの構造も持たない任意の集合とか任意の添字集合に対して選択を行おうとするとZFから証明できなくなってしまうのかなと思ったんですが
0524132人目の素数さん
垢版 |
2020/10/20(火) 19:50:45.59ID:4ocklzvA
>522
そうですね
なんか、かけません
変なこと言ってしまいました
すみません
その代わりといったらなんですけど、
集合族X={A_λ:λ∈Λ }に対して、λごとに定まるA_λから自然数への全単写g_λの存在を仮定して
置換公理から
{〈A_λ.(g_λ)^-1(1)〉:λ∈Λ }が存在して
これが選択関数というのもおかしいですか?
0526132人目の素数さん
垢版 |
2020/10/21(水) 00:46:42.22ID:s5Ls+51z
>>524
(g_λ)^(-1)が個別に存在するてこと使えるんならΛが可算でなくてもいいってことなるしょ
0527132人目の素数さん
垢版 |
2020/10/21(水) 21:12:39.08ID:v4jr74Bx
R^n上の関数fがC^2級でsup||▽^2 f(x)||<∞であるとき、正定数Cが存在して任意のx,y∈R^nにおいて
f(x+y)-2f(x)+f(x-y)≦C||y||^2
が成立することを確かめろという問題です
1変数なら微分の定義からわかるのですが、n変数だとヘッセ行列のノルムが有界というのが上手く扱えなくて困っています
誰かわかる方いたら詳しい説明お願いします
0529132人目の素数さん
垢版 |
2020/10/22(木) 01:01:17.93ID:4mMP3Ip3
>>527
普通にテーラーの定理でできるやん
任意の単位ベクトルnについて
g(t) = f(x+nt)
をtの関数として見る
座標系を回転させてn=(1,0,‥,0)としてもよい
(新しい座標系でのラプラシアンは元の座標系でのラプラシアンと等しいから同じ上限値で抑えられる)
結局g(t)は全ての実数tで定義されたC2級関数で2階微分が有界
あとはテーラーの定理で桶
0530132人目の素数さん
垢版 |
2020/10/22(木) 04:47:46.61ID:yLJU+Tzp
>>529
テイラー展開の部分詳しく聞いてもいいですか?
2次まで展開して剰余項以外消えるのは何となくわかるのですが多変数だと剰余項の部分が上手く評価出来なくて
0535132人目の素数さん
垢版 |
2020/10/22(木) 23:14:52.04ID:Ys48uXb+
奇数次元の実射影空間P^2n-1に対して
それを境界に持つようなコンパクト多様体を見つけろという問題が分かりません
よろしくお願いします
0536132人目の素数さん
垢版 |
2020/10/22(木) 23:29:05.09ID:DsqZXmsg
>>535
コーンじゃダメ?
0539132人目の素数さん
垢版 |
2020/10/23(金) 00:38:46.82ID:DQ5fP+Sg
調べたら実射影空間は偶数次元の時が向き付け不可能なんですね
偶数次元では確かに上のような境界にはなりえないらしいです
0540132人目の素数さん
垢版 |
2020/10/23(金) 00:44:52.80ID:WTLClmQf
>>538
奇数だし
0542132人目の素数さん
垢版 |
2020/10/23(金) 05:27:16.75ID:ekFhnpXo
>>519
> >>516
> >>517
>
> レスありがとうございます
> 要するに、何か不味いことが起こるというより、1つの連結成分だけとってきて議論した方が話が単純ということですかね

中間値の定理で、いちいち連結成分毎にとか面倒でしょ。
0543132人目の素数さん
垢版 |
2020/10/24(土) 08:25:09.90ID:nqzBIFc7
dx/dt = αx + βy
dy/dt = γx + δy
みたいな連立微分方程式の代入法の解法は
両辺をtで微分して式を変形して代入して解いていきますが、
普通の連立方程式、例えば
y = 2x + 3
y = x + 4
両辺xやyで偏微分とかすると正しい答えにたどり着けません。

なんで連立微分方程式は両辺微分したものがイコールになるの?
俺の質問が分かって答えが分かる人、教えてちょ。
0544132人目の素数さん
垢版 |
2020/10/24(土) 08:32:33.37ID:Tr81DWkK
微分方程式は両辺の関数が等しいことを表す等式だから、同じ関数に同じ微分という操作をしても等式は成り立つ
代数方程式は両辺の実数が等しいことを表す等式だから微分するとぶっ壊れる
0545132人目の素数さん
垢版 |
2020/10/24(土) 08:51:49.97ID:nqzBIFc7
おおお
なるほど、方程式と恒等式は似ているけど違うのと同じで、
代数方程式と微分方程式は似ているけど違うって感じか。
ありがとう。
俺の頭の中でドラクエのレベルアップの音が響きそうです。
0546132人目の素数さん
垢版 |
2020/10/24(土) 09:11:31.28ID:nqzBIFc7
・・・
恒等式だったら両辺微分しても成り立ちますよね?
恒等式では変数に何入れても両辺が一致する→両辺の関数が一致する
となると、世の中で言われている「微分方程式」というのは、
「微分恒等式」というのが正確な気がしてきましたが、
なんで、微分方程式っていうのでしょうか?

知ってる人、もしくは私の気のせいとやらがおかしいのを示せる人、教えてちょ。
0547132人目の素数さん
垢版 |
2020/10/24(土) 10:24:15.94ID:EJTxjb3Q
関数の意味で等式として成立しているものを考えてる、
単に数の意味で等式が成立しているものを考えている、
どちらも方程式であり、恒等式も方程式の仲間といえる

ただ、方程式を考えるときは、
動機として方程式を満たすもの求めたい
あるいは方程式を満たすものとして数を定めたい
そういうものがあげられる
それに対して、恒等式は、考えている構造の中で(本来はそれも記述すべき)
成立が明白な方程式を扱っているといえる

XY = YX というのは行列では一般に成立しないが
高校数学でいう多項式の範囲では恒等式になっている
このように考えている構造を明白にしたほうがわかりやすい
0548132人目の素数さん
垢版 |
2020/10/24(土) 13:43:49.85ID:t94wWBzk
>>546
恒等式だったら何を入れても成り立つから解く意味がない
特定のものでしか成り立たないから解く必要があるのが方程式
0549132人目の素数さん
垢版 |
2020/10/24(土) 14:06:46.77ID:yxufgm4S
代数方程式は「よくわからないけど何らかの実(または複素)数のうち等式を満たすもの」を考えていて、微分方程式は「よくわからないけど何らかの関数のうち等式を満たすもの」を考えている
微分方程式は関数空間上での等式であり、恒等ではない方程式

もちろんその微分方程式の解となる関数を固定した上で、任意の値で成り立つ等式と見れば恒等式だけど、普通はそんな見方をしない
(代数方程式を「解空間上での恒等式」とは普通見ないのと同様)
0550132人目の素数さん
垢版 |
2020/10/24(土) 19:40:57.70ID:JCTiIqGe
ぬおおお!
明日は数検一級の試験を受けるのに説明を受けても完全には理解できん。
俺のこの程度の理解力では、明日の数検1級の合格はやばいかもしれんで、これは!

要するに解く必要がある式に対して方程式という言葉を使うとaboutに理解しました。
0552132人目の素数さん
垢版 |
2020/10/24(土) 20:22:35.96ID:Tr81DWkK
方程式:変数を含む等式のこと。変数に値(この値は実数だったりベクトルだったり関数だったりする)を代入するごとに等式の真偽が定まる。例えばy=xという方程式にベクトル(1,1)を代入すれば真、ベクトル(1,0)を代入すれば偽。df/dx=fという方程式に関数f(x)=e^xを代入すれば真、関数f(x)=xを代入すれば偽。
恒等式:(ある集合上の)どんな値を代入しても真となる方程式のこと。例えばx+x=2xはどんな実数でも真で、d(2f)/dx=2df/dxはどんな関数でも真だからこれらは恒等式。
方程式を解く:方程式が与えられて、代入したら真となる値の全体を求める問題のこと。
0554132人目の素数さん
垢版 |
2020/10/24(土) 21:07:01.78ID:yjidmAYs
>>552
せめて解の存在、一意性とかそっちなら数学的意味合い感じるけどさ。
根と解の違いとか
作用素と演算子、iとjとか。
0556132人目の素数さん
垢版 |
2020/10/24(土) 21:44:41.14ID:BGIkrf1I
あと==と===もあるよ
0559132人目の素数さん
垢版 |
2020/10/26(月) 23:02:56.54ID:MydU/ZgE
>>558
「ガロアの淫夢」で男根を取っ替え引っ替えしても気付かない淑女達って章があったじゃろ?。
0562132人目の素数さん
垢版 |
2020/10/29(木) 15:20:19.16ID:hRaVFdO3
数学の証明問題で自分の答えと本の答えがどう違うのかわかりません。

数学の証明問題を解いた時にこれが答えと自分の証明で異なる方法で証明した場合に、それが正しいのかわからず、結局自信がないので教科書の書き式を全て真似(つまり暗記)して理解しようとしてしまいます。
自分で問題を解く時、どこまでを定義として示す必要があるのか、そういうことが全くわかりません。
まるで暗闇の中で何かを手探りで探し回っているような感覚です。

今は雪江代数の群論入門を実質独学していますが、問題を見て自分で解いてみるというやり方はしておらず、答えを見てその答えを暗記しているだけのように思います。

皆さんは同じような悩みはないのでしょうか?
なぜここまでは証明する必要がある、ここまでは証明しなくてもよいなどの判断はどうやってできるようになったんでしょうか?
よろしくお願いします。
0563132人目の素数さん
垢版 |
2020/10/29(木) 17:51:08.37ID:LGVp+b0G
そういった不安がなくなるところまで定義もしくは使った定理の仮定を満たすかの確認をすべき
0564132人目の素数さん
垢版 |
2020/10/29(木) 18:18:53.43ID:pbxxzVND
>>562
俺が20才ぐらいの時にガチ数学に興味を持ち始めて取り組んだ時に持った感情と同じことを感じてるな

俺の経験で言うと、徹底的に疑って自明なレベルまで納得の出来る数式表現なり、証明の進め方なりを求めていった。
例えば、「●●である、ゆえに■■である」って議論があったら、本当にそうなのか、この”ゆえに”の部分には一体どういう議論が省略されているのか
って事を疑いまくった。
で、それを繰り返している内にちょっとずつ力がついていって、この”ゆえに”の行間を埋めれるようになって、納得が出来るレベルにまでいけた。
例えば、「Aが成り立っている、同時にAならばBでもある。ゆえに、Bである」
↑こんな議論をみたら自明すぎて”ゆえに”の行間なんてありえないだろ?
こういう風にして”ゆえに”とか”従って”とか”だから”の言葉の裏にある省略された議論を引き出し、上記のようなA,Bに当たる命題まで遡っていったりしてた。

こういうやり方はかなり時間が掛かるけど、若い内の体力ある内にこういうタイプの「筋トレ」はしておいた方が良い。
ある程度慣れてくると、知ってる議論ならこの”ゆえに”が拒否反応を示さず納得できるようになってくる。

”ゆえに”に焦点を当てて語ってみたが、”=”についても同じで、この”=”を導く際に、他のどの数式を援用したのかとかも同じように考えまくってたな、俺は。

で、自分が考えた証明と模範解答の違いだけど、向き合う根本精神は全く同じで、自分が作った証明における”ゆえに”や”だから”や”なので”
について徹底的に「本当にそうなのか?」「ツッコまれたら答えれるのか?」とか自問自答しまくるといい。


俺は、そういう風な態度で臨んだ結果、論理や式変形にまで疑問を持ち始めて数学基礎論に興味を持った。
もうここでは、数学の証明は記号変形として定義されていて俺の脳にメッチャしっくりときた。
0565132人目の素数さん
垢版 |
2020/10/29(木) 18:19:09.74ID:pbxxzVND
あと、証明の理解については別の答え方をする人も居ると思う。過去にどこかで↓下みたいな答え方をする人が居た。
つまり、証明を何度も何度も手書きしてお経を唱えるように自分の心の中で唱える。
すると、ちょっとずつちょっとずつじわーっと自分の中に議論が染みてきて、"確かにそう言われれば正しいような気がする"という感覚になってくる。
言わば、幼児が親から言語を無定義のまんま与えられ、そのロジックをそのままで獲得するような過程。


やり方は大体上記のパターンに落ち着くのでは?(比重の置き方は千差万別だろうが)


で、どこまで証明すべきか?についてだが、結局は慣れだけど、一つに今までの議論で土俵としてきた内容は証明しなくてよいということは言える。
例えば何かの集合が群をなすことを証明する場合、上記のような納得という意味では、そもそもその集合が本当に集合と言ってよいのかぐらいまで疑う気概が必要とも言えなくは無いが、
証明としては大体の場合は議論の流れ的に集合であることは明らかだから一々集合であることの証明は必要なかったり、とか。
別の視点で言うなら、ちゃんとした正しい証明になるなら、いくらでも詳しく行間を埋めて書いてもいいと思う。


長くなったけど、俺が過去に体験した心境と同じだったから、つれづれと長文書いてしまったが、頑張ってくれ
0566132人目の素数さん
垢版 |
2020/10/29(木) 18:43:25.26ID:pbxxzVND
>幼児が親から言語を無定義のまんま与えられ、そのロジックをそのままで獲得するような過程。
この言葉だが、正しいことを、そうとは知らされず(?)、与えられたままそのまま獲得していっていくという感じという意味では、
天才は大体こっち系の人が多いような気がする。学問に限らず、スポーツ、芸能、等についても。
教育者や周りが優秀な環境だと、こっち系で育つことが出来るんだろうけど。。。

凡人は疑いまくってやっと正しい知見を獲得できる、みたいな。。。
0567132人目の素数さん
垢版 |
2020/10/29(木) 19:23:49.15ID:dcmo6QTY
そもそも教科書に書いてある事を理解できたなら暗記の必要はないから言ってることが矛盾してる
自分を誤魔化してるんじゃないか?
暗記して理解だと自分を誤魔化したら自信など付かないぞ
0568132人目の素数さん
垢版 |
2020/10/29(木) 23:43:13.03ID:wU5eqOSV
ID:pbxxzVND
数学の本スレでのレスを見る限り、この人の言うことを参考にしてはいけないように思う。
0569132人目の素数さん
垢版 |
2020/10/30(金) 01:46:06.89ID:mQcjkDrR
多様体のホモロジーで閉多様体のときは最高次の整数係数ホモロジーは向き付け可能ならZで不可能なら0ですが
コンパクトでない時にはどうなるのでしょう?
直感的には0になりそうな気はしますが証明がわかりません
よろしくお願いします
0570132人目の素数さん
垢版 |
2020/10/30(金) 05:38:40.39ID:G+Q3BLjB
少なくとも境界付き多様体の内点とみなせるような場合なら境界付き多様体のポアンカレ双対

H^i(M,∂M) = H_(n-i) (M)
をを使えば

H_n(M) = H^0(M,∂M)=0

になるな
0571132人目の素数さん
垢版 |
2020/10/30(金) 05:42:16.66ID:G7V/26oS
>>537
Cone(S^n)=D^{n+1}
>>541
知らなかったのか
0572132人目の素数さん
垢版 |
2020/10/30(金) 05:43:41.97ID:G7V/26oS
>>562
>皆さんは同じような悩みはないのでしょうか?
有るわけ無いだろ
0575132人目の素数さん
垢版 |
2020/10/30(金) 12:28:47.34ID:R1dQMz0s
>>574
いや>>534はRP^kが常にk+1次の境界付き多様体の境界になる事を示せであって境界になってるやつひとつ見つけろじゃないでしょ?
0576132人目の素数さん
垢版 |
2020/10/30(金) 12:56:49.37ID:R1dQMz0s
改めて見ると>>535ってそんなスッキリした答えあるのかな?
いわゆるコボルディズムの問題で奇数nに対してRP^nはゼロコボルダントってのは言えるのかもしれないけど、なんかピタッと「RP^nは×××の境界」って言える×××があるのかな?
すごい怪しい希ガス
0577132人目の素数さん
垢版 |
2020/10/30(金) 13:14:18.95ID:1nUs0o/p
>>563
そうですよね
わかってはいるんですが…
>>564
なるほど、確かに自分がハァ?となってしまうところは証明や定義のゆえに〜等の場所で起こっていたかもしれません。
何度も何度も定義を使っていくことでここではこの定義が必要であるという感覚は養えるともわかっていても心が折れそうでした。

どちらにせよ、数学科以外の人間が独学をするのはモチベーション的にもなかなか厳しい道ですね…
アウトプットとそれの訂正をしてくれる人がいないので
代数学の講義もオンラインで教授に聞きに行けないのが辛いです。

>>567
一応本にある問題はここでこういう定義を使えば証明できるという流れは暗記しているので同じ問題であれば解けると思うんですが違う問題が出たらおそらくわからないです…

>>572
どうやったらそのレベルまで達することができたんでしょうか?
本当にすごいことだと思っています
0579132人目の素数さん
垢版 |
2020/10/30(金) 14:51:59.41ID:yGnWFiRo
>>578
入る?
少なくともPL manifoldではない
MがPL manifild ⇔ 任意の点のスター近傍がD^n
だけどRP^kのスター近傍Nはどこまで行ってもRP^kのconeで∂N=RP^kにしかならないから∂D^n=S^(n-1)であるD^nにはなり得ない
PLで無理なら可微分ならもっと無理やろ
逆はあるかも知れんけど
0580132人目の素数さん
垢版 |
2020/10/30(金) 18:50:29.93ID:1hU7Rof4
>>577
《1.数学は暗記ではなく理解なのか》

数学は暗記すべきなのか,それとも理解すべきなのか,これはよくある問いである.一般に数学はただ暗記するだけではだめで,そのわけを理解しなければならないとされ,「理解」が「暗記」の対極であるかのように考えられているが,数学はそんなに単純ではない.



円周率π(パイ)は無理数であるというのはよく知られた事実だが,その証明を知っている人は少ない.π(パイ)は無理数であることの初等的な証明はI.Nivenによるものが有名であり,以下のサイトにその証明がある.

http://mathematics-pdf.com/pdf/pi_irrational.pdf



私も中学生のときからπは無理数であることはよく「理解」していたが,大学に入学するまでその証明は知らなかった.証明を知らなかったのだから「理解」していたのではなく「暗記」していたに過ぎないと言われるかも知れない.しかし不思議なことに,I.Nivenによる証明を初めて読んだときに,それによってπが無理数であるという事実が深まった,とは感じなかった.それよりはむしろπは無理数である,という明白な事実を確認したに過ぎない,と感じた.



I.Nivenによる証明は確かに「理解」はすぐにできた.しかし,だからといって「わかった」ような気がしなかった.なぜなら証明をはじめて読んだときに,巧妙な手品を見たような感じがしたからである.では,証明を暗記すれば「わかった」ような気がするかといえばそうでもない.
0581132人目の素数さん
垢版 |
2020/10/30(金) 18:50:57.75ID:1hU7Rof4
《2.数学がわかるようになるには》

数学の本を開いて見てみると,いくつかの定義と公理があって,定理とその証明が書いてある.定理を理解するにはまず証明を読んでその論証を辿って見る.ここで,定理の証明の論証を辿ってくのは定理の証明が正しいことを確かめるというよりは,むしろ定理が述べる数学的現象のメカニズムを見るためである.



これで証明がわかればよいが,わからないときは繰り返しノートに書き写してみるとたいていの場合わかるようになる.例えば,微積分の授業で登場するε-δ論法は毎年多くの大学生を苦しめているが,これも繰り返しノートに書き写してみるとだんだんとわかるようになる.それでもわからない,という人は書き写す回数が足りていない.どんなに難しい証明も,何百回もノートに書き写せば必ずわかるようになる.



このようにして一旦わかった定理の理解を深めるためには別証を考えてみるのが有効である.なぜなら別証は定理が述べる数学的現象のメカニズムの別の見方を示すからである.



さらに,定理の理解を深めるには,定理をいろいろな問題に応用して見ることが有効である.定理を自由自在に応用できるようになればその定理は完全に「わかった」訳で,確かにいろいろ定理を応用しているうちに証明を忘れてしまうことはあるが,証明を忘れても定理がわかっていることには変わらない.証明を忘れたために定理がわからなくなるということはなく,むしろ繰り返し応用していくうちに定理そのものはますますよくわかってくる.



定理の証明を知っていたけれども忘れてしまったということと,証明は全く知らないということは非常に違うように思える.しかし,自分が証明の論証を辿ったことを知っているということと,誰かが証明の論証を辿ったことを知っているということは大差ないとも考えられる.このように考えれば,証明は知らないがよく「わかっている」定理は,証明は忘れたけれどもよくわかっている定理と大差ないことになる.ゆえに確信を持って定理を応用することができる.



最後にまとめておくと,わからない証明は繰り返しノートに写してみる,別証を考えてみる,定理をいろいろな問題に応用してみる,たったこれだけで数学はわかるようになるのである
0582132人目の素数さん
垢版 |
2020/10/30(金) 18:51:42.47ID:1hU7Rof4
元のサイトのURL貼れなかったので書き写しました


特に小平邦彦先生による数学の学び方が大変参考になった.ググればすぐわかるように,小平邦彦先生は日本人初のフィールズ賞受賞者である(注;フィールズ賞とは数学界のノーベル賞といわれるものである).


フィールズ賞受賞者も数学は暗記だ、と言っているようですね
0584132人目の素数さん
垢版 |
2020/10/30(金) 19:42:25.02ID:U2stiFDk
信じたいなら信じとけばいいからウザイこと書くな
他人には他人の方法がある
0585132人目の素数さん
垢版 |
2020/10/30(金) 20:42:06.52ID:mQcjkDrR
535ですがいろいろコメントありがとうございます

調べたてら田村一郎の微分位相幾何の本の演習問題で
閉なn次元可微分多様体M上に不動点を持たない対合σがあれば
M×I上に同値関係(x,1)〜(σx,1)を入れるとn+1次元可微分多様体になり
(この部分がまだよく理解できていませんが)
それを使うとRP^2m+1をC^m+1の単位球面S^2m+1の商と見たとき
S^2m+1上での√-1倍写像がRP^2m+1上の不動点を持たない対合を定めることから言える
という記述を見つけました
>>583のリンク先の最初の例もこれと同じもののようなので参考にして読んでみます
感謝です
0587132人目の素数さん
垢版 |
2020/10/31(土) 11:42:23.00ID:ZpLtdWPp
つまりは不動点なしのinvolutionを持てばゼロコボルダントになるんだな
0588132人目の素数さん
垢版 |
2020/11/01(日) 00:39:19.43ID:BnpeHnqO
>>580
本当に手間かけていただきありがとうございます。
勇気が出ました
人の2倍時間をかけてでもやっていこうと思います。
定義の往復による理解と別証による応用までしっかりやっていきます!
0589132人目の素数さん
垢版 |
2020/11/01(日) 04:38:56.47ID:Fdz+cM+e
>>580
>不思議なことに,…証明を初めて読んだときに,
>それによって…事実が深まった,とは感じなかった.
>それよりはむしろ…事実を確認したに過ぎない,と感じた.
>…証明は確かに「理解」はすぐにできた.
>しかし,だからといって「わかった」ような気がしなかった.
>なぜなら証明をはじめて読んだときに,
>巧妙な手品を見たような感じがしたからである.

大数学者K.K(小平邦彦)のこの発言はごもっともである

証明は「なぜ…か」に対する回答ではない
所詮「…でないと矛盾することの証」でしかない
巧妙な手品が実は本質的である、と感じたとき
納得できるのだろう
0590132人目の素数さん
垢版 |
2020/11/01(日) 04:46:09.03ID:Fdz+cM+e
>>581
>定理を理解するにはまず証明を読んでその論証を辿って見る.
>ここで,定理の証明の論証を辿ってくのは
>定理の証明が正しいことを確かめるというよりは,
>むしろ定理が述べる数学的現象のメカニズムを見るためである.

>定理の理解を深めるためには別証を考えてみるのが有効である.
>なぜなら別証は定理が述べる数学的現象のメカニズムの
>別の見方を示すからである.

これこそガウスが重要な定理の証明を
何度も異なる方法で実施した理由だろうな

登山家が同じ山を何度も異なるルートで登頂するのと同じ
0591132人目の素数さん
垢版 |
2020/11/01(日) 04:55:46.57ID:Fdz+cM+e
>>591
>定理の理解を深めるには,定理をいろいろな問題に応用して見ることが有効である.
>定理を自由自在に応用できるようになればその定理は完全に「わかった」訳で,
>確かにいろいろ定理を応用しているうちに証明を忘れてしまうことはあるが,
>証明を忘れても定理がわかっていることには変わらない.
>証明を忘れたために定理がわからなくなるということはなく,
>むしろ繰り返し応用していくうちに定理そのものはますますよくわかってくる.

ぶっちゃけていえば、定理の理解は証明の理解とは別である
応用できればわかった気になる
小学生が足し算やかけ算の可換性の証明を知らなくても
可換性を「理解」するのと同じである
0592132人目の素数さん
垢版 |
2020/11/01(日) 05:04:34.71ID:Fdz+cM+e
>>585
>RP^2m+1をC^m+1の単位球面S^2m+1の商と見たとき
>S^2m+1上での√-1倍写像がRP^2m+1上の不動点を持たない対合を定める

RP^2mの場合、R^(2m+1)の単位球面S^2mの商だが
R^(2m+1)上では、上記の「√-1倍写像」のようなものが存在しない

ぶっちゃけていうと、S^nのオイラー類に関係する
https://ja.wikipedia.org/wiki/%E3%82%AA%E3%82%A4%E3%83%A9%E3%83%BC%E9%A1%9E
0593132人目の素数さん
垢版 |
2020/11/01(日) 05:20:32.16ID:Fdz+cM+e
ところで、
1.CP^(2n)は向き付け可能だが、これを境界とする多様体は存在しない
2.しかしCP^(2n+1)を境界とする多様体は存在する

2.を示せ
0595132人目の素数さん
垢版 |
2020/11/01(日) 09:46:00.60ID:mKyX5V6R
>>587
ブラックホールを別の宇宙に移動するポータルサイトにした場合、三次元じゃない別宇宙にうまく繋げる方法ってあるのかな?。
0596132人目の素数さん
垢版 |
2020/11/01(日) 09:54:32.10ID:mKyX5V6R
>>590
同じ定理の別証明同士もたまにトンネル効果で遷移する経路、登頂"コスト"の停留点的な古典経路、と見做せそう。
0597132人目の素数さん
垢版 |
2020/11/01(日) 12:52:41.36ID:3i9WU8PF
ガウスが平方剰余の相互法則に多くの別証明を与えたのは一般化への道を探るため
0599132人目の素数さん
垢版 |
2020/11/02(月) 02:00:11.18ID:Og6J8zPI
pが真
qが真
の時にp→qが真になるというのがよくわからないです。
p,qそれぞれが真である事とp→qが真である事は別の問題だと思うのですが
0600132人目の素数さん
垢版 |
2020/11/02(月) 07:27:00.33ID:pIXYajh5
→の定義だと思うのが良いと思います
「ならば」の意味がこもった定義の「→」も色々研究されています
0601132人目の素数さん
垢版 |
2020/11/02(月) 08:14:06.42ID:DcrGqrwa
”ならば”に対してのそういう疑問って至る所で耳にする。

”ならば”と言う概念を二値論理の中に埋め込むことについて、最も不合理で無い(=最も妥協的な)真理値が現在の”ならば”の真理値表。
なんなら、他の真理値表と比較すればいい。
0602132人目の素数さん
垢版 |
2020/11/02(月) 08:16:25.33ID:DcrGqrwa
>>599
そういう真理値について疑問が沸いたなら、ファジー論理、多値論理もググるといい

砂山がある。
砂山から1粒取り除いても砂山だ。

これが真なら、同じ議論を繰り返すことによって、何もなくても砂山になるけど、そういう所の議論を探るのがファジー論理とか
0603132人目の素数さん
垢版 |
2020/11/02(月) 08:31:34.48ID:he65m0K3
様相論理ってのが面白そうだけどあまり知らない
0605132人目の素数さん
垢版 |
2020/11/02(月) 13:25:06.06ID:/pAuBbgN
記号を使った抽象的な論理式で考えてるから分からなくなってる可能性がある
6は3の倍数だ→6は偶数だ
という命題が真なのは納得行く?
0606132人目の素数さん
垢版 |
2020/11/02(月) 14:09:57.70ID:TVmeClQG
>>599
直観主義論理というのがあって、そっちの考えの方があなたにはあってるかもしれないですね

AならばB

これは普通に考えたら、Aが真ならば必ずBも真になるということを意味しています

こう考えた時、AとBというのは変数的なもので、その命題の真偽というのは絶対的には決まらない相対的なものである、という感覚が何処かにあるのです

だから、命題の真偽は絶対的なものだと考える古典論理は感覚と合わないのですね
Aの真偽が決まる前にBの真偽はすでに決まっていると考えているから気持ち悪いのです

命題の真偽を相対化する仕組みを兼ね揃えた論理体系が直観主義論理です

クリプキモデルというのがあって、そこでは命題の真偽は絶対的なものではありません
場合によって変わり得るものだと考えます

それで、クリプキモデルにおけるAならばBの定義は、まさに、Aが真のときは常にBも真である、というような定義になっています
0607132人目の素数さん
垢版 |
2020/11/02(月) 14:15:01.14ID:Og6J8zPI
正直いって今は納得行きませんね
最初は違和感なくそんなもんかなと思ってたんですが
結論しか見てないならqが成り立ってりゃ何でもいいってことで 条件を考えている時のならばと扱いが違うように思うのですよ。

そもそも
命題の話をするとp,qがさりげに関係あるような感じの具体例を提示するから余計に混乱するんですが
pとqが命題であればなんでもいいんで

p 6は3の倍数である
q リンカーンは男だ
においても認められる事で
p→qも真だし
q→pも真だし

pとq同値なん?って頭がバグるんすよ
0609132人目の素数さん
垢版 |
2020/11/02(月) 14:23:23.70ID:TVmeClQG
>>607
クリプキモデル勉強してみるのが良いと思いますよ

あなたの疑問の答えがあります

直観主義論理を学んだ上で、古典論理はそこからどのような変更を加えているのかがわかれば、古典論理の定義も納得できるはずです
0610132人目の素数さん
垢版 |
2020/11/02(月) 14:29:31.03ID:vCYYR8um
→の読み方は「…だから…」ではなく「もし…ならば…」である、っていうのは違和感の答えになる?
真偽が与えられるのは推論の妥当性に対してではなく内容そのものに対してというか
0611132人目の素数さん
垢版 |
2020/11/02(月) 14:31:27.06ID:Og6J8zPI
>>608
んーまぁそれは理屈として知ってはいるんですよ。
昔は何の疑問も抱いていませんでしたし
むしろよくありがちな前件が偽の時には何であろうと真であるなんかは最初から疑問を抱いていませんでしたし今も何の疑問も抱かないぐらいですしね。

論理学的な同値な変形も今まで特に内容も吟味せずに当たり前の変換として使ってきてましたし

>>609
ありがとうございます。ちょっと調べてみます。何かお勧めな導入書的なものありますか?
0612132人目の素数さん
垢版 |
2020/11/02(月) 14:57:52.30ID:Og6J8zPI
>>610
あーif thenで考えると たしかになんか日本語よりしっくりくる気がします。
まぁでもホントに従属節なんてどうでもいいんですね。

自分でもなんでこんな事に疑問持つのかも分からんぐらい当たり前だったんですよ。呼吸する事意識し始めたら何か呼吸の仕方よく分からなくなって苦しいみたいな感覚なんです。
0613132人目の素数さん
垢版 |
2020/11/02(月) 15:08:20.13ID:lnucLH6c
>>610
条件文p→qは「pならばq」または「pはqの十分条件である」と読む
「pが真のときqは真である」という文が偽になるのはpが真のときqが偽であるときのみ
0616132人目の素数さん
垢版 |
2020/11/02(月) 16:49:46.83ID:72bY7xJF
実際の数学でも、(ある前提のもとで)AならばBという定理が証明されたあと、Aという条件は不要だったということが分かったりする。
AとBの間に本当に論理的関係があるのかどうかは難しい問題なので、形式論理の範疇を超える。
0617132人目の素数さん
垢版 |
2020/11/02(月) 17:12:03.00ID:qmyf9gGz
古典論理の範疇を超える、ですよ

ある段階での知識量において真偽は判断すべきである、というのは直観主義の考え方です

昔はAという条件がないとB証明できなかったけど、研究が進むにつれてAがなくても証明できることがわかった

これは直観主義論理で説明可能です
0618132人目の素数さん
垢版 |
2020/11/02(月) 17:31:26.24ID:DcrGqrwa
>>617
そんな直観主義の説明は初めて聞いたんだが?

俺の理解では直観主義は有限性や有限的手続きと結びつく概念なんだが
0619132人目の素数さん
垢版 |
2020/11/02(月) 17:47:08.96ID:7vrmI6Rt
古典論理が納得いかんって言ってるレベルの人間が直観主義数学なんてもんに手出して物になるわけない
0620132人目の素数さん
垢版 |
2020/11/02(月) 18:00:00.92ID:qmyf9gGz
>>618
それは統語的な側面ですよね

意味論としてのクリプキモデルを学べば、上のような解釈ができますよ
0621132人目の素数さん
垢版 |
2020/11/02(月) 18:16:04.81ID:pIXYajh5
>>617>>620
出典プリーズ
多分君の主張と少し違うことが書いてある
0623132人目の素数さん
垢版 |
2020/11/02(月) 18:24:37.31ID:pIXYajh5
>>622
自分の主張に合致した出典が出せないから誤魔化してるね
文献を眺めてるだけの人がよくやる
0624132人目の素数さん
垢版 |
2020/11/02(月) 18:28:31.60ID:qmyf9gGz
可能世界の到達可能関係、って言ったってあなたわかりませんよね?
ちゃんと勉強してから文句は言いましょうね
0625132人目の素数さん
垢版 |
2020/11/02(月) 18:40:19.66ID:pIXYajh5
学問は口調だけ優位に立ってもなんの意味もない
0626132人目の素数さん
垢版 |
2020/11/02(月) 18:45:09.98ID:he65m0K3
クリプキモデルは名前しか知らないけど
直観主義論理は単にP∨¬Pが恒真にならないってだけでしょ
そういうラティスを何て言ったっけな
実数の開集合の全体で
¬Pを(P^c)^oって定義する奴みたいな奴
0627132人目の素数さん
垢版 |
2020/11/02(月) 18:52:04.79ID:qmyf9gGz
ハンティング代数ってやつですか?

そっちはあまりよく知りませんけど、クリプキモデルの方が意味づけとしてはいい気がしますけどね
0628132人目の素数さん
垢版 |
2020/11/02(月) 18:58:38.20ID:qmyf9gGz
ちなみにですけど、直観主義論理でなぜ排中律が成り立たないかは、クリプキモデルで簡単にわかります

現段階の知識量をもってして、ある命題Aやその否定¬Aが正しいかどうか判断することは、一般にできないからです

リーマン予想とか典型的な例ですね

知識量が極大となってこれ以上新たな知識を取り得ることはあり得ないという、神の視点に立った時の論理が古典論理です

リーマン予想が正しいか正しくないかは、神のみぞ知るわけですね
0630132人目の素数さん
垢版 |
2020/11/02(月) 19:10:47.83ID:Z2T5Dnk9
めちゃめちゃやんwww
こんな論がたつなら今ある未解決問題全部神のみぞ知るやんww
一知半解でいい加減な言葉尻追っかけただけの議論で悦に浸ってるからこんな妙チキリンな結論に辿り着くんだよ
0631132人目の素数さん
垢版 |
2020/11/02(月) 19:12:32.98ID:qmyf9gGz
可能世界とはそういう概念ですよー
到達可能関係も同じです

悔しかったら、なぜ直観主義論理では排中律成り立たないのか説明してみてくださいねーw
0632132人目の素数さん
垢版 |
2020/11/02(月) 19:14:11.11ID:qmyf9gGz
直観主義論理では、¬Aが真であることと、Aが偽であることは違うことです

こんなのもここの住人たちは知らないのですね
0634132人目の素数さん
垢版 |
2020/11/02(月) 19:16:58.97ID:qmyf9gGz
直観主義論理では¬¬AとAは等価でない

なぜか?

直観主義論理ではAが偽であることと¬Aが真であることは違うから

これすらここの回答者はわからないんですねー
0636132人目の素数さん
垢版 |
2020/11/02(月) 19:24:24.95ID:1OyCoMWb
>>634
ゲーデル解釈を知っていると、低レベルな事をいってるなあ、としか思わない。
0637132人目の素数さん
垢版 |
2020/11/02(月) 20:45:28.02ID:DcrGqrwa
>>631
証明論入門(数学基礎論改題) (竹内外史,八杉満利子) 共立出版 65ページ
  Aが論理記号を含まない時、「   ⇒ A∨¬A」はLJでProvableでない
が証明されてる
0638132人目の素数さん
垢版 |
2020/11/02(月) 20:52:52.46ID:DcrGqrwa
>>628
復刊 数理論理学、松本和夫、共立出版、55ページ

直観主義の立場では、A∨¬Aが成り立つというのは、次のうち少なくとも1つが成り立つことであった。
1 Aが成り立つことが証明されている。
2 Aが成り立つという証明が与えられたと仮定した時、この事から矛盾を導くことの出来る具体的な手段を持っている。
3 A,¬Aのどちらか一方が成り立つことを示す具体的な手段を持っている。しかしAとして現在未解決の問題を取ってくれば1〜3のいずれも成り立たないことは明らかである。

まぁ、俺のレスは形式論の立場からの言及だな
意味論の立場からの直観主義論理についてなんか本ある?
0639132人目の素数さん
垢版 |
2020/11/02(月) 21:05:51.71ID:qmyf9gGz
私は適当なpdf見ただけなのでよくわかりませんね

クリプキ意味論に基づく直観主義論理の完全性定理の証明とか書いてあったのがどっかにあったと思うんですけど見当たりません

削除されたんですかね
0642132人目の素数さん
垢版 |
2020/11/02(月) 21:25:12.93ID:qmyf9gGz
>>638
に私が言ったリーマン予想と同じこと書かれてるじゃないですか

それでも疑うんですか??
0644132人目の素数さん
垢版 |
2020/11/02(月) 22:00:19.26ID:pIXYajh5
ID:qmyf9gGzは余分な前提の話と構成的な証明の話をごっちゃにしたから間違えちゃったんだね
0646132人目の素数さん
垢版 |
2020/11/02(月) 22:37:06.65ID:he65m0K3
>>627
クリプキモデルの方を知らないから何とも言えないんだけど
古典論理の意味を考えると言うよりはブール代数考えるのが直接的で理解しやすいんじゃない?
直観主義論理の意味を考えるよりはハイティング代数か
それ考える方が直接の理解に繋がるんじゃないかなあ
0647132人目の素数さん
垢版 |
2020/11/02(月) 22:48:19.51ID:qmyf9gGz
>>643にクリプキモデルの説明あるので見てみてくださいね

私も後でハイティング代数についても探してみます
0648132人目の素数さん
垢版 |
2020/11/02(月) 23:01:28.65ID:Z2T5Dnk9
ググれば出てくるとかまーたネットでチョロチョロ読んだだけでわかった気になってる
前回のゲーデルの不完全定理の理解もボロボロやったし
いつまでたってもおんなじレベルで足踏みしとる
0649132人目の素数さん
垢版 |
2020/11/02(月) 23:03:22.33ID:pIXYajh5
>>645>>647
偉そうにしても無駄だ
0651132人目の素数さん
垢版 |
2020/11/02(月) 23:16:21.40ID:voSFvcMU
直観主義論理と余分な前提の関連を説明できないID:qmyf9gGzは何も分かっていない
0652132人目の素数さん
垢版 |
2020/11/02(月) 23:18:55.67ID:qmyf9gGz
余分な前提というの何かと思ったら、もしかしてクリプキモデルのことですか??

統語論と意味論の話理解してないから、意味論が余分に思えるだけですよそれw
0653132人目の素数さん
垢版 |
2020/11/02(月) 23:19:32.05ID:qmyf9gGz
統語論と意味論は等価である

完全性定理の主張はこれなわけですが、意味論は余分な前提なのですねー
0654132人目の素数さん
垢版 |
2020/11/02(月) 23:31:54.85ID:pIXYajh5
馬鹿を晒している
元の話はA→BでAがBと関係ない命題でもかまわない事への違和感だ
私はAの事を余分な前提と言っている
0655132人目の素数さん
垢版 |
2020/11/02(月) 23:46:43.02ID:qmyf9gGz
クリプキモデルならば、全ての命題は可能世界を変数に取る述語として考えることが可能ですから、関係ない命題は存在しません

AならばB

↑Aが真となる任意の可能世界において、Bもまた真となる、という意味ですよ、クリプキモデルで解釈するならば
0656132人目の素数さん
垢版 |
2020/11/03(火) 00:10:39.63ID:lFLkta16
それは君の解釈が間違っている
0657132人目の素数さん
垢版 |
2020/11/03(火) 00:12:04.43ID:lFLkta16
クリプキ解釈の解釈が間違っている、という意味ね
0658132人目の素数さん
垢版 |
2020/11/03(火) 00:16:53.20ID:WzeT9Eh0
あなたがわかってないだけですよね

クリプキモデルにおいて、A→Bが真となるときはどういう時か説明してみてくださいねー
0659132人目の素数さん
垢版 |
2020/11/03(火) 00:35:57.80ID:lFLkta16
>>658
あれ?君は


知識量が極大となってこれ以上新たな知識を取り得ることはあり得ないという、神の視点に立った時の論理が古典論理です

と言って古典論理を批判していたのに、任意の可能世界みたいな超越性はOKなんだ?
0661132人目の素数さん
垢版 |
2020/11/03(火) 01:16:32.23ID:WzeT9Eh0
>>659
意味不明なんですけどw

別に古典論理の批判なんてしてないですよ

直観主義論理と古典論理の関係を述べただけです
0662132人目の素数さん
垢版 |
2020/11/03(火) 03:03:36.87ID:2/pn8sKH
出典もあやふやな情報でドヤってる時点で数学どころかいかなる分野の専門教育も受けていないことが分かる
0663132人目の素数さん
垢版 |
2020/11/03(火) 05:51:55.85ID:eqSA35Si
>>643 5ページ
補題 3.3 (遺伝性).
任意の命題論理式 φ と Kripke モデル M = (W, ?, V ),世界 w ∈ W について,
w |= φかつ w ? w' ならば w'|= φ となる.


遺伝性は感覚的には「一度確定した,あるいは証明された事実は覆らない」ということを表現しているのだということもできる.
w |= ¬φ つまり w |= φ → ⊥ は,任意の w' ? w に対し w' ?|= φ であることと同値である.
つまり「これ以降,未来永劫 φ が証明されることはない」ということであり,
したがって現時点で φ であることが確信できないからといって w |= ¬φ を結論することはできない.
今はまだわからなくても,いずれ φ が正しいことが確定する日が来るかもしれないからである.
0665132人目の素数さん
垢版 |
2020/11/03(火) 09:50:04.95ID:sBsLer6j
>>647
>>>643にクリプキモデルの説明あるので見てみてくださいね
Vは命題変数からWのcofinalでfaithfulな部分集合への写像だけど
論理式からWへの写像に拡張して考えると
V(P)がPが「真」であることを意味するわけですね

単純な例として実数Rに通常の順序を入れたものがWのときは
V(P)はφかRか右開区間だけど
V(P)がφかRかしか無い場合が古典論理に当たると

V(⊥)=φ
V( T )=R
∧∨は単にかつまたはで定義するつまり
V(P∧Q)=V(P)∩V(Q)
V(P∨Q)=V(P)∪V(Q)
そして
V(P→Q)=R (V(P)⊂V(Q)のとき)or V(Q) (それ以外)
V(¬P)=V(P→⊥)=R (V(P)=φのとき) or φ(それ以外)
V(P∨¬P)=V(P∨(P→⊥))=R(V(P)=φのとき) or V(P)(それ以外)

<が時間の前後というか左の「先(行く末)」に右があるみたいな解釈
0666132人目の素数さん
垢版 |
2020/11/03(火) 09:59:02.42ID:sBsLer6j
>>665
>論理式からWへの写像に拡張して考えると
V(P)={w|>>643の説明でのw|=P}としました
こうしたら{V(P)}は包含でハイティング代数にならないかなあ
0667132人目の素数さん
垢版 |
2020/11/03(火) 10:10:34.47ID:sBsLer6j
>>665
>V(P)はφかRか右開区間だけど
右開区間って言うんじゃ無いんだっけ
右が∞の区間のこと
0668132人目の素数さん
垢版 |
2020/11/03(火) 10:16:17.99ID:sBsLer6j
>>665
>V( T )=R
というよりV(P)=RであるPのことを恒真と考えるというべきか
0669132人目の素数さん
垢版 |
2020/11/03(火) 10:27:10.67ID:sBsLer6j
Vの定義はそれこそ無数にあるから
その全てでV(P)=Wなものが恒真命題と

たとえば
P→P∨Qはどんなw∈Wに対しても
w∈V(P)なら∨の定義からw∈V(P∨Q)だからw∈V(P→P∨Q)だし
そうでなくても→の定義からw∈V(P→P∨Q)だし
0670132人目の素数さん
垢版 |
2020/11/03(火) 10:31:23.35ID:sBsLer6j
w'出てきてないのはV(P)に先々まで全部入れちゃってるからこれでいいと思ったけどどうかな
0671132人目の素数さん
垢版 |
2020/11/03(火) 10:40:47.73ID:sBsLer6j
同じRを基底集合としてもブール代数はR自体には順序考えず
部分集合全体の包含で
V(⊥)=φ
V( T )=R
V(P∧Q)=V(P)∩V(Q)
V(P∨Q)=V(P)∪V(Q)
ここまでは同じで
V(P→Q)=V(P)^c∪V(Q)
V(¬P)=V(P→⊥)=V(P)^c
とするから
V(P∨¬P)=V(P∨(P→⊥))=V(P)∪V(P)^c=R
となって恒真と
0672132人目の素数さん
垢版 |
2020/11/03(火) 11:11:19.35ID:sBsLer6j
Rの開集合全体の包含によるハイティング代数の場合は
V(⊥)=φ
V( T )=R
V(P∧Q)=V(P)∩V(Q)
V(P∨Q)=V(P)∪V(Q)
V(P→Q)=(V(P)^c)^o∪V(Q)
V(¬P)=V(P→⊥)=(V(P)^c)^o
とするから
V(P∨¬P)=V(P∨(P→⊥))=V(P)∪(V(P)^c)^o=R(V(P)=φ,Rの場合) or ≠R(それ以外)となる
0673132人目の素数さん
垢版 |
2020/11/03(火) 11:16:09.68ID:sBsLer6j
>>666
>こうしたら{V(P)}は包含でハイティング代数にならないかなあ
Rの普通の順序によるクリプキモデルから作った
{V(P)}={φ,(a,∞),[a,∞),R}は確かにハイティング代数になってるけど
一般にはどうだろ
多分なってるとは思うけど
0674132人目の素数さん
垢版 |
2020/11/03(火) 11:37:06.25ID:5LtiWQS8
私は頭のおかしい人を相手にしていたようだ
馬鹿な事をした
0675132人目の素数さん
垢版 |
2020/11/03(火) 11:57:27.84ID:sBsLer6j
君は ID:pIXYajh5 ID:lFLkta16 の人? ID:eqSA35Si の人も?
ID:qmyf9gGz ID:WzeT9Eh0 の人じゃナイよね?
俺は ID:he65m0K3
0676132人目の素数さん
垢版 |
2020/11/03(火) 12:37:33.83ID:WzeT9Eh0
>>664
ほぼ同じですよね
到達可能とか言ったってわかんないだろうから省いただけですよー
0677132人目の素数さん
垢版 |
2020/11/03(火) 13:07:36.73ID:sBsLer6j
>>672
>V(P→Q)=(V(P)^c)^o∪V(Q)
間違えた
正しくは
V(P→Q)=(V(P)^c∪V(Q))^o
だので
V(P∨¬P)=V(P∨(P→⊥))=V(P)∪(V(P)^c)^o=R(V(P)=φ,R)or≠R(それ以外)
だけど
V(P→P)=(V(P)^c∪V(P))^o=R
0678132人目の素数さん
垢版 |
2020/11/03(火) 13:11:56.69ID:sBsLer6j
>>673
>多分なってるとは思うけど
なってた
だからクリプキモデルもハイティング代数も等化じゃないかな
ハイティング代数からクリプキモデルが構成できることを言えば良い
0679132人目の素数さん
垢版 |
2020/11/03(火) 13:27:44.47ID:WzeT9Eh0
等価なのはもちろんそうですよ

私がハイディングの方知らなかっただけです

でも可能世界の方が意味的にいい気がするというお話ですね
0680132人目の素数さん
垢版 |
2020/11/03(火) 14:25:06.48ID:sBsLer6j
自分はラティスで考える方がしっくりくるな
ブール代数はごく普通の
全体集合のあるベン図によって理解できるし
時間に関する用語(将来とか)を使うのに違和感があるから
けど
様相論理というのがそういうものらしいから
クリプキモデルの方が発展性あるのかもね
様相論理も名前しか知らないけど
クリプキモデルの主戦場はそこらしいじゃん
0681132人目の素数さん
垢版 |
2020/11/03(火) 14:34:33.97ID:zq3/CRH+
>>593
2.はRP^2m+1の時C^m+1を考えたところを四元数Hを使ってH^m+1で置き換えて
同じくi倍写像で対合を作ればよい感じですか

オイラー類はまだよく知らないのですが
関係が分かるようになることをとりあえずの目標にして勉強してみます
いろいろとありがとうございます
0683132人目の素数さん
垢版 |
2020/11/03(火) 17:53:33.62ID:JF6/IDZ0
記号論理学における推論規則って、論理式の集合から論理式の集合への写像だと考えるのは間違っていますか?
言語を記号の集合として定義するのはたまに見かけるんですが、推論規則を集合論で定義してる書籍を見かけないのですが
0685132人目の素数さん
垢版 |
2020/11/03(火) 18:05:00.05ID:Ea5rEgSq
>>682
一応細かいチェック入れてない証明っぽいもの持ってる
それは特性類の理論は使ってない
初等的だけど煩雑ってやつ
なので難しい話し知らなくてもなくてできるのはできると思う
0686132人目の素数さん
垢版 |
2020/11/03(火) 18:17:46.01ID:sBsLer6j
>>683
>論理式の集合から論理式の集合への写像
一部にしか適用できないよ
定義域のある写像?
どこに適用するかも決める?
0687132人目の素数さん
垢版 |
2020/11/03(火) 19:29:26.37ID:F9WRUhYe
>>681
CP^2m+1をH^m+1の単位球面S^4m+3の商と見たとき
S^4m+3上でのi倍写像がCP^2m+1上の不動点を持たない対合を定める
・・・かどうかはご自分で確かめてください
0688132人目の素数さん
垢版 |
2020/11/03(火) 19:33:50.43ID:F9WRUhYe
なお、CP^2mの場合は同じ手は使えないし、実際境界にはならない
ただ、その証明はコボルディズムの話だから難しい
0689132人目の素数さん
垢版 |
2020/11/03(火) 20:08:36.84ID:Ea5rEgSq
m=0のときもうダメやん
H^を左からC^で割ってCP^1とみなす
右からHは作用してるけど
i倍は1が不動点
j倍でも1+i+j-k
(1+i+j-k)j = j+k-1+i=i(1+i+j-k)
なので1+i+j+j-kが不動点
そもそもCP^1は不動点なしのinvolutionもたないと思う
0690132人目の素数さん
垢版 |
2020/11/04(水) 10:27:21.05ID:u7ff//2f
>>686
モーダスポーネンスの場合は
言語Lの論理式の全体の集合をFとして
F×F×Fの要素で〈ψ,ψ→φ,φ〉の形をしているもの全体をMPとしたときのdomMPが定義域かなと思ったんですが
論理式の形を区別する部分が言語Lで書けないからフォーマルな写像にはならないのですかね
0691132人目の素数さん
垢版 |
2020/11/04(水) 11:34:19.87ID:q5Zd2nMX
幾何学の対称モノイダル圏と基礎論の対称モノイダル圏とでの共通認識って何だろうね。
0693132人目の素数さん
垢版 |
2020/11/04(水) 13:19:27.31ID:kdWwbDED
やっぱりCP^n^n→CP^nは固定点ないinvolution持てないな

(∵) まずC^∞のときを考える
固定点持つものがあるとして f : S^2n → S^2n をその持ち上げとする
この時任意のS^(n+2)上の点pに対してf(p)≠±pであるからpとf(p)を結ぶ大円の劣弧が一意に決まる
この経路の定める接ベクトルX(p)を考えれば全ての点で0にならないベクトル場が定まるがつむじの定理に矛盾する

http://physmathseminar.web.fc2.com/discourse/2016/spring/ochanomizu-sp_abst/ocha_hori.pdf

一般の場合は十分小さいεでε近似したC^∞級の関数fεに対し必ずf(pε)=±pεを満たすpεが取れるが{pε}の集積点pがf(p)=±pを満たす
0694132人目の素数さん
垢版 |
2020/11/04(水) 16:01:25.31ID:8hRnPJ6W
MathJaxについての質問。(Javascript質問系のスレは大分更新されてないので)

MathJax ver3で使える、機能豊富な可換図式描画ライブラリって何がある?
0695132人目の素数さん
垢版 |
2020/11/05(木) 00:46:57.31ID:dkTicM1m
>>694
自分で作って公開
0696132人目の素数さん
垢版 |
2020/11/05(木) 00:47:25.59ID:dkTicM1m
>>691
定義が同じ
0697132人目の素数さん
垢版 |
2020/11/05(木) 17:20:54.84ID:vWIWN4jE
すまん>>685です
自分が持ってると思ってた証明間違ってた
というわけで私初等的な証明持ってません
wikiに載ってるwhitney class関連の公式全部認めれば>>593が正しいのは確認できるけど
やっぱりコレ初等的に示すの難しい希ガス
0698132人目の素数さん
垢版 |
2020/11/05(木) 20:05:20.12ID:vWIWN4jE
自分で正しいと思ってる(black boxだらけだけど)>>593の証明

thm
CP^n がzero cobordant iff n : odd

lem
cp^n の最高次のwhitney number ≡ n+1 (mod 2)
(∵) 一般に最高次のwhitney number ≡ euler 標数 (mod 2) であるが、
β_i(CP^n) = 1 (if 0≦i≦2n, i:even)
. 0 (otherwise)
より主張は従う

lem
i≦m ≦ n に対して自然な埋め込みの誘導するH^i(CP^n,Z/2Z)→H^i(CP^m,Z/2Z)は単射
(∵) iが奇数なら両方ゼロだからiは偶数として良い
この時両方一次元だから0でない事をしめせばよい
そのためには誘導されるホモロジー群の準同型が0でない事を示せば良い
埋め込みの像はF=“[x0,x1,‥,xm,0,‥,0]の全体“でこれは“G=[0,‥,xm,‥,xn]“の全体と一点で交叉的に交わるのでFとGのコホモロジー類はともにゼロではない

(thmの証明) nが偶数のとき補題より最高次のホイットニー類が0でないのでzero cobordantではない
nが奇数とする
最高次のホイットニー類は補題より0
2n=i1+i2+‥+ikを2nの分割としてn
が奇数だからいずれかは奇数、または4の倍数でない偶数
iが奇数のときi次のホイットニー類は0
i=2m、m奇数のときi次のホイットニー類wiを自然な埋め込みでCP^mに引き戻すと補題により0
しかしこの引き戻し写像は単射であったからwiは0
0702132人目の素数さん
垢版 |
2020/11/07(土) 19:08:31.06ID:nKb9K0zx
>>698
いろいろとありがとうございます
特性類はミルナーの本で勉強しながらですがホイットニー数が全て0になればゼロコボルダントという結果を認めれば言える事は理解できました
ただその証明はかなり難しいみたいですね

あと>>693の証明についてですがCP^n→CP^nがS^2n→S^2nにリフトされるというのはすぐ分かることでしょうか?
S^2n+1からCP^nの射影しか知らずどう構成するのかが
0703132人目の素数さん
垢版 |
2020/11/07(土) 19:42:37.89ID:ffVIZT4D
>>702
π : S^2n→CP^nはcoveringなんだから任意の弧状連結空間Xからの連続写像 f : X → CP^n は X → S^2n に持ち上がるでしょ?
HLPとかいうやつ
0704132人目の素数さん
垢版 |
2020/11/07(土) 19:52:28.62ID:nKb9K0zx
>>703
被覆S^2n→CP^nがどう取れるのかで詰まってます
実の時と同じように考えるとC^n+1での球面S^2n+1からの射影しか得られなくて
0705132人目の素数さん
垢版 |
2020/11/07(土) 20:26:30.16ID:ffVIZT4D
ごめん
弧状連結じゃなくて単連結
一般にXが単連結、Y→Zが被覆写像ならg:X→Zはf:X→Yに持ち上がる
適当にx0を固定して各xごとにpath μ:x0→xを選んでおく
それとg(x0)の持ち上げy0も選んでおく
gはコレをg(μ):g(x0)→g(x)にうつすけどこのパスはY→Zがcoveringならこのパスはy0→yに持ち上がる
この終点をf(x)と定める
パスの選び方によらないのは別のパスμ'でやるとμ^(-1)μ'はループになるけどXが単連結ならこのループはある特異2単体の境界
この特異2単体ごとYに持ち上がるのでμの持ち上げの端点=μ'の持ち上げの端点になる
0707132人目の素数さん
垢版 |
2020/11/07(土) 21:53:02.32ID:iJVmc3BL
>>703
CP^nのidentityはCP^n→S^nに持ち上がるか
0708132人目の素数さん
垢版 |
2020/11/07(土) 22:03:47.17ID:iJVmc3BL
>>703
>π : S^2n→CP^nはcovering
じゃないが
0709132人目の素数さん
垢版 |
2020/11/07(土) 22:35:47.03ID:ffVIZT4D
>>703
だから訂正してるやん
Xは単連結でないと持ち上がらん
>>703
すまん
そこは間違い
ちょっと修正できるかみてみる
0710132人目の素数さん
垢版 |
2020/11/07(土) 22:40:03.46ID:ffVIZT4D
あぁS^2n→CP^nはつむじの定理でCP^oddの固定点なしのinvolutionの時に使ったやつだな
結果は合ってると思うんだけど
0711132人目の素数さん
垢版 |
2020/11/07(土) 22:42:53.31ID:ffVIZT4D
少なくともn=1の時、つまりリーマン球面の時固定点ないinvolutionないのはつむじの定理そのままやな
0713132人目の素数さん
垢版 |
2020/11/07(土) 23:14:06.13ID:ffVIZT4D
ダメだ
やっぱりCP^oddが固定点持たないinvolutiomもつか持たないかはそんなに簡単には答えられそうにない
というわけで>>693は撤回します
とは言えじゃあCP^oddが必ずそう言うinvolution持つのかも答えられないし
その上の方でできたってやつもちょっとおかしいみたいだし
0714132人目の素数さん
垢版 |
2020/11/07(土) 23:33:01.38ID:iJVmc3BL
>>709
>Xは単連結でないと持ち上がらん
CP^nは単連結だが
0716132人目の素数さん
垢版 |
2020/11/07(土) 23:40:25.58ID:ffVIZT4D
結局今のところCP^oddがゼロコボルダントの初等的な証明は今のところないのは合ってる?
上の方にある証明はm=0の時成り立ってないとおもうけどそれ以外ではいけるとかある?
0717132人目の素数さん
垢版 |
2020/11/08(日) 00:31:39.79ID:MuM62ej1
イヤ違う
jかける作戦うまく行くのか
{ x ∈ H^n | |x|^2 = 1 }
をG = { z | |z| = 1 }の右からのactionで割るとき、j倍の作用も右からかければいいのか
通常右で割った加群には右側加群の構造は期待できないけど今は「商空間に作用できるか」しか問題にしてないから右から作用させられるんだ
実際(Yk) = (Xk) cis(θ) (Xk),(Yk)∈H^n が等しい右側G軌道にある)とき
(Yk) j = (Xk) cis(θ) j = (Xk) j cis(-θ)
により(Xk) jと(Yk) jは等しい右側G軌道にあるからこの右側 j 倍作用はwell definedなんだ
しかもXk=0である場合を除いて(Xk) j = (Xk) cis(θ)となるθが存在することもない
吊ってくる
0719132人目の素数さん
垢版 |
2020/11/08(日) 02:45:11.00ID:MuM62ej1
イヤ違う
n:evenならC^(n+1) = H^(n+1)/2 がそもそも無理なのか
お騒がせしました
0720132人目の素数さん
垢版 |
2020/11/10(火) 08:28:05.58ID:qTo2VM4J
>>702
>S^2n+1からCP^nの射影しか知らずどう構成するのかが
CP^nの単体分割は
e^0∪e^2∪e^4∪…∪e^2n
おそらく
i:e^2n⊂CP^n⊂CP^∞:[z1:…:zn:1:0:…]
と誤解している
0721132人目の素数さん
垢版 |
2020/11/11(水) 12:00:02.98ID:vZr2WsHv
解析(でも何でもいいが)で、x^yはしょっちゅうでてくるのに、何でx^(y^z), x^(y^(z^w))ってほぼ現れないんですか?
0722132人目の素数さん
垢版 |
2020/11/11(水) 14:55:38.71ID:lobS0qKZ
>>721
ラムゼイ理論とかやるとしょっちゅうあらわれる
0724132人目の素数さん
垢版 |
2020/11/14(土) 20:53:02.17ID:rL3ay87E
論理って一階述語論理だけやればいいですか?高階論理ってやる意味ありますか?
0726132人目の素数さん
垢版 |
2020/11/14(土) 23:05:40.17ID:rL3ay87E
>>725
それなら一階述語論理で十分みたいですね
0728132人目の素数さん
垢版 |
2020/11/21(土) 15:53:58.75ID:pqihBUM1
語彙L={∈}に対する構造∈-モデルって声に出すときなんて言えば良いですか?
0729132人目の素数さん
垢版 |
2020/11/22(日) 13:27:43.67ID:UjdrCFnt
群Gはその部分群Hによって左剰余類に関する類別が得られることは成り立ちますが、
その逆で群Gのとある部分集合Sと、Gの任意の元gを使ってgSによってGの類別が可能ならばその部分集合SはGの部分群であることは成り立つのでしょうか?
0730132人目の素数さん
垢版 |
2020/11/22(日) 15:01:43.63ID:03o3b8Sl
>>729
ダメ
0731132人目の素数さん
垢版 |
2020/11/22(日) 17:50:47.72ID:UjdrCFnt
>>730
Sは部分群とは言えないのですね。
となるとGを類別できるような部分集合Sはどういった条件になるのでしょうか…?
0733132人目の素数さん
垢版 |
2020/11/22(日) 19:24:20.82ID:03o3b8Sl
>>731
>Gを類別
じゃなくて聞きたいのはG作用のある類別だろ?
0735132人目の素数さん
垢版 |
2020/11/22(日) 20:30:05.64ID:dR0FbWeM
類別というのが
関係g〜hをg^(-1)h∈Sで定義したとき同値関係になるということなら
確かにSが部分群であることが必要十分だな
0736132人目の素数さん
垢版 |
2020/11/22(日) 20:40:10.99ID:03o3b8Sl
G作用がある類別でeを含む類のイソトロピー群Hを考えたら?
0737132人目の素数さん
垢版 |
2020/11/22(日) 21:17:00.63ID:UjdrCFnt
類別と言うのが良くなかったかもしれません。
G={0,1,2,3,4,5}演算.はmod6の加法で、
S={2,3}とするとき、
G.S={{2,3}{3,4}{4,5}{5,0}{0,1}{1,2}}
となり、中の集合の要素に重複がありますが、

S={1,4}とするとき、
G.S={{1,4}{2,5}{3,0}}
と、Gの要素が被りなく分けられましたが、S={1,4}は部分群ではない

という感じのが例になります。
もちろん、
S={0,3}のときのSは部分群なので
G.S=G/S={{0,3}{1,4}{2,5}}
と類別されます。

今回はZ/6Zの例ですが、一般的にGが被りなく分けられるSの条件が気になりました。
0738132人目の素数さん
垢版 |
2020/11/22(日) 22:23:34.57ID:dR0FbWeM
G=∪giS(g0=e)でgiS∩gjS≠ΦならgiS=gjS
ということかな
それでも同じで
eがあるgiSに含まれるけど、そこが部分群になってることが示せて、結局これはその部分群による左類別になってる
0739132人目の素数さん
垢版 |
2020/11/22(日) 22:49:59.93ID:UjdrCFnt
>>738
まさしくその条件でした
つまり、S={1,4}は部分群ではないけど、0が含まれてるgiS=2.S={3,0}が部分群になってるので結局類別されるということでしたか…

この場合は群Gの部分集合Sによる類別と呼ぶのではなく、前の方がおっしゃるようにG作用のある類別などと呼ぶのが適切なのでしょうか
0740132人目の素数さん
垢版 |
2020/11/22(日) 23:06:26.79ID:03o3b8Sl
>>729
>>737
0741132人目の素数さん
垢版 |
2020/11/22(日) 23:47:56.73ID:0I7s1r1R
fをR^n上の滑らかな関数でf'(p)≠0とします。
このときpの近傍Uと、R^nの開集合Vと、微分同相g:V→Uを、
f(g(x_1,...,x_n))=x_nとなるようにとれることの証明を教えて下さい。
0742132人目の素数さん
垢版 |
2020/11/22(日) 23:50:01.98ID:dR0FbWeM
>>739
自分も今回聞かれて気づいたけど
G作用をちゃんと考えておかないと
ただG=S ∪ gS ∪ g'S ∪…(disjoint)
とするだけなら色々作れるね
例えばZ/4Z={0,3} ∪ 2{0,3}

もっというとGの部分群Hの右剰余系G=∪Hgi (g0=e) から代表higi∈Hgiを選んできて
S=∪{higi}とおけば類別G=∪hS(h∈Hを動く)
が作れる
0743132人目の素数さん
垢版 |
2020/11/23(月) 00:17:13.38ID:8MdyC1X9
>>741
R^n の座標の内 ∇f 方向に一致しない n-1 個を x_1〜x_{n-1} として
F : R^n → R^n を F = (x_1, 〜, x_{n-1}, f ) とすれば局所同相だから逆関数を
g = F^(-1) とすれば良い
0744132人目の素数さん
垢版 |
2020/11/23(月) 07:44:47.89ID:/QjQPOsE
>>743
ありがとうございます。
でもこれでf(g(x_1,...,x_n))=x_nとなる理由をもう少し詳しく教えてほしいです。
n=1ならわかりますが、、、
0746132人目の素数さん
垢版 |
2020/11/23(月) 15:03:58.43ID:NdcoW5qQ
>>743
>R^n の座標の内 ∇f 方向に一致しない n-1 個を x_1〜x_{n-1} として
各点の接空間の中に取るの?
大域化するにはどうする?
0751132人目の素数さん
垢版 |
2020/11/23(月) 21:20:33.24ID:6XH6V60V
「平面上の点(0, 2)を通る直線の族が満たす微分方程式を求めよ」みたいな問題がありますけど、
求めた微分方程式は何かに使えるんですか?
0753132人目の素数さん
垢版 |
2020/11/24(火) 17:35:17.83ID:tU3zWjqM
G_nを複素n次元グラスマン多様体,Eをその標準複素ベクトル束γ^nの全空間,
E0をEからゼロセクションにあたる部分を除いたものとし
f:E0→G_n-1,(X,v)→(Xの中のvの直交補空間)
がコホモロジーの同型を導くことの証明の中で
fの有限次元への制限
f_N:E0(γ^n(C^N))→G_n-1(C^N)
が次元が2(N-n)以下でコホモロジーの同型を導くことから
Nを無限に飛ばした帰納極限ではfが全ての次元でのコホモロジーの同型を導く
という議論がありますが,これはなぜ言えるのでしょうか?

帰納極限で一般に成り立つことではなく,f_Nがfの中のCW複体として低い次元のところを
全部含んでしまっているみたいな事を使っているのでしょうか
ご教示下さい
(ちなみに出典はMilnorの特性類の本のth14.5です)
0755132人目の素数さん
垢版 |
2020/11/24(火) 18:25:51.81ID:rx/P7w/j
「代数体」の定義として「有理数体の有限次拡大体」を採用した場合と「有理数体の有限次拡大体と同型な体」を採用した場合に違いはありますか?
0758132人目の素数さん
垢版 |
2020/11/24(火) 19:49:36.32ID:tU3zWjqM
>>754
そのことが一般に言えるんですか
ホモロジーがcolimをcolimに移すことは見るもののそっちは見たことがなくて
一般には言えないのかと思っていました
特異コホモロジーで書いてあってその結果が載っている本とかってないでしょうか
0759132人目の素数さん
垢版 |
2020/11/24(火) 22:00:11.68ID:z0JupO0u
>>758
幾何だと何に載ってるのかな?
オレは代数畑で代数圏の教科書で勉強したけど
carl faithのalgebra I
まず空間Xiに対して特異単体はTop(S^*,X)でS^*がコンパクトであることからこの関手をTop→Setとしてcolimと可換
Free関手S→ZSもcolimと可換だから合成したTop→Abも桶
termeiseにcolomと可換だから^*を走らせてTop→C(Ab)も桶
最後のC(Ab)→Abがちょい難しいけどココも桶
よって特異単体ホモロジー関手H_:Top→Abはcolimと可換
Ab(-,Z)を途中で挟んでH^:Top)→Ab^opはcolimをlimに持っていく
多分難しいのは最後の→のとこ
すなわち

定理
‥Xi3→Xi2→Xi1→Xi0→0
がコチェインの族である時
H_j(colim[i]Xi_) = colim[i] H_j(Xi_)

初等的には第0項が同型になる事を示して‥です
0760132人目の素数さん
垢版 |
2020/11/25(水) 00:31:50.53ID:Ze0T25Ab
>>759
ありがとうございます
ホモロジーの時と変わるのはAb(-,Z)あてて反変した後の最後のとこですよね
代数苦手なので読めそうなものをもうちょい探してみます
0761132人目の素数さん
垢版 |
2020/11/25(水) 01:12:00.72ID:KHzOo8Uf
>>749
なら元の疑問の答えではない
0762132人目の素数さん
垢版 |
2020/11/25(水) 01:22:31.75ID:KHzOo8Uf
>>754
一般に言える?
2;Z->ZつなげてcolimはZ[1/2]
dualも2:Z->Zだけどlimは0
0763132人目の素数さん
垢版 |
2020/11/25(水) 09:37:49.36ID:2qKqSe/3
>>262
cokernelはA→Bの図式のcolimではないよ
A→B

0
の図式のcolim片方が0のpushout
(-,X)で片方0のpullbackにうつってkernelになる
0764132人目の素数さん
垢版 |
2020/11/26(木) 17:17:27.90ID:WSnRNVhM
>>752
役に立ってないじゃないですかー
0766132人目の素数さん
垢版 |
2020/11/26(木) 19:06:37.58ID:gdFyrZjb
連立漸化式や積分方程式に解があるとか代数的に解けるとかの判断基準ってあるの?
0767132人目の素数さん
垢版 |
2020/11/26(木) 21:02:26.74ID:WSnRNVhM
「問題が解ける」ならまだしも「作れる」って余計なことしてるだけじゃないですかー
0768132人目の素数さん
垢版 |
2020/11/26(木) 21:13:38.64ID:z51YICjQ
ここにいる方々って
数学科卒のバックグラウンド持ってる人達ばっかなんですか?
他の分野から来て数学を研究又は学んでる人いますか?
0771132人目の素数さん
垢版 |
2020/11/27(金) 02:18:36.41ID:HgDXikZH
>>770
ゴミ文系卒した所で何の意味にもならんやろ、せいぜい「卒業した」って言う箔だけ
0773132人目の素数さん
垢版 |
2020/11/27(金) 03:56:20.51ID:zeE2fsU+
外積代数って微分形式を定義するのに使う以外に用途ありますか?
0774132人目の素数さん
垢版 |
2020/11/27(金) 09:25:30.62ID:rY5rhTUo
微分形式のアナロジーだけど、可換環上の加群に対して外積代数を定義できるので、相対微分(ケーラー微分)の理論などに用いられる
0775132人目の素数さん
垢版 |
2020/11/27(金) 11:40:10.94ID:sDi+TIaR
異なるレートパラメーターを持つ異なる指数分布からn個のサンプルが与えられた場合、特定のサンプルが最小である確率は、
そのレートパラメーターをすべてのレートパラメーターの合計で割ったものに等しくなります。

これについて説明があるサイトをご存知の方いませんか?
0776132人目の素数さん
垢版 |
2020/11/27(金) 18:25:45.88ID:R3V2Lswu
ちょっと違うけど
http://www.math.wm.edu/~leemis/chart/UDR/PDFs/ExponentialM.pdf
0777132人目の素数さん
垢版 |
2020/11/27(金) 18:39:59.41ID:R3V2Lswu
これを参考にしたら、あなたの求めたい論証がすぐに得られるはず
0778132人目の素数さん
垢版 |
2020/11/27(金) 21:37:51.28ID:zeE2fsU+
>>774
外積代数の動機はほぼ広い意味での微分形式ですか?
0779132人目の素数さん
垢版 |
2020/11/28(土) 02:11:44.10ID:L4r9q1cg
>>776-777
これは大変にありがたい
考えてみます

ところで>>775では「最小」になっていますが、「最大」側も同じことが言えるでしょうかね?
言えるのだろうと思っていて、「極値」について対称性があると思っているのですが
0781132人目の素数さん
垢版 |
2020/11/28(土) 06:24:18.85ID:Adf5Z+e6
>>778
動機??
じゃ
多項式環の動機って何?
0782132人目の素数さん
垢版 |
2020/11/28(土) 06:46:48.56ID:kv5+y3o0
>>781
そのレベルの人はいらないです
0784132人目の素数さん
垢版 |
2020/11/28(土) 07:14:36.38ID:Adf5Z+e6
>>782
整数環の動機も教えて
0785132人目の素数さん
垢版 |
2020/11/28(土) 12:48:47.91ID:iDO1vFik
>>778
外積代数はグラスマン多様体を利用したもの
グラスマンのアイデアをたどれ
0787132人目の素数さん
垢版 |
2020/11/28(土) 14:41:06.23ID:Xt8UUb5s
動機がないとわからないのは少なくともその分野は向いてないんだろう、他の分野をやった方がいいと思う
もとより隣接分野の知識やアイデアを仕入れてるだけならまあ頑張れ
0788132人目の素数さん
垢版 |
2020/11/28(土) 19:20:58.27ID:kv5+y3o0
>>785
調べてみます

>>786
使う以前に何に使えるのか分からないと・・・
0789132人目の素数さん
垢版 |
2020/11/28(土) 21:57:31.12ID:Adf5Z+e6
>>788
>使う以前に何に使えるのか分からないと・・・
じゃあ
悩まなくてイイよ
0790132人目の素数さん
垢版 |
2020/11/28(土) 22:26:21.14ID:kv5+y3o0
>>789
多項式環が分からないレベルの人はいらないです...
0793132人目の素数さん
垢版 |
2020/11/28(土) 22:51:01.02ID:Adf5Z+e6
>>790
では君は多項式環の動機が分かるんだね
教えてくだされ
整数環の動機もね
0794132人目の素数さん
垢版 |
2020/11/28(土) 22:54:07.57ID:Adf5Z+e6
また別のが来たか
0795132人目の素数さん
垢版 |
2020/11/28(土) 23:36:15.39ID:kv5+y3o0
>>793
>>783
0796132人目の素数さん
垢版 |
2020/11/29(日) 01:24:48.40ID:W9oDpRxp
>>784
自然数から整数への拡張は動機になりうる要素が多すぎるけど、例えば減法の全域的な定義
0797132人目の素数さん
垢版 |
2020/11/29(日) 01:30:26.12ID:F0QEEzEc
前もって勉強するなんて時間がいくらあっても何処にも行けない
必要が出てから勉強すりゃいい
0798132人目の素数さん
垢版 |
2020/11/29(日) 02:01:54.77ID:nLrkJ/Ox
>>795
噴飯
整数環は?
0799132人目の素数さん
垢版 |
2020/11/29(日) 02:04:22.53ID:nLrkJ/Ox
>>796
それは自然数から整数への拡張の動機と言って良いかもしれないが
整数環の動機なの?それ
0800132人目の素数さん
垢版 |
2020/11/29(日) 03:04:56.61ID:aIMvO3No
クレクレ君うぜえ
0802132人目の素数さん
垢版 |
2020/11/29(日) 12:40:39.09ID:MqOAtv8+
>>776-777
ゆっくり読んでみて理解できましたが、やはり最大のほうはこうならないようですね
なぜ最大のほうにこだわるかというと、最大のほうを説明なしに同様に扱っていると思われる論文があったことと
直観的には最大のほうだろうなぁと思ってたからです
例えば、大きなレートパラメーターを持つ確率変数ほどその確率変数が最大値をとるサンプルが多くなりそうな気がします
その逆に、大きなレートパラメーターを持つ確率変数ほどその確率変数が最小値をとるサンプルが多くなるというのは意外です
モンテカルロシミュレーションで再確認してみようと思います。

ありがとうございました
0804132人目の素数さん
垢版 |
2020/11/29(日) 12:53:55.71ID:eI6jE607
>>785

>>787

>>791
グラスマンなんて生前はオカケツやグロタンより変なこと言ってる扱いされてたからなあ。
グロタンのダルマとか六つの操作とかモチーフのモチベーション並みにアレだし。
0805132人目の素数さん
垢版 |
2020/11/29(日) 13:26:52.90ID:rsupXatX
>>802
パラメーターが大きいほど半減期が短いでしょ?当たり前のことのように思えるけど
0806132人目の素数さん
垢版 |
2020/11/29(日) 15:09:43.74ID:MqOAtv8+
>>805
そうですね。パラメーターが大きいと小さな確率変数が選ばれる可能性が大きくなるから最小として
サンプルされる可能性が高くなるということですか

私が直観的に意外に感じたのは、この問題の元ネタとして、重み付きサンプリングに使うという話が
あって、重みをレートパラメーターにマップすれば正規化された重み付きのサンプリングができるという話
だったからです
重みの大きさと確率変数の大きさが区別できていなくて、重みが大きいほど確率変数も大きいかな?
と思いましたが、勘違いでしたね。
Proportional selectionやroulette wheel selectionという方法では、重みに比例した長さの直線を
考えて、振った乱数がどの部分に属するかを探索して重み付きサンプリングするのですが、そのときの
考え方を引きずっていました。

指数分布は確率変数の単調減少関数だから重みが大きい(パラメーターが大きい)ほど確率変数の
大きさは小さめになるんですね正規化する部分に指数関数の性質が活かされているのですね

直観的にもスッキリしました。こうなると論文は間違っていたのか、最小を選ぶところを最大と書いてしま
ったケアレスミスがあったのにレフェリーが気がつかずに通ってしまったんでしょうね
0807132人目の素数さん
垢版 |
2020/11/29(日) 15:47:17.81ID:f9WF+OZR
物事、状態、環境、状況が整った状態や乱れた状態を数式で表す概念ってあるんですか?

例えば、整理整頓されたホコリ1つ無い部屋と嵐と洪水が過ぎ去った部屋ではモノの整理整頓状態が真逆だし、
それらの状態の間にも整理整頓状態は連続的にあるわけで、それらの連続的な状態変化は何らかの数値・数式によって表現されても何もおかしくないと思うんだが。
0808132人目の素数さん
垢版 |
2020/11/29(日) 15:52:20.29ID:f9WF+OZR
で、しかも、一般的に、物事・状態・状況は整えることの方が難しくて、初期状態としては乱れていることの方が一般的で、
その乱れている状況等に対して適切なエネルギーを作用させることによって、その状況等が整った状況に移行する。
で、それでいて、整った状況等というものは、安定・平衡状態にはあらず、些細な不的確な外的作用によって容易に乱れている状況等に移行する。
そういう風に考えることが出来る。

エントロピーというワードを聞いたことがあるが、そっちけいか?


部屋は整理整頓するのはしんどいのに、散らかすのは簡単
糸はほどくのはしんどいのに、絡まるのは一瞬
ドミノは並べるのはしんどいのに、倒れるのは一瞬

こういう概念って絶対数学でカテゴライズできるよな
0809132人目の素数さん
垢版 |
2020/11/29(日) 16:07:39.40ID:N7kSECVq
https://imgur.com/xdlOICF.jpg

この解答は間違っていると数学者に言われました.
というか,意味不明だとさえ言われました.
どこが間違っているのか,また正しい解答をお願いします.
0810132人目の素数さん
垢版 |
2020/11/29(日) 16:14:54.26ID:yeCmdoxf
>>809
示せと言われてるのが陰関数定理で実質「陰関数定理を証明せよ」なのに陰関数定理使ってるからじゃね?
0811132人目の素数さん
垢版 |
2020/11/29(日) 16:47:35.36ID:N7kSECVq
>>810
ありがとうございました.
この問題ですが,陰関数定理の章の章末の演習問題です.ですので,陰関数定理については使うことができます.
0812132人目の素数さん
垢版 |
2020/11/29(日) 17:15:44.80ID:aIMvO3No
本人に聞け
お前の教師だろ
0813132人目の素数さん
垢版 |
2020/11/29(日) 19:27:27.96ID:rsupXatX
>>806
その論文を教えて下さい
あなたが勘違いしてるか、それとも本当に論文が間違ってるか調べたくなりました
0814132人目の素数さん
垢版 |
2020/11/29(日) 19:36:20.08ID:nLrkJ/Ox
>>808
さあてどうかしら
エントロピーは単にあり得る状態の数の逆数の2を底とした対数
あり得る状態が2倍になればエントロピーは1減るというだけだけど
0817132人目の素数さん
垢版 |
2020/11/29(日) 20:55:06.50ID:MqOAtv8+
>>813
元の論文は
https://search.ieice.org/bin/summary.php?id=e85-a_6_1229
なんですが、微妙に変わってます
(8)式なんですが、元の論文では、
di=max{dj,j=1,2,...,N}
を満たすargument iを得る、だったのにpdfではdjの和になってますね
これもケアレスミスなのかな?それとも和だとmaxで正しくなる?
0818132人目の素数さん
垢版 |
2020/11/30(月) 10:27:24.13ID:R/fj8I5+
なんかこの論文、いろいろと怪しいです。単に記述が正確ではないだけですけど
第三者から聞いたことを著者が自分で理解せずに書いてるような感じ
0819132人目の素数さん
垢版 |
2020/11/30(月) 15:35:13.77ID:/l4XlfON
>>816
> >>813
> http://www.wseas.us/e-library/conferences/corfu2004/papers/488-428.pdf
> にダウンロード可能で同内容の論文がありました
wseasって、いわゆるハゲタカ出版社じゃないですか?
既存の論文を出版社が勝手に自分の所で出版したように装っているとかもありうるのかもしれませんが。

ついでに、
>https://search.ieice.org/bin/summary.php?id=e85-a_6_1229
は、電子情報通信学会の論文誌ということで、数学の論文ではないですね。
査読がついているかどうか知りませんが、査読ついていても、その分野の専門家の数学者の査読がついている可能性は低い気がします。
0820132人目の素数さん
垢版 |
2020/11/30(月) 17:31:07.78ID:R/fj8I5+
>>819
前者についてはあまり考えなくてもよいと思います。異なる論文です。
後者について、数学じゃありません。この中で重み付き乱択を指数分布乱数を利用して行っていると思われる
記述があり、そこでminではなくmaxを選んでいた次第です。
査読付きかどうかも知りませんが、他の部分も含めて不正確さを含む論文と思います。
多分記述がおかしいだけで、結果は間違いではない(実際はminを使ってる)と判断しています。

自分的には何をやっているのか全貌がはっきりしたので満足です。ありがとうございました。
0821132人目の素数さん
垢版 |
2020/11/30(月) 18:26:15.90ID:W3ZtItX/
cos^2(Arctanx)の積分結果が納得いきません
Arctanx=tと置換すると
x=tantになって
dx=(1/cos^2(x))dt
これとcos^2(t)をかけると
1になって、結果はt=Arctanxになってしまう
Arctanxの微分は1/1+x^2のはずなのに
これはArctanxの微分はcos^2(Arctanx)ともなってしまうことを示している
計算機でもやってみたんですが、こうなってしまう
どうしてか教えてください
0822132人目の素数さん
垢版 |
2020/11/30(月) 18:30:32.90ID:W3ZtItX/
ごめん
積分定数忘れてた
0824132人目の素数さん
垢版 |
2020/11/30(月) 19:27:50.06ID:W3ZtItX/
>>823
確かにそうですね
ありがとうございます
0825132人目の素数さん
垢版 |
2020/12/01(火) 16:57:44.42ID:g7sJKnef
Knotsってジャンル分けするなら、幾何 - 位相幾何(Topology) - 結び目 ?
もっと適切な表現あったらお願いします
0827132人目の素数さん
垢版 |
2020/12/01(火) 17:00:33.77ID:g7sJKnef
>>825,826
というよりもむしろ、Knotsに限らず、キーワードから、そのキーワードの属するジャンルをツリー的に特定したいんだが
例えばジョルダン標準形なら、代数 - 線形代数 - ジョルダン標準形 みたいな感じで
0828132人目の素数さん
垢版 |
2020/12/01(火) 17:45:02.90ID:g7sJKnef
っていうかそもそも、分野名やジャンル分けって学会か協会的に正式にされてるものなん?
0830132人目の素数さん
垢版 |
2020/12/01(火) 18:43:09.72ID:upzTgLnk
たった2時間で催促、しかも其の催促の仕方もゴロ撒き口調…、お前はヤクザの実子か?
どう育ったらそんな、世間を舐めた生き方ができるんだ?まさか組の若い衆から「脅しの為」と騙して借りて来て
こっちに向けて銃なんかブッ放したりなんかしねぇよなぁ?ただでさえ俺は
実の親の両方からそれぞれ別件で包丁傷を付けられた過去があってビビり症なんだから勘弁してくれよなぁ?
0831132人目の素数さん
垢版 |
2020/12/01(火) 18:57:56.73ID:WR3Qpg9R
>>825
結び目理論で単項目でいいよ
数論と同様に研究のために様々な手法が使われる分野だ
解析数論、代数的整数論、数論幾何とあるようなもの
0835132人目の素数さん
垢版 |
2020/12/01(火) 20:14:24.71ID:g7sJKnef
>>833
参考になります。
落としました。
ファイルの整理はこれを見ながらすればおkだな
0836132人目の素数さん
垢版 |
2020/12/01(火) 21:09:00.22ID:V2+1FrUl
綺麗なノートを書くのが自己目的化して勉強ができない奴と同類だな
0838132人目の素数さん
垢版 |
2020/12/01(火) 22:16:37.45ID:g7sJKnef
割と、ゴミ捨て場に本が束になって捨てられてるの見たらジロジロ見るぐらいだしな
拾ったこともあるし
0839132人目の素数さん
垢版 |
2020/12/01(火) 23:05:46.00ID:l9VJN3G5
> 実の親の両方からそれぞれ別件で包丁傷を付けられた過去
詳しく教えて
0841132人目の素数さん
垢版 |
2020/12/02(水) 06:28:28.56ID:pV8MmGTK
>>827
>キーワードから、そのキーワードの属するジャンルをツリー的に特定したい
自分でやりなよ

どうせなら「キーワード」でなく「主要定理」を分類してほしいかな
でも、数学がわからない素人には無理かw
0843132人目の素数さん
垢版 |
2020/12/02(水) 15:58:31.02ID:wyvN+u78
>>841
んじゃ、主要定理に限定せずキーワードで分類したいと思う奴が素人って根拠は?
0844132人目の素数さん
垢版 |
2020/12/02(水) 16:10:45.04ID:cYLKc95T
高校質問スレが機能してなさそうなのでこっちで質問します
z=(-3+4icosθ)/(5+4sinθ)で0≦θ≦πってどんなグラフになる?
mathematicaで描いたら中心が(-5/3)の円になるらしいが過程がさっぱりわからない
0845132人目の素数さん
垢版 |
2020/12/02(水) 16:38:53.72ID:R2BBbmP0
なんで指数分布と関連しているかイメージがつかめてきました
0846132人目の素数さん
垢版 |
2020/12/02(水) 16:46:24.37ID:jidSh+Ez
>>844
x:=re(z)=-3/(4 sin(θ) + 5), y:=im(z)=(4 cos(θ))/(4 sin(θ) + 5)を
(x+5/3)^2+y^2に代入すると16/9になるから, zは(-5/3, 0)中心の半径4/3の円の一部
0848132人目の素数さん
垢版 |
2020/12/02(水) 17:26:45.13ID:P6EOSsA1
軌跡問題の典型的な誤答だな
0849132人目の素数さん
垢版 |
2020/12/02(水) 19:19:28.71ID:+4Zl3d+H
間違ってるか?
0850132人目の素数さん
垢版 |
2020/12/02(水) 19:34:37.71ID:wyvN+u78
>>841
ありゃりゃ、答えられないアホか

なんだろ、論文や書籍を大量に持ってて整理するのに困ってる話をしたらこうやって妬みって言うか訳の分からんアホが絡んでくるのってなんなんだろ
まぁどこにでもこういう頭のおかしい狂ったゴミって居るものだけど、本を大量に持ってるって言う話題がキチガイを反応させるトリガーになってるのがなんか意外ww
0851132人目の素数さん
垢版 |
2020/12/02(水) 19:39:16.37ID:jbyVc/xj
「の一部」で逃げを打っているな
数学の解答としては駄目だ
0852132人目の素数さん
垢版 |
2020/12/02(水) 22:52:43.78ID:q9FdBayQ
>>827
アホなこと考えるもんだな
0853132人目の素数さん
垢版 |
2020/12/02(水) 22:55:55.76ID:q9FdBayQ
>>850
>なんだろ、論文や書籍を大量に持ってて整理するのに困ってる話をしたらこうやって妬みって言うか訳の分からんアホが絡んでくるのってなんなんだろ
なんか
拗れてる人そう
0856132人目の素数さん
垢版 |
2020/12/03(木) 05:57:59.65ID:4fdNxQnp
>>853
俺はソイツの親がゴネ得常習者で継承してるんだと思う、つまり白い目で見られ尚且つ見て見ぬ振りをされる人。
0857132人目の素数さん
垢版 |
2020/12/03(木) 10:06:41.97ID:TJHNN9pg
集合論に関する質問です。
加算濃度: N₀, 連続体濃度: N = 2^N₀ とします。
無限濃度: m について m > N₀ ならば m^N₀ = m が言えるでしょうか?
N^N₀ = (2^N₀)^N₀ = 2^(N₀N₀) = 2^N₀ = N つまり m=N の時は正しいと分かります。
一般的な m についてはどうなのか。証明または反例があれば教えてください。
0858132人目の素数さん
垢版 |
2020/12/03(木) 13:24:39.46ID:/SAGqwid
少なくとも連続体仮説がないと可算と連続体の中間濃度Kに対してN0<K<N=2^N0≦K^N0となってしまうような
0859132人目の素数さん
垢版 |
2020/12/03(木) 13:48:47.27ID:CuFqTKDW
m ≧ 2^N₀ なら証明できそうだが
m = ℵ₁ だと連続体仮説と同様に証明できない事だな
0860132人目の素数さん
垢版 |
2020/12/03(木) 14:44:27.51ID:iFUz5m5l
>>857
の質問は
N(1) = 2^N(0) (連続体仮説が正しい)のとき任意のxに対してN(x+1) = 2^N(x)か?
( ただし基数cに対して2^cは#(pow(c)) )
でしょ?
x∈ωなら正しいとは思うけど
0863132人目の素数さん
垢版 |
2020/12/03(木) 21:38:37.39ID:TJHNN9pg
>>858, >>859
ありがとうございます。 なるほど、連続体仮説ですか...
こうなってくると松坂や赤摂也の本に載ってないのも当然ですね。
0864132人目の素数さん
垢版 |
2020/12/04(金) 12:13:16.19ID:PGSfJ89l
テイラー展開にn階微分が出てくる理由を教えてください
1階微分なら、接線の傾きなのでわかるのですが...
0865132人目の素数さん
垢版 |
2020/12/04(金) 12:27:24.73ID:KdT/xkVy
大学レベルじゃないな
0866132人目の素数さん
垢版 |
2020/12/04(金) 12:34:18.78ID:D0b51m9c
溝畑の数学解析って古風な感じに見えるけどいい本なの?
0868132人目の素数さん
垢版 |
2020/12/04(金) 20:36:48.53ID:en3er9CF
剰余項が微分の形だとしても、平均値の定理を繰り返し使う(高次導関数)だけだからn回微分が出てくるのは当たり前としか
0869132人目の素数さん
垢版 |
2020/12/04(金) 21:40:50.69ID:KdT/xkVy
1階微分だと1次式しか表現できないじゃないかぁ
0870132人目の素数さん
垢版 |
2020/12/05(土) 06:38:26.64ID:HSh14pIV
>>867
積分型に平均値の定理使えば微分型になるわ
0871132人目の素数さん
垢版 |
2020/12/05(土) 14:40:15.66ID:k91gn20I
斎藤毅の集合と位相(第4刷)のツォルンの補題の証明に使われてる補題7.3.4(p187)に関して質問です
{\mathcal R}_{F(T)}が{\mathcal A}の定義の(iii)を満たすことが上手く示せないのでもし分かる方が居たら教えて欲しいです
0873132人目の素数さん
垢版 |
2020/12/05(土) 16:19:05.65ID:k91gn20I
>>872
うおーピンポイントな解答ありがとうございます
しっかり理解出来ました
よくこんなにピッタリなサイトご存じでしたね
検索力に感服しました
0874132人目の素数さん
垢版 |
2020/12/06(日) 10:19:55.62ID:SE6q+SwO
DをR^2平面の連結集合とします。またE_n(n=1,2,3,...)を連結な有界閉集合の列(ただしE_nはDの部分集合)でE_1⊂E_2⊂E_3⊂...を満たしているとします。

このとき以下の2条件が同値になることは証明できるでしょうか。
(1)全てのE_nの和集合がDに等しい:∪_(n=1)^∞ E_n=D
(2)任意の連結な有界閉集合G(G⊂E)に対して E_n⊃Gとなる番号nが存在する

1点集合は閉集合なので(2)から(1)が導かれることは明らかなのですが、逆が導けるのかが気になっています。不可能な場合反例をいただけると助かります。
広義重積分の定義や定理で現れている条件なのですが、よろしくお願いします。
0875132人目の素数さん
垢版 |
2020/12/06(日) 10:22:46.99ID:SE6q+SwO
>>874
(2)の条件は(G⊂E)ではなく(G⊂D)でした。
書き間違いすみません。
0876132人目の素数さん
垢版 |
2020/12/06(日) 10:29:38.86ID:cT9Fbn3B
>>874
r≦1,0≦θ≦2π-1/n
0877132人目の素数さん
垢版 |
2020/12/06(日) 10:31:57.37ID:cT9Fbn3B
>>876
>r≦1,0≦θ≦2π-1/n
r≦1,1/n≦θ≦2π
0878132人目の素数さん
垢版 |
2020/12/06(日) 12:33:27.59ID:SE6q+SwO
>>876
>>877

反例どうもありがとうございます。
わかりました。
0880132人目の素数さん
垢版 |
2020/12/08(火) 07:55:31.07ID:0Ve6whCt
スレの目的とはちょっと違うかも知れないけど
四色問題ってクラスPに分類されるの?
なんか結局総当りに近いことやって解かれたのでNPっぽい感じがするけど…
0881132人目の素数さん
垢版 |
2020/12/08(火) 10:20:31.09ID:LNakKQfU
そもそもpとかnpとかいうのは帰納的関数に関する用語
真か疑か問う問題にpもnpもない
まず“帰納的関数がp”とは何意味してるかの勉強からしないと何も始らん
0882132人目の素数さん
垢版 |
2020/12/08(火) 12:36:52.57ID:XBd/xw/G
具体的に配色を求める問題ならP/NPに意味あるだろ
どちらでもないような気がするが
0883132人目の素数さん
垢版 |
2020/12/08(火) 12:58:18.65ID:4gBCy8UJ
> 真か偽か問う問題にpもnpもない
???
P も NP も、判定問題(決定問題) に関する話だから、むしろ真逆で、真か偽か問う問題しかないんじゃないのか

・素数判定問題は はクラスPに属する
・ハミルトン閉路問題 はクラスNPに属する
・合成数判定問題 はクラスNPに属する
0884132人目の素数さん
垢版 |
2020/12/08(火) 13:13:19.40ID:gbWiWLFJ
>>883
それは四色問題と言われてる問題を別の意味で捉えてる
元々数学の世界で4色問題といえば
「あらゆる閉面グラフにいわゆる“4色塗り分け”か”できるか?”」

そもそもほんとに“帰納的関数”の意味がわかってる奴が>>881の指摘を受けて意味が通じないわけがない
0885132人目の素数さん
垢版 |
2020/12/08(火) 14:28:48.59ID:Kc5O31eS
与えられたグラフが3色塗り分け可能かどうかの判定はNP完全であることが知られているので、全く見当外れの質問ではない
0886132人目の素数さん
垢版 |
2020/12/08(火) 14:56:52.65ID:gbWiWLFJ
>>885
まぁしかし塗り分けできるかなら常に可能を返せばいいだけだからクラスNだわな
しかも4色塗り分けを具体的に見つける関数と読み替えても、一番最初に発見されたHakenと誰かの方法ですでにクラスNやろ
0887132人目の素数さん
垢版 |
2020/12/08(火) 18:20:35.33ID:2i888ihm
平面グラフという条件を外すと4色塗り分け可能か否かの判定はNP完全
0888132人目の素数さん
垢版 |
2020/12/08(火) 18:47:35.28ID:4gBCy8UJ
>>884
4色問題に関して、その指摘を受けて意味が通じないかどうかは置いておいて、

「真か偽か問う問題にpもnpもない 」
この書き方は完全に間違いでしょ
0890132人目の素数さん
垢版 |
2020/12/08(火) 19:08:08.14ID:tMd//NqD
複素対数関数の
Log z = ln |z| +Arg z
の導出の仕方を説明してくれ
0891132人目の素数さん
垢版 |
2020/12/08(火) 19:21:09.72ID:gbWiWLFJ
log z = ∫[t:1〜z]dt/t
から |arg z| + |arg w| <π の時
log zw
= ∫[t:1〜zw]dt/t
= ∫[t:1〜z]dt/t + ∫[t:z〜zw]dt/t
= ∫[t:1〜z]dt/t + ∫[u:1〜w]du/u (t =zu)
= log z + log w
z = |z| exp iθ として(θ=arg z)
log z = log |z| + log exp(iθ)
ここで一般に
log exp(w)
= ∫[t:1〜exp(w)]dt/t
= ∫[u:0〜w] w exp(wu)/exp(wu)du
=w
0892132人目の素数さん
垢版 |
2020/12/08(火) 19:35:22.61ID:tMd//NqD
[t:1~z]とかの意味を教えてくれ
すまん
0893132人目の素数さん
垢版 |
2020/12/08(火) 19:36:52.45ID:tMd//NqD
今分かった
定積分の区間?
0894132人目の素数さん
垢版 |
2020/12/08(火) 19:39:15.45ID:tMd//NqD
それはそうと
教えてくれてありがとうございます
0896132人目の素数さん
垢版 |
2020/12/08(火) 19:46:08.67ID:tMd//NqD
連投すまん
最後の積分を説明してくれ
0897132人目の素数さん
垢版 |
2020/12/08(火) 20:01:23.23ID:gbWiWLFJ
すまん、一ヶ所間違ってる
t = exp(wu)と置換
t:1〜exp(w) ⇒ u:0〜1←ココ
dt ⇒ w exp(wu)du

log exp(w)
= ∫[t:1〜exp(w)]dt/t
= ∫[u:0〜1] w exp(wu)/exp(wu)du
= w ∫[u:0〜1]du
= w
0898132人目の素数さん
垢版 |
2020/12/08(火) 20:15:02.60ID:tMd//NqD
>>897
ありがとうございます
0900132人目の素数さん
垢版 |
2020/12/08(火) 20:37:40.88ID:Kc5O31eS
>>889
君が間違っている
それだけのこと
0901132人目の素数さん
垢版 |
2020/12/08(火) 21:29:58.18ID:tMd//NqD
>>891
|argz|+|argw|<π
これはどういう意味なん?
解決した後なのにすまない
0902132人目の素数さん
垢版 |
2020/12/09(水) 05:53:20.23ID:W1cyDP22
u(x)=∫_[0,x] v(w(s)) ds において
vの台がコンパクトでw'が有界ならuの台もコンパクトになるらしいのですが
どうやってわかりますかね?
0903132人目の素数さん
垢版 |
2020/12/09(水) 06:27:05.38ID:75nNMcbm
>>901
そこはlogの定義域をどこに取るかによります
私のレスの想定ではlogの定義域としてC\(-∞,0)を取っています
多分一番多い取り方
このときarg(z)の値域は|arg(z)| < πになります
例えばz=w=-1+iのときlogz = logw = (1/2)log2 + 3/4πiによりlogz+logw=log2+3/2πですが一方でzw=-2iによりlogzw=log-π/2iとなって
logz+logw-logzw = 2πi
となりlogz+logwとlogzwはずれます
コレは>>891でlogzwを計算するための線積分1→zwを1→z+z→zwと分けたときの積分路1→-2iが原点を正の向きに回るのに対して直接分けずに計算するときには負の向きに回ることに依ります
このような現象が起こらないようにするにはlogの定義域が単連結になるように0と∞を結ぶなんらかのpathを選んで抜いておかないとダメです
それを多くの場合(-∞,0]に選ぶことが多い
>>891では z = |z| × exp iθ においてarg |z| = 0なので大丈夫
逆に言うとlogの定義域を定めるときの“切り込み線”が同じsrgumentの線を跨いでるとズレます
なのでそのような場合にはちょっと証明もめんどくさくなります
0905132人目の素数さん
垢版 |
2020/12/09(水) 21:07:06.78ID:LZzZAaaW
>>890
>Log z
主値?
0907132人目の素数さん
垢版 |
2020/12/11(金) 18:25:24.61ID:Qns/0iSi
αが無理数の時、
 複素数列: zₙ = e^{i2π nα} (n=1, 2, 3, ... )
は単位円周を稠密に覆う事を示してください。
昔、何か数論の本で見た気がするんですが証明は忘れてしまいました。
0909132人目の素数さん
垢版 |
2020/12/11(金) 20:03:37.39ID:UMEKJndu
>>907
背理法でやると値の間隔に下限 δ > 0 が存在する
→ δ ≦ | z_n - z_m | < δ + δ/2 となる z_n, z_m が存在する
→ | z_n - z_m | = δ となる → 2π は δ の整数倍 → α は有理数
0911132人目の素数さん
垢版 |
2020/12/11(金) 21:17:21.08ID:LfSsSCOw
一様は難しいが稠密は簡単という話ではないの?
0912132人目の素数さん
垢版 |
2020/12/11(金) 23:18:09.76ID:4lWnVr7q
見た感じ、
{ nαの小数部分 | n:1,2,3,。。。} は[0,1)の稠密な部分空間な感じがする
で、これはZと同相かな?
0914132人目の素数さん
垢版 |
2020/12/12(土) 01:30:49.09ID:jxtD8CLr
>>912

> 見た感じ、
> { nαの小数部分 | n:1,2,3,。。。} は[0,1)の稠密な部分空間な感じがする

こいつは孤立点含まないけど

> で、これはZと同相かな?

は全部同じ孤立点
0915907
垢版 |
2020/12/12(土) 02:00:17.16ID:b8b8mId+
>>910 ありがとうございます。 キーワードを頼りにググって一様分布定理の証明も理解できましたが、
自分が思ってたのは クロネッカーの稠密定理のほうでした。
鳩の巣原理を使って云々するのをブルーバックスの数論本のどれかで読んだ覚えがあります。
0916132人目の素数さん
垢版 |
2020/12/12(土) 17:54:20.31ID:V0eci866
「数学の本」スレに書き込みしたら、スレ違いと言われてしまったので、こちらでお聞きします。

河野俊丈「場の理論とトポロジー」(岩波書店)を読んでいます。
固有名詞の読み方(発音?)がわかりません。

第1章 1節
Belavin-Polyakov-Zamolodchikov ---> ベラビン-ポリヤコフ-ザモロチコフ ?
Wess-Zumino-Witten ---> ウェス-ズミノ-ウィッテン ?
Virasoro代数 ---> ヴィラソロ代数 ?
Knizhnik-Zamolodchikov接続 ---> ニズニック-ザモロチコフ接続 ?
Maurer-Cartan形式 ---> モーラー・カルタン形式? モレ・カルタン形式?

これであってますか?
0917132人目の素数さん
垢版 |
2020/12/12(土) 23:10:24.40ID:q29CG4NN
ググると
Belavin ベラーヴィン
Zamolodchikov ザモロドチコフ
Knizhnik クニーズニク
Zamolodchikov ザモロドチコフ
Maurer マウレル, マウラー, マウラ, マオラー, マーラー, モウラー, モーラー, モラー
0919132人目の素数さん
垢版 |
2020/12/13(日) 08:13:54.12ID:KdLrYZfk
   条件αにより
   A=B=C=D=E
   が分かる
っていう記述は数学書では当たり前のようにでてくるけど、俺はこういう記述は分かり易さの観点から絶対ダメと指摘する
なぜなら条件αがどこで効いてるかが不明瞭だから
「そんなこと議論の流れを見れば分かるだろ」っていうのは的外れ
なぜならどこで効いてるかを一々検証する労力を読者に割かそうとすること自体が分かりにくさの一因になり、読解を阻害するから
「そんな影響はごく少量だろ」っていうのは的外れ
なぜなら微細な影響が積もる事こそちりも積もれば〜の理屈で読解の阻害になるから。だから小さな芽から摘んでおけということ
0920132人目の素数さん
垢版 |
2020/12/13(日) 11:21:17.80ID:jqyzQbt4
1ビット脳ならそうかもね
0921132人目の素数さん
垢版 |
2020/12/13(日) 12:28:49.71ID:3B7uuO7V
>>919
分かるのは読者の義務だけど?
0922132人目の素数さん
垢版 |
2020/12/13(日) 13:09:08.30ID:OHI65L2g
一眼でわかることを一々説明されたら、かえって分かりにくいわ
自分のレベルに合わん本を読むのが間違い
0923132人目の素数さん
垢版 |
2020/12/13(日) 13:40:01.28ID:jS6UuC4J
広中平祐(フィールズ賞受賞者)もブルーバックスから出てる本で「分からないことに躓いたらすぐ本で調べようとしてしまう」と嘆いていた
本にギャップを見つけたときに調べるのではなく考えようとするのは、広中平祐(フィールズ賞受賞者)の性格とは真逆
0924132人目の素数さん
垢版 |
2020/12/13(日) 14:21:26.40ID:8Cflf/0h
複素解析学からです。
正則関数の導関数は常に連続ではないのですか?今読んでいる本が、コーシーの積分定理で導関数の連続性を態々仮定したり、あるいは導関数の連続性を仮定しない場合の定理を言ったりしているのですが、正則関数は無限回微分可能だから導関数も連続ではないのですか?
0926132人目の素数さん
垢版 |
2020/12/13(日) 17:54:30.04ID:OHI65L2g
正則関数のほとんどの性質はコーシーの積分定理から証明されることだろ
積分定理の証明に使えるわけがない
0927132人目の素数さん
垢版 |
2020/12/14(月) 01:05:07.07ID:xX5Yji0+
>>924
それ自明にするっての
lim sinθ/θ=1の証明にロピタル使うようなもんだよ
0929132人目の素数さん
垢版 |
2020/12/14(月) 20:09:41.40ID:zVmRR4my
E. T. WHITTAKER AND G. N. WATSON, A COURSE OF MODERN ANALYSIS より
https://cdlbb.github.io/WandW/CMA02-2-SeriesMN.html#thecomparisontheorem

2.34 The Comparison Theorem の Example2
 zₙ = e^{in} (n=1,2,3,...) とした時の
 級数: 1/(1² (z-zᵢ)) + 1/(2² (z-z₂)) + 1/(3² (z-z₃)) + 1/(4² (z-z₄)) + ...
 (定義域は z≠zₙ の複素平面)
について、 |z|=1 なら (おそらく)発散する事を示してください。 (本文では |z|≠1 における収束のみを考察)

「簡単のため (For simplicity...) |z| = 1 での収束性は議論しない ... ... 収束領域が単位円周で二つに分断されてるのが興味深い」
とあるので、著者は |z| = 1での発散を知っていたのではないかと思います。

稠密定理(>>907 で質問しました)より |z-zₙ| はいくらでも小さくなりえるわけですが、
それに応じて n² が十分大きくなっている可能性を排除する必要があります。なかなか難しそうですが如何でしょうか?
0930132人目の素数さん
垢版 |
2020/12/14(月) 21:23:13.97ID:24RiWxhe
球の極座標変換x=rsinθcosφ、y=rsinθsinφ、z=rcosθを用いてyを積分します。
それぞれの範囲は大丈夫です。ヤコビアン行列からyがr^3sin^2θsinφに変換できると思ったのですが、違うみたいです。
何が違うのでしょうか。
0932132人目の素数さん
垢版 |
2020/12/14(月) 22:10:58.65ID:zLO82iyF
>>929
>>908に貼ったpdfを参考にしてください
0933132人目の素数さん
垢版 |
2020/12/14(月) 22:15:42.74ID:24RiWxhe
>>931
読んでもよくわかりませんでした、、、
r、s、tの範囲は求められて、ヤコビアンがr^2sinθになるのまではわかったのですが、その先のyを積分するところがどうしてもわかりません。
y=rsinθsinφ、dxdydz=r^2sinθdrdsdt(rとθの範囲から絶対値外せる)となるはずなのですが、どこが違うんでしょうか。
0934132人目の素数さん
垢版 |
2020/12/14(月) 22:34:50.18ID:CcteXPuI
意味不明なんですよね、ぶっちゃけ

>>930
>それぞれの範囲は大丈夫です。ヤコビアン行列からyがr^3sin^2θsinφに変換できると思ったのですが、違うみたいです。


この計算もなんのこと言ってるのかさっぱりわかりませんよ?
0935132人目の素数さん
垢版 |
2020/12/14(月) 22:47:06.55ID:XXEfGQML
>>933
何かしらの領域Vがあって, ∫_V y dVを求めたいってことかな?
何を質問しているのか不明瞭だから何かしら解けない問題があるならそれを直接貼るとかした方が良いかと
0937132人目の素数さん
垢版 |
2020/12/14(月) 23:10:26.12ID:24RiWxhe
>>934
>>935
すみません、問題の説明がめちゃくちゃでした。
yを次の領域Dで積分します。D={(x,y,z)|x^2+y^2+z^2≦a^2、0≦x,y,z}
このとき、次の変数変換を行い計算をします。
x=rsinθcosφ、y=rsinθsinφ、z=rcosθ
0939132人目の素数さん
垢版 |
2020/12/14(月) 23:32:14.93ID:XXEfGQML
途中で書き込まれてしまった
∫∫∫_D y dxdydz
=∫_[0,a](∫_[0,π/2](∫_[0,π/2](r^3 sin^2θsinφ)dφ)dθ)dr
=(∫_[0,a] r^3 dr)(∫_[0,π/2] sin^2θ dθ)(∫_[0,π/2] sinφ dφ)
=a^4/4 * π/4 * 1
=πa^4/16
という計算になると思う
0940132人目の素数さん
垢版 |
2020/12/15(火) 00:36:58.94ID:ue7mAg93
>>932 全くの無関係な話ではなさそうですが...
よくある「無理数の連分数による近似は最良近似!」それ以上の内容は読み取れませんでした。
0941132人目の素数さん
垢版 |
2020/12/15(火) 00:42:04.89ID:ue7mAg93
そして、稠密定理の方は既に納得してるのです。
しかしこれで >>929 の級数が(たぶん)発散することが明らかとは言えませんね... という話です。
0942132人目の素数さん
垢版 |
2020/12/15(火) 00:49:10.34ID:oPucdSd+
>>939
そうなると思ったんですけど、その2行目のr^3 sin^2θsinφが違うみたいなんですよね
なのでどこか間違えているのかと思ったんですけど、わからないです
0944132人目の素数さん
垢版 |
2020/12/15(火) 01:33:58.62ID:hC1qHKwE
>>940
このpdfの23ページの式を使えばよい
0946132人目の素数さん
垢版 |
2020/12/15(火) 14:33:56.21ID:oPucdSd+
>>945
オンラインで出題されて、解けたら穴埋めをしていく、という形でこの問題が出題されました。
ヤコビアンの穴埋めや、r、θ、φの範囲の穴埋めもあり、変数変換したあとの積分も穴埋めする形であります。
最後の答えπa^4/16とヤコビアン、r、θ、φは合っていて、そこの部分は正解というふうに表示されます。
しかし、r^3 sin^2θsinφが不正解というふうに表示されてしまい、答えはわかりません。
0947132人目の素数さん
垢版 |
2020/12/15(火) 14:37:29.10ID:/X0jBh/s
答えあってるんなら、そのプログラムが間違ってるんじゃないですか?

適当に打ち直してみて、答えが上のものとずれてるなら、それはプログラムが間違ってるということですよ
0948132人目の素数さん
垢版 |
2020/12/15(火) 14:37:59.75ID:/X0jBh/s
最終的な答えあってるんなら、そのプログラムが間違ってるんじゃないですか?

適当に打ち直してみて、途中式の答えが上のものとずれてるなら、それはプログラムが間違ってるということですよ
0949132人目の素数さん
垢版 |
2020/12/15(火) 14:41:02.44ID:oPucdSd+
>>947
>>948
そうですよね、計算を間違えているわけではなくプログラムの不具合ですよね。自信がなかったのですが、自分が間違えてないと思えました。
ありがとうございました。
0950132人目の素数さん
垢版 |
2020/12/15(火) 14:56:29.18ID:ue7mAg93
>>943
一様性定理が使えるということでしょうか。もう少し詳しくお願いします。

>>944
そこから例えば
∃c>0 ∀N>0 ∃n>N |z-zₙ| < c/n²
これが導けるなら発散すると言えますが、ちょっと遠いように思います。
0952132人目の素数さん
垢版 |
2020/12/15(火) 15:11:33.79ID:OkjGLufS
>>950
何で分からんの
稠密性を示すときの議論を見れば言えるでしょう
0953132人目の素数さん
垢版 |
2020/12/15(火) 17:24:52.80ID:ue7mAg93
>>952 それって鳩の巣論法ですよね。
単位円周をN分割すると、zₖ は 1≦ k ≦ N のどこかで端っこの鳩の巣に落ちる。
つまり zₖ∈Arc[1] または Arc[N]
z∈Arc[m] (1≦ m ≦ N) の時、 n = m*k または (N-m+1)*k とすれば |z-zₙ| ≦ 1/N となる。
ここからどうにかなるのでしょうか?ほんと分からないので教えてください。

もし c/n ≦ 1/(n² |z-zₙ| ) のように絶対値で下限を抑えられたとしても交代級数的な収束は排除できませんよね。
0955943
垢版 |
2020/12/15(火) 17:48:38.14ID:5Y6mC4+Z
>>950
これを示すことか、示した後か、どっちが分からんのだ?
後の方なら平均が n なら逆数の和は全部が平均値の場合より大きくなるぞ
0956132人目の素数さん
垢版 |
2020/12/15(火) 18:25:35.97ID:ue7mAg93
>>955
"これ" とは
> e^{in} の間隔が平均的に n の反比例で減少すると示せば良い
の事でしょうか。すみません、これについては言ってる事の意味すら分かりませんでした。

(稠密定理ではなく)一様分布定理より |z-zₙ| < c となる確率は "平均的に" zやn に依らず P(c) と表せる。
つまり確率 P(c) で 1/(cn²) < 1/(n² |z-zₙ|) となる。 そういう類いの話ですか?
だとしてもその先が見えません。収束列で下を抑えても発散かどうかの判定には使えませんし。
0957132人目の素数さん
垢版 |
2020/12/15(火) 21:24:42.87ID:hC1qHKwE
>>953
君、1-1+1-1…を交代級数と思ってる?
0958132人目の素数さん
垢版 |
2020/12/15(火) 21:41:07.09ID:5Y6mC4+Z
>>956
n ≦ N までの e^{in} の密度の事を言ってる
e^{in} が 1周して z に最接近する e^{in} だけの部分列を考えると
| z - z_n | は e^{in} 間隔より小さいからな
0959132人目の素数さん
垢版 |
2020/12/15(火) 22:46:14.67ID:4L9X2FBx
>>957
これは恥ずかしい
部分和の符号ではなく各項の符号が正負交互に現れるものを交代(交項)級数と呼ぶ
1-1+1-1+…は交代級数だよ
0960132人目の素数さん
垢版 |
2020/12/15(火) 22:51:24.55ID:4L9X2FBx
あ、でも部分和の符号が正負交互に現れるものは明らかに交代級数だし、この意味で交代級数と定義してる危篤な本もあるのかな
0961957
垢版 |
2020/12/16(水) 00:25:50.39ID:JhiarpKk
ごめんごめん、「収束する」交代級数だと思ってる?
0963132人目の素数さん
垢版 |
2020/12/16(水) 01:11:16.91ID:9h6A2h4v
考え直してみたら>>929は{zₙ}が無理数回転であることは本質的ではないな。単に{zₙ}が円周上稠密であればよい。
実はC上の任意の領域が正則領域であることの証明を
単位開円板に適用したものになっている。
https://www.ms.u-tokyo.ac.jp/publication/documents/saito-lectures.pdf
の24〜25ページを参照
0964132人目の素数さん
垢版 |
2020/12/16(水) 01:30:48.98ID:/+MECXfG
pdfの方でD上稠密な可算列{Zn}から∂D上稠密な可算列{ζn}を作っているのは一般の領域Dに対して∂D上稠密な可算列を構成するため
0965132人目の素数さん
垢版 |
2020/12/16(水) 07:36:16.14ID:ewx45c0p
>>963 文献の紹介ありがとうございます。
指摘の箇所では、z が 内側から境界上の zₙ に真っ直ぐ近づいた時に発散することを述べていますね。
zₙ 以外の 境界上の z における収束性については触れていないように見えます。
0966132人目の素数さん
垢版 |
2020/12/16(水) 08:17:46.10ID:cA16A4Ot
∂Dの点z0を固定する。
z0に収束するζnの部分列をとり改めてζnとおく。
各nに対しζnに収束するDの点列zn,mでm→∞のときf(zn,m)→∞となるようなものをとる
このとき点列{zn,n}はz0に収束しn→∞のときf(zn,n)→∞
0967132人目の素数さん
垢版 |
2020/12/16(水) 09:15:14.58ID:cA16A4Ot
最後の行は正しくないな
各nに対し|f(zn,m(n))|>nとなるm(n)をとる
このとき点列{zn,m(n)}はz0に収束しn→∞のときf(zn,m(n))→∞
0968132人目の素数さん
垢版 |
2020/12/16(水) 10:09:02.10ID:ewx45c0p
>>967
なるほど、ここで境界の点 z での級数和が収束すると仮定するとアーベルの連続性定理より矛盾となり...
いや... この場合、ストルツ条件を保ったまま近づくとは限らないので何も言えないのでは...
要するに、内側からの近づき方が特殊だから発散するのであって真っ直ぐ近づいたら収束する(仮定と矛盾しない)のかもしれない。
そこをはっきりをさせないと収束性は分からないのでは? という事です。
0969132人目の素数さん
垢版 |
2020/12/16(水) 11:05:46.78ID:cA16A4Ot
教えてもらうふりをして教えようとしてくる人だったか
0970132人目の素数さん
垢版 |
2020/12/16(水) 11:52:21.26ID:ewx45c0p
>>969 いや本当に分からないから聞いてますよ「教えてもらうふり」じゃなくて。
まあ、あの文献のレベルで真の解答を示されても自分には理解できないかもしれませんが。
0971132人目の素数さん
垢版 |
2020/12/16(水) 18:33:44.65ID:vC6aTsA5
アーベルの定理ってべき級数の話でしょ?
0972132人目の素数さん
垢版 |
2020/12/17(木) 02:04:33.67ID:8WYwrHy/
a(t)=dv(t)/dt=ーA*v(t)+B v(0)=0 a(0)=B
で表されるとき積分とかして
速度v=なんとか* t
のようにtを含んだ式を求めたいのです どうなりますか?
Bがないときは v=C * e^(-At) になるようです
0973132人目の素数さん
垢版 |
2020/12/17(木) 07:41:39.76ID:aXnU/fZt
>>972 べき級数について定理なのは確かにそうでした。
それでも、>>966, >>967 の方針で進めるとして
・ lim(内側から近づく点列極限) と Σ(級数和) が交換可能 (収束するなら)、
 あるいは lim Σ が発散するなら、Σ lim も発散する。
・limの点列は経路に寄らない。
 あるいは >>967 の点列は素性の良い経路を辿ると見なせる。
この辺りが、明らかとは思えません。
0976132人目の素数さん
垢版 |
2020/12/23(水) 15:25:38.28ID:DVZqAJRR
C[x,y]/(x^2+y^2)≅C[x,y]/(xy)
は正しいのでしょうか?
よろしくお願いします。
0978132人目の素数さん
垢版 |
2020/12/23(水) 16:58:25.05ID:+TUB+VXn
正しい
0979132人目の素数さん
垢版 |
2020/12/23(水) 17:09:21.82ID:Np4GaYAi
C[x,y]においてz=x+iyとw=x-iyは代数的に独立で、この対応により同型C[x,y]=C[z,w]=C[x+iy,x-iy]を得る
このときイデアル(x^2+y^2)はイデアル(zw)に対応するので
同型C[x,y]/(x^2+y^2)=C[z,w]/(zw)を得る
0981132人目の素数さん
垢版 |
2020/12/24(木) 18:07:10.18ID:BLLu6S1v
ツイッター上のこの反映定理についてのやりとりなんですが、反映定理は構造の絶対性に関するもので、別にモデルの存在を保証する定理ではないですよね
この人、京大の講師みたいだけど、関係ないこと言ってますよね?
ZFCの無矛盾性が暗黙に仮定されてることこの場合は関係ないと思うのですが
https://twitter.com/ytb_at_twt/status/1068353172077330432?s=19
https://twitter.com/5chan_nel (5ch newer account)
0984132人目の素数さん
垢版 |
2020/12/27(日) 14:56:31.64ID:AQlSrRdH
Σ[m=0,...,n]m^k が簡潔な式で表記できないことの証明って分かりますか?
0985132人目の素数さん
垢版 |
2020/12/27(日) 16:03:27.62ID:oCthruqo
♪愛をー償ーえば〜 別れーに成るーけど〜

親父の世代の曲なんで後が分からない
確か柳葉敏郎が言うにはマザー・テレサの娘だとかって
0986132人目の素数さん
垢版 |
2020/12/27(日) 17:09:29.50ID:urEyqWb7
>>984
n(n+1)/2, n(n+1)(2n+1)/6, {n(n+1)/2}^2が簡潔ではないと?
0987132人目の素数さん
垢版 |
2020/12/27(日) 18:03:37.73ID:0E0H3F4m
二項係数から導く
Σ_{k=1〜n-r} k(k+1)…(k+r-1)/r! = n(n-1)…(n-r)/(r+1)!
の方が簡潔な式だと言いたいんじゃない?
0988132人目の素数さん
垢版 |
2020/12/27(日) 19:31:49.77ID:JtIEYzMm
いやどうみてもk乗の逆数和を(kとnを使って)求める簡潔な公式があるかどうか
0993132人目の素数さん
垢版 |
2020/12/27(日) 23:22:22.34ID:WGL7kcop
>>992
アレ?

でもEを頂点としてABCEを底面とする錐はABCE二つ分のコピーの貼り合わせでさらにそれを4つ合わせるとxy平面上の正方形|x|<1,|y|<1を底面として(0,0,1)を頂点とする錐のかたちになる
なのでそのコピーを6個頂点でピッタリ貼り合わせられる
この時頂点に集まってる立体角はもとの四面体の立体角の48個分だから一つ分の立体角は
4π÷48=π/12
じゃないの?
0994132人目の素数さん
垢版 |
2020/12/27(日) 23:43:12.32ID:WGL7kcop
もしかしてABCEとABCFが逆なんじゃない
ABCFの方の頂点の立体角は正八面体の頂点の立体角の1/4でacos(-1/3)-π/2でこちらが不可能の方じゃないの?
0995132人目の素数さん
垢版 |
2020/12/28(月) 06:57:07.89ID:IWz63S9v
積の形の素因数分解の定理はよく知られてるけど、
和の形の"素因数分解"って成り立つんかな?
0998132人目の素数さん
垢版 |
2020/12/28(月) 09:29:15.15ID:JVvWFjOu
その例だと2を19個足せば終わりやんけ

どんな自然数nも1のn個の和で書けるし、積と同様に1を"素因数"に含めないとしても2と3の和として2以上の自然数を表現可能(偶数なら2のみの和、奇数なら3をひとつ足せばいい)
n=p[1]…p[k]と素因数分解されるときにp[1]+…+p[k]=nが成り立つか?ということならもちろん成り立つわけがないし、和についての"素因数"と"素因数分解"をもうちょっと明確にしないと考察するに値しないよ
0999132人目の素数さん
垢版 |
2020/12/28(月) 09:38:58.43ID:TwE4aRKR
2進数表記の一意性みたいなしょうもない話にしかならんと思う
1000132人目の素数さん
垢版 |
2020/12/28(月) 09:49:41.33ID:IWz63S9v
そうかそうか
素数の定義自体が積に基づいてるから、素因数分解を和の形にしようって発想自体が浅いな
10011001
垢版 |
Over 1000Thread
このスレッドは1000を超えました。
新しいスレッドを立ててください。
life time: 166日 4時間 21分 47秒
10021002
垢版 |
Over 1000Thread
5ちゃんねるの運営はプレミアム会員の皆さまに支えられています。
運営にご協力お願いいたします。


───────────────────
《プレミアム会員の主な特典》
★ 5ちゃんねる専用ブラウザからの広告除去
★ 5ちゃんねるの過去ログを取得
★ 書き込み規制の緩和
───────────────────

会員登録には個人情報は一切必要ありません。
月300円から匿名でご購入いただけます。

▼ プレミアム会員登録はこちら ▼
https://premium.5ch.net/

▼ 浪人ログインはこちら ▼
https://login.5ch.net/login.php
レス数が1000を超えています。これ以上書き込みはできません。

ニューススポーツなんでも実況